revista (format .pdf, 5.8 mb)

94
RECREA Ţ II MATEMATICE REVIST Ă DE MATEMATIC Ă PENTRU ELEVI Ş I PROFESORI 10 ani de la apariţia revistei “Recreaţii Matematice” Asocia ţ ia “Recrea ţ ii Matematice” IA Ş I - 2009 Iulie Decembrie 2009 Anul XI, Nr. 2 1 i e

Upload: phungnguyet

Post on 28-Jan-2017

390 views

Category:

Documents


35 download

TRANSCRIPT

Page 1: Revista (format .pdf, 5.8 MB)

RECREAŢII MATEMATICE

RE V I S TĂ D E M A T E M A T I CĂ P E N T R U E L E V I Ş I P R OF E S O R I

10 ani de la apariţia revistei “Recreaţii Matematice”

A s o c i a ţ i a “ R e c r e a ţ i i M a t e m a t i c e ”

I AŞ I - 2 0 0 9

Iulie – Decembrie 2009Anul XI, Nr. 2

1ie

Page 2: Revista (format .pdf, 5.8 MB)

Semnificaţia formulei de pe copertă: Într-o formă concisă, formula 1ie leagă cele patru ramuri fundamentale

ale matematicii: ARITMETICA reprezentată de 1 GEOMETRIA reprezentată de ALGEBRA reprezentată de i ANALIZA MATEMATICĂ reprezentată de e

Redacţia revistei :

Petru ASAFTEI, Dumitru BĂTINEŢU-GIURGIU (Bucureşti), Temistocle BÎRSAN, Dan BRÂNZEI, Alexandru CĂRĂUŞU, Constantin CHIRILĂ, Eugenia COHAL, Adrian CORDUNEANU, Mihai CRĂCIUN (Paşcani), Paraschiva GALIA, Paul GEORGESCU, Mihai HAIVAS, Gheorghe IUREA, Lucian-Georges Lăduncă, Mircea LUPAN, Gabriel MÎRŞANU, Alexandru NEGRESCU (student, Iaşi), Gabriel POPA, Dan POPESCU (Suceava), Florin POPOVICI (Braşov), Maria RACU, Neculai ROMAN (Mirceşti), Ioan SĂCĂLEANU (Hârlău), Ioan ŞERDEAN (Orăştie), Dan TIBA (Bucureşti), Marian TETIVA (Bârlad), Lucian TUŢESCU (Craiova), Adrian ZANOSCHI, Titu ZVONARU (Comăneşti). Materialele vor fi trimise la una dintre adresele e-mail : [email protected] [email protected] [email protected] COPYRIGHT © 2008, ASOCIAŢIA “RECREAŢII MATEMATICE”. Toate drepturile aparţin Asociaţiei “Recreaţii Matematice”. Reproducerea integrală sau parţială a textului sau a ilustraţiilor din această revistă este posibilă numai cu acordul prealabil scris al acesteia.

TIPĂRITĂ LA SL&F IMPEX IAŞI Bd. Carol I, nr. 3-5 Tel. 0788 498933 E-mail: [email protected]

ISSN 1582 - 1765

Page 3: Revista (format .pdf, 5.8 MB)

Catre cititoridupa zece ani de aparitie a revistei

Cascada canta:Imi revars cu bucurie apele,daca un strop din ele ajunge ınsetatului

Rabindranath Tagore

In toamna anului 1999 apare la Iasi primul numar al revistei Recreatii Matematice.Continuandu-si aparitia ın mod constant cu doua numere anual, revista a atins zeceani de existenta ın primavara acestui an. Revista este gandita de initiatorii ei sa fieo continuatoare a vechii reviste Recreatii Stiintifice (1883–1888), de la care a preluatsi numele, putin modificat, noua revista avand un continut strict matematic.

Revista Recreatii Stiintifice este prima revista stiintifica din tara care s-a adresattineretului cu chestiuni din toate ramurile stiintei, dar cu un continut predominantmatematic; ın cuvantul de ınceput al celui de-al saselea an de aparitie se spune, cumodestie, dar si cu mandrie: Credem ca noi am tras cea ıntai brazda [...] Brazda′imica si ıngusta, dar exista!

Este o mare ındrazneala sa-ti propui a fi continuator al unei reviste care a facutopera de pionerat ın cultura romaneasca si a avut o contributie importanta ın educatiamatematica a tineretului. Preluand scopurile generoase ale creatorilor RecreatiilorStiintifice, actuale si astazi, dorinta noastra suprema este de a asigura noii reviste unnivel de calitate si de utilitate la ınaltimea celui avut de ”strabuna” sa.

Acum, la ımplinirea a zece ani de aparitie, Recreatiile Matematice este o revistaconsiderata printre primele publicatii de gen din ıntreaga tara. Fondatorii revis-tei Recreatii Matematice sunt (ın ordine alfabetica): T. Bırsan, C. Calistru, Al.Carausu, C. Cocea, A. Corduneanu, Gh. Iurea, care au format primul comitetde redactie. In timp, li s-au alaturat alti profesori si elevi. De Alin Spuma siMihai Gartan, disparuti din randurile noastre, ne vom aminti cu pietate, dar si curecunostinta pentru aportul lor entuziast la aparitia revistei ın perioada de ınceputa ei. Cu nr. 2/2001 intra ın redactie Gabriel Popa, care devine ın scurt timp unuldintre principalii realizatori ai revistei.

Revista este distribuita si are colaboratori, elevi si profesori ın toata tara: Barlad,Brasov, Bucuresti, Craiova, Galati, Harlau, Orastie, Pascani, Pitesti, Rm. Valcea,Satu Mare, Sf. Gheorghe (Tulcea), Suceava, Timisoara, Vaslui etc. Mentionam cativacolaboratori din tara atasati Recreatiilor Matematice: D. Popescu, F. Popovici, I.Serdean, M. Tetiva, D. Tiba, L. Tutescu, T. Zvonaru s.a.

Structura revistei comporta trei parti mari: I) evenimente din lumea matematicii(conferinte, aniversari, comemorari etc.), II) partea ”teoretica” cu rubricile: articolesi note, nota elevului, chestiuni metodice, istoria matematicii, corespondente s.a.,III) partea aplicativa cu rubricile: concursuri si examene, probleme si solutii, pa-gina rezolvitorilor. Mai sunt presarate ın cuprinsul numerelor de revista amuzamentematematice, diverse informatii, premii si elevi premiati.

Se gasesc prezentate ın paginile revistei figuri de mari matematicieni: P. Fermat,N.H. Abel, H. Poincare, L. Euler, A.N. Kolmogorov, W.R. Hamilton s.a., ilustre numeale matematicii romanesti: Spiru Haret, Gh. Vranceanu, Al. Myller, Gr. C. Moisil,

85

Page 4: Revista (format .pdf, 5.8 MB)

M. Haimovici s.a., probleme celebre ale matematicii din toate timpurile: postulatul Val lui Euclid, constructii cu rigla si compasul, problema celor patru culori, conjecturalui Poincare s.a., institutii si reviste care au contribuit la progresul ınvatamantuluisi cercetarii matematice romanesti: Seminarul Matematic ”Al. Myller” din Iasi, Ob-servatorul astronomic din Iasi, revista Recreatii Stiintifice, Revista Stiintifica ”V.Adamachi” s.a., evenimente importante: Congresele internationale ale matematicie-nilor romani din 2003 si 2007, cat si reflectarea altor aspecte ale vietii matematice.

Revista Recreatii Stiintifice, pe care ıncercam sa o continuam, a fost sarbatoritaın 2003, la 120 de ani de la aparitia sa, prin publicarea unor materiale omagiale, darsi a unor articole selectate din numerele acestei vechi reviste. A fost sarbatorita maiamplu ın 2008, la 125 de ani de la aparitie, ın cadrul unui simpozion desfasurat ınaula filialei Iasi a Academiei Romane. O importanta actiune a fost, reeditarea celorsase tomuri de Recreatii Stiintifice prin redarea textului originar cu mijloace modernesi prin producerea unei variante electronice a lui de catre firma Kepler Systemesd′Information (director Marinela Ghigea).

In rubricile partii ”teoretice” a revistei au fost publicate ın acesti 10 ani, un numarde 216 articole: 121 note matematice, 26 note ale elevilor, 34 chestiuni metodice, 10”corespondente” (articole primite din afara tarii), 9 articole adresate claselor primaresi 16 articole repartizate ın alte rubrici.

Datorita stradaniilor unor membri ai redactiei revistei, au fost atrasi, ca rezolvitorisau colaboratori cu note matematice, elevi talentati, olimpici, care, fara ındoiala, sevor afirma ın scurt timp ca valorosi cercetatori ın domeniul matematicii: GabrielDospinescu, Adrian Zahariuc, Marius Pachitariu, Irina Mustata, Vlad Emanuel,Cezar Lupu, Iurie Boreico (Chisinau), Marius Tiba, recompensati cu premii ın banisi diplome.

La rubrica Corespondente articolele au aparut ın limba ın care au fost redactate(romana, franceza si engleza) si provin de la corespondenti din tarile: RepublicaMoldova, Germania, Kazahstan, Franta si China.

Date statistice sumare dau o imagine asupra partii de probleme si solutii (toateproblemele au primit solutii dupa un an de la publicare): ciclul primar–173 probleme,gimnaziu–432, liceu–400, de tip G (avansate pentru gimnaziu)–165, de tip L (avansatepentru liceu)–165. La aceastea trebuie adaugate si problemele date la diferite concur-suri nationale care sunt prezentate ın revista editie de editie: Concursul ”Al. Myller”,Concursul ”Florica T. Campan”, Concursul ”Recreatii Matematice” s.a. Pentru treiaparitii la rubrica Pagina rezolvitorilor, elevii harnici sunt premiati cu cate o diplomasi carti; ın acest interval de 10 ani au fost premiati 234 de elevi rezolvitori, cei maimulti fiind din ciclul primar.

Eforturile colectivului de redactie au fost usurate datorita colaborarii unui numarmare de profesori din ıntreaga tara, care prin materialele oferite au contribuit laridicarea nivelului revistei.

Revista Recreatii Matematice este recenzata ın G.M.-A si ın revista de referateZentralblatt fur Didaktik der Mathematik (ZDM). Pagina web a revistei poate fivizitata la adresa: http://www.recreatiimatematice.ro

Redactia revistei

86

Page 5: Revista (format .pdf, 5.8 MB)

NECULAI GHEORGHIU

(1930-2009)

In luna ianuarie 2009, Profesorul nostru,Neculai Gheorghiu a plecat dintre noi pentrutotdeauna. Va sta de acum ıncolo la masa um-brelor.

Scriu aceste randuri, nu numai ın calitate dedecan al Facultatii de matematica din Iasi, fa-cultate pe care Profesorul Gheorghiu a slujit-o cudaruire si pasiune o viata, ci mai ales ın calitatede discipol al sau.

Am urmat doua cursuri tinute de domnia sa,cel de analiza matematica din semestrul I al anuluiI si cel de analiza functionala din anul al III-lea.In plus, Profesorul Gheorghiu a fost ındrumatorulstiintific al lucrarii mele de licenta. Ca asistentuniversitar, am facut multi ani la rand seminarulla cursul de analiza functionala din anul al III-lea.

Perioada studentiei mele a reprezentat o etapade varf a carierei Profesorului. Astfel, ın anul 1973 a aparut, ın colaborare cu prof.Teodor Precupanu, cursul litografiat Analiza matematica, publicat ın 1976 la Edituradidactica si pedagogica, iar ın 1974 a aparut, la Editura Academiei Romane, carteaIntroducere ın analiza functionala. In aceeasi perioada, 1972-1976, a fost prodecan alFacultatii de matematica a Universitatii ”Al.I. Cuza” Iasi, punand ın slujba acesteiacalitatile sale de bun organizator si om cu gandire limpede si de perspectiva. Anterior,fusese decanul Facultatii de matematica a Institutului Pedagogic de 3 ani din Iasi,unde a fost colaborator apropiat al prof. Ilie Popa, rectorul institutului, si a militatneobosit pentru o calitate superioara a ınvatamantului. A facut parte din comitetulde conducere al Seminarului matematic ”Al. Myller” din Iasi.

Profesorul Neculai Gheorghiu a fost ınzestrat cu un talent didactic de exceptie.Lectiile sale erau de o claritate, de o rigoare si de o naturalete deosebite. Nu ampredat decat lucruri simple, obisnuia sa spuna Profesorul; ın realitate, concepte cuun ınalt grad de abstractizare si teorii complexe deveneau simple si transparentedatorita harului sau pedagogic. Ele au contribuit decisiv la orientarea mea spredomeniul analizei matematice. Nu putini au fost si mai sunt aceia care, la randullor, au afirmat si afirma cu recunostinta ca, precumpanitor, si-au modelat gandireamatematica tocmai datorita acestor lectii.

Avea si un mod original de a se comporta cu studentii si cu cei din jur. Eradeschis, direct, sever si uneori dur, dar aceste asperitati ale firii sale erau completatesi compensate de bogatia sufletului sau si omenia sa. Fata de studenti dovedea largidisponibilitati de a-i asculta si ıntelege, dar si de a-i ajuta, ın masura ın care o

87

Page 6: Revista (format .pdf, 5.8 MB)

putea face. Era ıntotdeauna corect si cinstit. Toate acestea au facut ca ProfesorulNeculai Gheorghiu sa fie foarte mult apreciat si iubit de multe generatii de studenti.Aproape de fiecare data cand eram undeva unde se gaseau fosti studenti ai sai, apareaıntrebarea Ce mai face Profesorul Gheorghiu? sau, mai direct si mai sugestiv, Ce maiface Nae?, ca si cum Nae era un fost coleg sau un prieten comun.

Avea nostalgia locului natal; vorbea cu afectiune de Mileanca (fostul judet Doro-hoi), satul ın care s-a nascut si a urmat clasele primare si nu uita sa adauge cu mandrieca la Mileanca se afla cel mai bun cernoziom. Isi amintea cu placere de scolile pe carele-a urmat si de faptul ca a fost propunator de probleme la Gazeta Matematica (toatede geometrie, tinea sa precizeze).

O dovada a posibilitatilor ıntinse de comunicare a Profesorului NeculaiGheorghiu cu semenii sai din domenii diverse de activitate este implicarea sa ınviata sportiva a Iasului, la un moment dat avand functii de conducere ın baschetul sifotbalul iesean.

S-a preocupat si de ınvatamantul matematic preuniversitar. In repetate randuri aonorat ca presedinte comisia judeteana a Olimpiadei nationale de matematica, a tinutprelegeri pentru elevi, a propus probleme la concursuri si olimpiade si este coautorla o frumoasa culegere de probleme pentru elevi intitulata Matematici elementare;probleme de sinteza (Editura Junimea, Iasi, 1983). Evident, ca orice culegere deprobleme bine scrisa si aceasta contine rezolvari integrale si multe comentarii utile.Exista acolo o superba problema propusa de Profesorul Gheorghiu la etapa finala aOlimpiadei de matematica din 1963.

Profesorul Neculai Gheorghiu avea o inteligenta sclipitoare. A avut rezultatenotabile si ın activitatea de cercetare stiintifica, domeniul sau predilect de preocuparifiind acela al ecuatiilor diferentiale ordinare. Teza sa de doctorat, sub conducerea prof.dr. doc. Ilie Popa, cu titlul Despre comportarea asimptotica a solutiilor ecuatiilordiferentiale de ordinul doi a fost foarte bine apreciata. Rezultatele cu totul deosebitedin activitatea didactica ımpreuna cu rezultatele din activitatea de cercetare au facutca Profesorul Gheorghiu sa avanseze rapid ın cariera universitara, astfel ca la 40 deani era deja profesor universitar.

Profesorul Neculai Gheorghiu continua sa fie prezent printre noi si dupa dis-paritia sa fizica; este de gasit ın rafturile de carti ale Seminarului Matematic ”Al.Myller”, dar si pastrat cu recunostinta si iubire ın inimile numeroaselor promotii destudenti care l-au avut ca dascal.

Va ramane mereu ın amintirea noastra ca un profesor reprezentativ al Facultatiide matematica a universitatii iesene, cu o contributie importanta ın progresul ınva-tamantului matematic romanesc!

Prof. dr. Ovidiu CARJADecan al Facultatii de Matematica

Universitatea ”Al.I. Cuza” Iasi

88

Page 7: Revista (format .pdf, 5.8 MB)

Conjectura Beal pentru polinoame

Temistocle BIRSAN 1, Gabriel DOSPINESCU 2

Abstract. This paper deals with the solution of equation (1) in the set of polynomials withinteger coefficients and it has an informative aim. As well as in the case of Fermat′s Conjecture, thesolution to Beal′s Conjecture (i.e., the claim that Eq. (1) with p, q, r- integer numbers greater than2 has no solution with x, y, z positive and mutually prime) in the set C [X]is elementary (Proposition1). The triples (p, q, r) with p, q, r in N∗ such that equation (1) has solutions in C [X] are: (1, q, r),(2, 2, r), (2, 3, 3), (2, 3, 4), (2, 3, 5).

Keywords: polynomials, Beal′s Conjecture, Fermat′s Conjecture, regular polyhedra.

MSC 2000: 11D41.

In 1966, Andrew Beal instituie un premiu pentru demonstrarea sau infirmareaasa-numitei acum Conjecturi Beal [1]:

Ecuatia

(1) xp + yq = zr,

unde p, q, r sunt numere ıntregi mai mari ca 2, nu are nici o solutie cu x, y, z ıntregipozitivi si relativ primi.

Pentru p = q = r (= n) ecuatia devine

(2) xn + yn = zn,

ecuatie care a fost subiect de preocupari pentru lumea matematica ınca din anti-chitate si mai este si acum, desi a fost realizata rezolvarea ei completa. Scoala luiPitagora a demonstrat ca ecuatia x2+y2 = z2 admite o infinitate de triplete (x, y, z)formate cu numere ıntregi, pozitive si prime ıntre ele ce o verifica. Pierre Fermat aafirmat ca ecuatia (2) pentru n ≥ 3 nu admite nici o solutie cu numere x, y, z ıntregisi nenule si a notat pe marginea unei pagini a Aritmeticii lui Diofant ca poseda odemonstratie minunata a acestui fapt. Generatii de matematicieni, aflati pe urmeleacestei demonstratii, n-au reusit sa o gaseasca sau sa lamureasca care ar fi pututsa fie aceasta, ceea ce a facut pe unii sa presupuna ca Fermat a gresit pe parcursuldemonstratiei sale. In sfarsit, ın 1995, Andrew Wiles demonstreaza Marea teo-rema a lui Fermat, asa cum istoria matematicii retine afirmatia lui Pierre Fermat [6];demonstratia lui Wiles este, ınsa, accesibila unui cerc foarte restrans de specialisti.

In acest context, apare cu atat mai uimitor si remarcabil faptul ca rezolvareaecuatiei (2) ın multimea C [X] a polinoamelor cu coeficienti complecsi este elementara,

1Prof. dr., Catedra de matematica, Univ. Tehnica ”Gh. Asachi”, Iasi2Student, Ecole Normale Superieure, Paris

89

Page 8: Revista (format .pdf, 5.8 MB)

accesibila unui elev de liceu. Mai precis, cu ajutorul teoremei Mason - Stothers,enuntata mai jos, se poate dovedi afirmatia urmatoare:

Daca n ≥ 3, atunci ecuatia (2) nu are solutii ın C [X] cu polinoame neconstantesi relativ prime.

O prezentare a acestor fapte este facuta ın [4]; cititorii revistei sunt informatiasupra acestui subiect ın [2].

In aceasta ordine de idei, se impune de la sine ınlocuirea ecuatiei (2) cu (1) sirezolvarea acesteia din urma ın multimea C [X]. Este tocmai ceea ce ne propunem.Lasam, ınsa, cititorilor placerea de a rezolva Conjectura lui Beal propriu-zisa (ın Z)(informatii asupra premiului oferit de A. Beal sunt date ın [1]).

Vom apela si ın acest caz la

Teorema Mason - Stothers ([4], [2]). Fie f, g, h ∈ C [X] neconstante si relativprime. Daca are loc egalitatea f + g = h, atunci

(3) max deg f,deg g,deg h ≤ n0 (fgh)− 1

(pentru f ∈ C [X] cu descompunerea f (X) = αkQ

i=1

(X − ai)mi , se noteaza deg f =

m1 +m2 + · · ·+mk – gradul lui f si n0 (f) = k – numarul radacinilor sale distincte).

Demonstratia este elementara si poate fi gasita ın [2].

Propozitia 1 (Conjectura Beal pentru polinoame). Daca p, q, r ∈ Z sip ≥ 3, q ≥ 3, r ≥ 3, atunci ecuatia (1) nu are solutii ın C [X] cu polinoame necon-stante si relativ prime.

Demonstratie. Procedam ca si ın cazul ecuatiei (2). Presupunem ca ecuatia(1) cu exponentii p, q, r satisfacand conditiile din enunt admite o solutie (f, g, h) cupolinoame neconstante relativ prime. Ca urmare, putem aplica Teorema Mason –Stothers polinoamelor (f (X))

p, (g (X))

q, (h (X))

rsi scrie

deg (f (X))p ≤ n0 (f (X))

p(g (X))

q(h (X))

r − 1,

de undep deg f (X) ≤ n0 (f (X) g (X)h (X))− 1

si, deci,p deg f (X) ≤ n0 (f (X)) + n0 (g (X)) + n0 (h (X))− 1.

Scriind si inegalitatile analoage acesteia relativ la polinoamele g si h si adunandu-lemembru cu membru, obtinem relatia

(p− 3) deg f (X) + (q − 3) deg g (X) + (r − 3) deg h (X) ≤ −3,

care este falsa, caci p ≥ 3, q ≥ 3, r ≥ 3, si demonstratia este ıncheiata.

Inainte de a vedea ce se ıntampla atunci cand unul (cel putin) dintre exponentiiecuatiei (1) este mai mic ca 3, vom face cateva

90

Page 9: Revista (format .pdf, 5.8 MB)

Observatii. 1) In prezenta egalitatii f + g = h, faptul ca polinoamele f, g, h suntprime ıntre ele este echivalent cu conditia ca f, g, h sa fie relativ prime doua catedoua.

2) Daca εp1 = 1, εq2 = 1 si εr3 = 1 si (x, y, z) este o solutie ın C [X] a ecuatiei(1), atunci sunt solutii ale acestei ecuatii si tripletele: (ε1x, ε2y, ε3z), (x, ε2y, ε3z),(x, y, ε3z) etc. Vom face abstractie de aceste solutii derivate ale unei solutii gasite.

3) Fara a restrange generalitatea, putem considera ca ın tripleta (p, q, r) a exponen-tilor ecuatiei (1) avem

(4) p ≤ q ≤ r.

Intr-adevar, putem lua p ≤ q schimband, eventual, x si y ıntre ele ın ecuatia (1).Daca r ≤ p, scriem (1) sub forma zr + (ε1x)

p= yq, unde εp1 = −1. Daca p ≤ r ≤ q,

punem (1) sub forma xp + (ε2z)r= (ε3y)

q, unde εr2 = εq3 = −1.

Conform Propozitiei 1 si tinand seama de (4), rezulta ca p ∈ 1, 2.

I Cazul (1,q,r) este banal: solutia generala a ecuatiei

(5) x+ yq = zr

este data de

(6) x = hr − gq, y = g, z = h, ∀g, h ∈ C [X] ,

pentru orice exponenti q, r ∈ N∗.

II Cazul (2,q,r). Cu teorema Mason - Stothers vom obtine limitari importanteın privinta exponentilor q si r. Fie (x, y, z) o solutie ın C [X] a ecuatiei x2 + yq = zr

(cu x, y, z neconstante si relativ prime) si fie a = deg x, b = deg y, c = deg z. Teoremaamintita ne spune ca

max 2a, qb, rc ≤ n0

x2yqzr

− 1,

de undemax 2a, qb, rc ≤ a+ b+ c− 1,

deci avem

(7) a < b+ c− 1 si max qb, rc ≤ 2 (b+ c− 1) .

Cum max qb, rc ≥ 1

2(qb+ rc) ≥ q

2(b+ c), combinand cu a doua relatie din (7) vom

obtineq

2(b+ c) ≤ 2 (b+ c− 1), ceea ce conduce la q ∈ 2, 3.

II.1 Subcazul (2,2,r), cu r ≥ 2. Ecuatia

(8) x2 + y2 = zr

se mai scrie(x+ iy) (x− iy) = zr

91

Page 10: Revista (format .pdf, 5.8 MB)

si cum x+ iy si x− iy sunt polinoame prime ıntre ele rezulta ca fiecare este puterea deexponent r a unui polinom: x+ iy = f r si x− iy = gr. In final, ecuatia x2 + y2 = zr

are solutia data de

(9) x =1

2(f r + gr) , y =

1

2i(f r − gr) , z = fg, ∀f, g ∈ C [X] .

Verificarea faptului ca (9) este o solutie a ecuatiei (8) este imediata.

II.2 Subcazul (2,3,r), cu r ≥ 3. Din (7), avem max 3b, rc ≤ 2 (b+ c− 1), dincare rezulta ca

b < 2 (c− 1) si rc < 6 (c− 1) ,

deci r ∈ 3, 4, 5. Asadar, au ramas trei situatii de analizat: (2, 3, 3), (2, 3, 4) si(2, 3, 5).

II.2.1 Ecuatia x2 + y3 = z3. Vom vedea ca rezolvarea acestei ecuatii se reducela un caz anterior studiat. Intr-adevar, scriem ecuatia ın forma

(10) (z − y) (z − εy)z − ε2y

= x2,

unde ε2 + ε + 1 = 0. Intrucat z − y, z − εy si z − ε2y sunt relativ prime doua catedoua, din egalitatea precedenta deducem ca fiecare dintre acestea este patratul unuipolinom:

z − y = f2, z − εy = g2, z − ε2y = h2.

Pentru ca acest sistem liniar ın z si y sa fie compatibil, impunem polinoamelor f , g,h conditia

−εf2 + (1 + ε) g2 = h2.

Aceasta, ınsa, se reduce dupa substitutii evidente la ecuatia x2 + y2 = z2, care serezolva conform cazului II.1.

II.2.2 Ecuatia x2 + y3 = z4. Procedam ca ın cazul precedent. Scriem ecuatia ındiscutie sub forma

(11)z2 − x

z2 + x

= y3.

Cum z2 − x si z2 + x sunt relativ prime, urmeaza ca ele sunt cuburi de polinoame:

z2 − x = f 3, z2 + x = g3.

Astfel, suntem condusi la ecuatia 2z2 = f3 + g3 care se reduce la randul ei la ecuatiax2 + y3 = z3 ıntalnita ın subcazul II.2.1. Practic, solutiile ecuatiei x2 + y3 = z4 vorfi obtinute parametric si se vor exprima ın functie de solutiile cazului (2, 2, r).

II.2.3 Ecuatia x2 + y3 = z5. Rezolvarea acestei ecuatii pare sa fie deosebit dedificila. In lipsa unei descompuneri de tipul (10) sau (11), nu putem proceda ca maisus.

92

Page 11: Revista (format .pdf, 5.8 MB)

Stabilim doar faptul ca ecuatia are solutii. Se poate verifica direct ca tripleta(x, y, z) data de

x = X30 + 522X25 − 10005X20 − 10005X10 − 522X5 + 1,

y = −X20 + 228X15 − 494X10 − 228X5 − 1,(12)

z =5√1728

X11 + 11X6 −X

este o solutie a acestei ecuatii ([3] , [5]).

Observatie. In [3], Felix Klein pune ın evidenta legatura stransa care existaıntre poliedrele regulate si solutiile ın C(X) ale ecuatiei (1). Astfel, cazul (2, 3, 3)este legat de tetraedrul regulat, cazul (2, 3, 4) de cub si octogonul regulat, iar cazul(2, 3, 5), cu solutia (12), de dodecaedrul si de icosaedrul regulat.

Aceasta legatura este mentionata si discutata ın [5].

In concluzie, tripletele de exponenti (p, q, r), p, q, r ∈ N∗, pentru care ecuatia (1)are solutii ın C [X] sunt: (1, q, r), (2, 2, r), (2, 3, 3), (2, 3, 4), (2, 3, 5) si toate per-mutarile acestora.

Bibliografie

1. *** - Beals Conjecture, The New Zeland Math. Mag., 35(1998), no.2, 38.2. T. Bırsan - Marea teorema a lui Fermat pentru polinoame, RecMat - 1/2004, 5-9.

3. F. Klein - Vorlesungen uber das Ikosaeder und die Auflosung der Gleichungen vomfunften Graden, Teubner, Leipsig, 1884.

4. S. Lang - Math Talks for Undergraduates, Springer, 1999.

5. V. V. Prasolov - Essays on Numbers and Figures, Amer. Math. Soc., MathematicalWorld, v. 16, 2000.

6. A. Wiles - Modular elliptic curves and Fermat ′s Last Theorem, Annals of Math.,142(1995), 443-551.

In 1962 Bachet de Meziriac publica o versiune latina a Aritmeticii ce includeapeste o suta de probleme si avea margini largi ale textului. Pe paginile unui astfelde exemplar, Pierre Fermat ısi nota solutiile, comentariile si rezultatele proprii. Pemarginea Cartii a II-a Fermat nota afirmatia :

ecuatia xn + yn = zn nu are solutii ın numere ıntregi si nenule pentru n ≥ 3.

cunoscuta acum ca Marea Teorema a lui Fermat. Apoi a scris comentariul:

(continuare la pagina 20)

93

Page 12: Revista (format .pdf, 5.8 MB)

Teorema lui Brouwer - un caz particular elementar

Cornelia-Livia BEJAN 1

Abstract. By following the ideas developed by T. Traynor in [2], an elementary proof ofBrouwer′s fixed point theorem is presented for the restricted case of continuously differentiablefunctions.

Keywords: closed ball, ball, fixed point, continuously differentiable function.

MSC 2000: 54H25.

Teorema de punct fix a lui Brouwer are o istorie lunga. Ideile ce conduc lademonstratia acesteia se gasesc la Henri Poincare ınainte de 1886. L.E.J. Brouwera demonstrat teorema pentru n = 3 ın 1909. In 1910 J. Hadamard da primademonstratie pentru n arbitrar, iar Brouwer da o alta ın 1912.

In [2], autorul a dat o prezentare elementara a teoremei de punct fix a lui Brouwer.In aceasta nota, adaptand demonstratia elementara data ın [2], ne propunem saprezentam un caz particular al teoremei lui Brouwer, ıntr-o forma si mai accesibilaprofesorilor si studentilor.

Peste tot vom nota cu D discul unitate din plan si cu C frontiera sa, adica cercul.Teorema de punct fix a lui Brouwer afirma ca orice aplicatie continua f : D → Dadmite macar un punct fix, adica exista macar un element x ∈ D astfel ıncat f(x) = x.Altfel spus, daca deformam ın mod continuu discul unitate ın el ınsusi, atunci existamacar un punct care nu-si schimba pozitia.

Vom prezenta un caz particular al acestei teoreme (ipoteze ıntarite) si anume:

Teorema. Daca f : D → D este o functie cu derivatele de ordinul ıntai continue,atunci ea admite macar un punct fix.

Lema. Nu exista nici o aplicatie f : D → C cu derivatele de ordinul ıntai continuecare sa lase fixe toate punctele cercului, adica f(x) = x, ∀x ∈ C.

Demonstratie. Presupunem ca ar exista o astfel de aplicatie f si notam g(x) =f(x)− x, ∀x ∈ D. Constatam ca g este o functie cu derivatele de ordin ıntai continuesi, ın consecinta, majorate (ın norma) de o constanta k. Din teorema valorii mediiavem ∥g(x) − g(y)∥ ≤ k∥x − y∥, ∀x, y ∈ D, adica distanta ıntre g(x) si g(y) estemajorata de distanta dintre punctele x si y, multiplicata cu k.

Definim aplicatia ft(x) = x + t · g(x) = (1 − t)x + tf(x), ∀t ∈ [0, 1]. Se verificafaptul ca ft lasa fixe punctele cercului, ∀t ∈ [0, 1]. Aratam ca ft este injectiva pentru0 ≤ t < 1

k . Intr-adevar, daca ft(x) = ft(y), atunci ∥x − y∥ = t∥g(x) − g(y)∥ ≤tk∥x− y∥, de unde obtinem x = y ıntrucat tk < 1.

In plus, pentru fiecare t ∈ [0, 1] fixat, avem f ′t(x) = I+tg′(x), unde I este aplicatia

identica a planului. Pentru 0 ≤ t < 1k se vede ca aplicatia ft duce discul D ın el ınsusi,

adica ft(D) = D. In continuare sa scriem:

π = Aria(D) = Aria(ft(D)) =

ZDdetf ′

t(x)dx, 0 ≤ t <1

k.

1Prof.dr., Catedra de matematica, Univ. Tehnica ”Gh. Asachi”, Iasi

94

Page 13: Revista (format .pdf, 5.8 MB)

Deci membrul drept al relatiei precedente este constant ın raport cu t. Mai mult,cantitatea de sub integrala fiind un polinom ın t, rezulta ca prin integrare se obtinetot un polinom ın t. Din faptul ca membrul drept este pe de o parte o constantasi pe de alta parte un polinom ın t, urmeaza ca valoarea sa este o constanta pentruorice t ∈ [0, 1], nu numai pentru t ∈ [0, 1

k ). Trecand la limita pentru t → 1 ın relatiaprecedenta si tinand cont de f(D) = C, gasim:

π = AriaD = limt→1

Aria(ft(D)) = Aria(f(D)) = Aria C = 0,

ceea ce este fals. Contradictia ne arata ca Lema este adevarata.

Demonstratia Teoremei. Presupunem ca aplicatia din ipoteza teoremei nu arepuncte fixe. Atunci din inegalitatea Cauchy-Buniakowski-Schwarz se obtine

f(x) · x ≤ ∥x∥∥f(x)∥ ≤ 1, ∀x ∈ D.

Egalitatea s-ar atinge cand x si f(x) ar fi liniar dependente. Cum membrul drept ≤ 1,daca membrul stang f(x)·x ar fi = 1, am avea egalitate, ın care caz am avea f(x) = αxcu α ∈ R si ∥x∥ = 1. Deci α = αx · x = 1, de unde f(x) = αx = x; deducem ca f aravea puncte fixe, ceea ce contrazice presupunerea facuta. In concluzie, egalitatea nuse atinge si deci avem x · f(x) < 1, ∀x ∈ D.

Construim o aplicatie h(x) = x− 1− x · x1− x · f(x)

f(x), ∀x ∈ D. Aceasta aplicatie este

bine definita ıntrucat 1− x · f(x) > 0. Evident, aplicatia f are derivatele de ordinulıntai continue, iar h(x) = x, ∀x ∈ C. Pe de alta parte, avem:

Caz 1. Daca (x · f(x))x = (x · x)f(x), atunci

h(x) =x− (x · f(x))x− f(x) + (x · x)f(x)

1− x · f(x)=

x− f(x)

1− x · f(x)= 0,

deoarece am presupus ca f nu are puncte fixe.Caz 2. Daca (x·f(x))x = (x·x)f(x) rezulta ca x si f(x) nu sunt liniar dependente.

Dupa modul cum este definita aplicatia h, deducem ca h(x) = 0 caci ın caz contrar xsi f(x) ar fi liniar dependente.

In concluzie, constatam ca h nu se anuleaza si deci putem defini o aplicatie H(x) =h(x)

∥h(x)∥, ∀x ∈ D. Se vede ca H : D → C are derivatele de ordin ıntai continue si

fixeaza punctele cercului, adica H(x) = x, ∀x ∈ C. In baza Lemei, ınsa, am ajuns lao absurditate, ıntrucat o astfel de aplicatie H nu exista. Demonstratia este ıncheiata.

Observatie. O generalizare de la plan la spatiul cu n dimensiuni se face urmandpas cu pas calea parcursa mai sus. In liceu, elevii ıntalnesc teorema lui Brouwer ıncazul unidimensional: o functie f : [0, 1] → [0, 1] continua are cel putin un punct fix.

Bibliografie

1. L.E.J. Brouwer - Uber Abbildungen von Mannigfaltigkeiten, Math. Ann., 71(1912),97-115.

2. T. Traynor - An Easy Analitic Proof of Brouwer′s Fixed Point Theorem, Atti Sem.Mat. Fis. Univ. Modena, XLIV (1996), 479-483.

95

Page 14: Revista (format .pdf, 5.8 MB)

Exponentul de triangularitate al unui triunghi

Dorin MARGHIDANU1

Abstract. The following problem is investigated: if a, b, c denote the lengths of the sides of

a triangle, it is required to determine the values of the real and positive exponents α such that

the powers aα, bα, cα can still be the side lengths of a triangle. It is introduced the notion of

triangularity exponent t and it is proved that t = 2 for the right-angled triangles, t ∈(1, 2) for the

obtuse-angled triangles and t > 2 for the acute-angled triangles (Proposition 2).

Keywords: triangularity exponent.

2000 MSC: 51M15.

In aceasta nota dam un raspuns la urmatoarea ıntrebare:

Daca a, b, c sunt lungimile laturilor unui triunghi, pentru care numere α reale sipozitive puterile aα, bα, ca pot forma de asemenea un triunghi?

In prima parte a notei prezentam instrumentul algebric de lucru – o extindere ainegalitatii 11.19 din [1], p.99, iar ın cea de-a doua introducem exponentul de trian-gularitate al unui triunghi, notiune necesara rezolvarii problemei propuse.

1. La fel cum a fost demonstrata inegalitatea 11.19 din [1], putem stabili si

Propoztia 1. Daca pentru numerele reale strict pozitive a, a1, a2, . . . , an existaun numar real α > 0 astfel ıncat aα = aα1 + aα2 + . . .+ aαn, atunci au loc:

(1) 1) aβ > aβ1 + aβ2 + . . .+ aβn, ∀β ∈ R, β > α;

(2) 2) aβ < aβ1 + aβ2 + . . .+ aβn, ∀β ∈ R, 0 < β < α.Demonstratie. Din relatia de conditie rezulta ca a > a1, a > a2, . . . , a > an.1) β > α implica aβ−α > aβ−α

1 , . . . , aβ−α > aβ−αn si avem

aβ = aβ−α · aα = aβ−α(aα1 + . . .+ aαn) = aβ−α · aα1 + . . .+ aβ−α · aan> aβ−α

1 · aα1 + . . .+ aβ−αn · aαn = aβ1 + . . .+ aβn.

2) β < α implica aβ−α < aβ−α1 , . . . , aβ−α < aβ−α

n si, inversand semnul de inegali-tate ın calculul de la punctul 1), obtinem (2).

Observatii. 1. Relatia (1) ramane valabila si ın conditia aα > aα1 +aα2 + . . .+aαn.2. Pentru n = 2, α = 2 si β ∈ N, β > 2, punctul 1) revine la afirmatia 11.19 [1].

Corolar. Daca a, b, c sunt lungimile laturilor unui triunghi si este verificata con-ditia aα = bα+cα, atunci (aβ , bβ , cβ) formeaza un triunghi daca si numai daca β < α.

Vom indica doua aplicatii directe, ın geometrie, ale rezultatelor precedente.

Aplicatia 1. Intr-un paralelipiped dreptunghic de muchii a, b, c si diagonala mareD, avem Dp > ap + bp + cp, ∀p ∈ R, p > 2.

Demonstratie. Fie d diagonala fetei de laturi a, b. Conform punctului 1) alPropozitiei 1 (cu n = 2, α = 2), aplicat de doua ın cazul particular al triunghiuluidreptunghic avem Dp > dp + cp > ap + bp + cp.

1Prof. dr., Liceul Teoretic din Corabia, [email protected]

96

Page 15: Revista (format .pdf, 5.8 MB)

Aplicatia 2. Fie OABC un tetraedru tridreptunghic cu varful O si OA = a,OB = b, OC = c. Atunci ABp +BCp + CAp > 2(ap + bp + cp), ∀p ∈ R, p > 2.

Demonstratie. Cu Propozitia 1, aplicata la triunghiurile dreptunghice OAB,OBC si OCA, obtinem relatiile ABp > ap+ bp, BCp > bp+ cp si CAp > cp+ap care,adunate, conduc la inegalitatea din enunt.

2. Vom introduce o notiune, cu rol decisiv, prin urmatoarea

Definitie. Numarul real pozitiv t se numeste exponent de triangularitate al tri-unghului ABC cu laturi de lungimi a, b, c, daca tripleta (as, bs, cs) formeaza un tri-unghi pentru orice s ∈ R∗

+, s < t, si nu formeaza un triunghi pentru s ≥ t.

Rezolvarea problemei propuse este data de

Propozitia 2. Sunt adevarate urmatoarele afirmatii:1) t = 2, daca triunghiul este dreptunghic;2) t ∈ (1, 2), daca este obtuzunghic;3) t > 2, daca este ascutiunghic; ın acest caz, t este numar real (finit), daca

triunghiul nu-i isoscel sau daca este isoscel si unghiul opus bazei sale este strict maimare de 60 si t = +∞, daca triunghiul este isoscel si are unghiul opus bazei mai micsau egal ca 60.

Demonstratie. Fie ABC cu a = maxa, b, c. Consideram functia continuaf(x) = ax − bx − cx, x > 0.

1) Triunghiul fiind dreptunghic, avem a2 = b2 + c2. Conform Corolarului de maisus, deducem ca t = 2.

2) In acest caz, f(1) = a− b− c < 0 si f(2) = a2 − b2 − c2 > 0 (triunghiul ABCfiind obtuzunghic). Rezulta ca exista o valoare t∈(1, 2) (unica, conform Propozitiei1) astfel ıncat f(t) = 0, adica at = bt + ct. Din nou apeland la Corolar, deducem cat este exponentul de triangularitate al triunghiului.

3) Avem f(2) = a2 − b2 − c2 < 0 (triunghi ascutiunghic) si f(∞) = limx→∞

f(x) =

limx→∞

ax1−

b

a

x

− ca

x= +∞, daca triunghiul nu-i isoscel sau daca este isoscel

si baza sa este mai mare ca laturile sale (echivalent, masura unghiului din varf estestrict cuprinsa ıntre 60 si 90). Deci, exista t ∈ (2,∞) unic astfel ıncat at = bt + ct.Conchidem ca t astfel gasit este exponentul de triangularitate ın subcazul considerat.

In sfarsit, daca triunghiul este isoscel cu baza mai mica sau cel mult egala culaturile sale (echivalent, unghiul din varf are masura mai mica sau cel mult 60),atunci se constata direct ca puterile de exponent α ale laturilor acestuia formeaza untriunghi de acelasi tip, oricare ar fi α ∈ R. Asadar, ın acest subcaz avem t = +∞.

Consideratiile precedente sugereaza examinarea triunghiurilor care verifica conditiica a3 = b3 + c3 (sau

√a =

√b +

√c etc.), asa cum s-a facut ın cazul triunghiurilor

dreptunghice (a2 = b2 + c2) sau altor triunghuiuri speciale (triunghiuri mediane,triunghiuri cu laturi ın progresie aritmetica etc.).

Bibliografie

1. O. Bottema, R.Z. Djordjevic, R.R. Janic, D.S. Mitrinovic, P.M. Vasic –Geometric inequalities, Wolters-Noordhoff, Groningen, 1969.

97

Page 16: Revista (format .pdf, 5.8 MB)

Asupra unui sir de integrale Riemann

Dan POPESCU 1

Abstract. This Note is an extension of a former paper–[3] in the reference list. The authorremarks that many problems proposed as topics to various contests are direct consequences of Propo-sitions 1 and 2 in the note.

Keywords: continuous function, periodic function, Rimann integral.

MSC 2000: 26A42.

In cele ce urmeaza, sunt prezentate doua conditii suficiente de convergenta a siruluireal de integrale Z b

af (x) g (nx) dx

n∈N

,

unde f si g sunt functii integrabile Riemann care asigura corectitudinea definiriisirului.

Un prim rezultat este prezentat ın [3]:

Propozitia 1. Fie f : [0, T ] → R o functie integrabila Riemann si g : [0,∞) → Ro functie cu perioada T >0, astfel ıncat restrictia g|[0,T ] este integrabila Riemann.Atunci

(1) limn→∞

Z T

0f (x) g (nx) dx =

1

T

Z T

0f (x) dx

Z T

0g (x) dx

.

O aplicatie directa a acestui rezultat este urmatoarea:

Fie f : R → (0,∞) o functie continua cu perioada 1. Atunci

limn→∞

Z 1

0f (x) · f (nx) dx =

Z 1

0f (x)

2

.

(Cristinel Mortici, etapa judeteana, 2003)

In lucrarea [2], se propune tot o consecinta directa a rezultatului (1):Fie functia f : R → R continua si periodica cu perioada T > 0. Sa se demonstreze

ca, pentru a, b ∈ R, a < b, avem

limx→∞

Z b

af (nx) dx =

b− a

T

Z T

0f (x) dx.

Tot ca o consecinta a rezultatului (1), este prezentata urmatoarea problema:

Fie functia continua f : [0, 1] → R. Sa se demonstreze ca sirul (an)n≥1, definit

de relatia an =

Z 1

0nx2 f (x) dx, este convergent si sa i se afle limita, ın functia f ,

unde x = x−max k ∈ Z; k ≤ x ,∀x ∈ R.(Octavian Purcaru, Lista scurta, O.N.M., 2003)

1Profesor, Colegiul National ”Stefan cel Mare”, Suceava

98

Page 17: Revista (format .pdf, 5.8 MB)

Demonstratie. Cum functia g : R → Rg (x) = x2 este periodica cu perioadaprincipala 1, au loc:

an = limn→∞

Z 1

0f (x) g (nx) dx =

Z 1

0f (x) dx

Z 1

0x2 dx

=

1

3

Z 1

0f (x) dx,

deoarece

Z 1

0x2 dx =

Z 1

0x2dx, functiile de integrat fiind egale pe [0, 1) .

In lucrarea [1], apare urmatoarea problema, semnata de Mihail Bencze:

Sa se calculeze limn→∞

Z 1

0t2 nt dt, unde xare semnificatia de mai sus.

Un al doilea rezultat util:

Propozitia 2. Fie functiile continue g : [0,∞) → R si f : [0, a] → R, unde a >0.Daca lim

x→∞g (x) = L ∈ R, atunci

(2) limn→∞

Z a

0g (nx) f (x) dx = L

Z a

0f (x) dx.

Demonstratie. Intr-adevar, daca h (x) = g (x)− L,∀x ∈ [0,∞), atunciZ a

0g (nx) f (x) dx =

Z a

0(h (nx) + L) f (x) dx =

Z a

0h (nx) f (x) dx+ L

Z a

0f (x) dx.

Notand nx = t, se obtine:Z a

0h (nx) f (x) dx =

1

n

Z na

0h (t) f

t

n

dt.

Cum ∃b > 0 astfel ıncat |f (x)| ≤ b,∀x ∈ [0, a] ,Z a

0h (nx) f (x) dx

≤ b

n

Z na

0|h (t)| dt = ab

na

Z na

0|h (t)| dt.

Daca H este o primitiva pentru |h| pe intervalul [0,∞) ,

limx→∞

R x0 |h (t)| dt

x= lim

x→∞

H (x)−H (0)

x= lim

x→∞|h (x)| = 0,

ceea ce asigura ca limx→∞

Z a

0h (nx) f (x) dx = 0.

Observatie. Relatia (2) se poate scrie si

(2′) limn→∞

Z a

0g (nx) f (x) dx =

lima→∞

1

a

Z a

0g (x) dx

Z a

0f (x) dx,

99

Page 18: Revista (format .pdf, 5.8 MB)

ceea ce justifica prezentarea celor doua siruri de integrale Riemann ımpreuna .

O aplicatie a acestui rezultat:

Fie functiile f : [0,∞) → R si g : [0, 1] → R, astfel ca limx→∞

f (x) = L ∈ R. Sa se

demonstreze ca

limn→∞

1

n

Z n

0f (x) g

xn

dx = L

Z 1

0g (x) dx.

(Laurentiu Panaitopol, etapa judeteana, 2003)

Demonstratie. Cu substitutia x = nt, limita devine

limn→∞

Z 1

0f (nt) g (t) dt = L

Z 1

0g (x) dx

si finalizarea este clara.

Propunem cititorului urmatoarele exercitii:

1. Sa se demonstreze ca

limn→∞

Z π

0e−x cosnx dx =

21− e−ππ.

2. Sa se calculeze limn→∞

Z 1

0nx [x] dx, justificandu-se ıntai existenta limitei, unde

[x] = max k ∈ Z|k ≤ x si x = x− [x] ,∀x ∈ R.3. Sa se demonstreze ca, pentru orice functie continua f : [0, a] → R, are loc

relatia

limn→∞

Z a

0e−nxf (x) dx = 0.

Bibliografie

1. D.M. Batinetu - Giurgiu s.a. – Analiza matematica. Probleme pentru clasa aXII-a, Editura Matrix, 2004.

2. R. Miculescu s.a. – Probleme de calcul integral, Editura GIL, 2005.

3. D. Popescu, F. Popovici – O generalizare a lemei lui Riemann, Recreatii Matem-atice, Iasi, 4(2002), nr.1, 12-13.

100

Page 19: Revista (format .pdf, 5.8 MB)

Dualitatea unor sume combinatorialeAndrei VERNESCU 1

Abstract. The main result consist in the combinatorial identity (6), where Ωn is the notation

in (1). Some connections of this identity with other combinatorial identities of similar kind, that are

known or are expected to be established, are formulated.

Keywords: combinatorial identity.

MSC 2000: 05A19.

1. In lucrarea [1], din numarul 2/2007 al acestei reviste, s-a aplicat o ideee foarte

eleganta pentru calculul unei sume care contine expresiile(2k − 1)!!

(2k)!!, k = 1, 2, 3, . . . , n.

In nota de fata venim ın continuarea lucrarii citate, prezentand calculul bazatpe aceeasi idee al altei sume, care ar putea fi considerata, din punctul de vedere almijloacelor folosite, ca fiind duala celei precedente.

Pentru facilitarea calculelor care vor urma, vom utiliza notatia

(1) Ωn =(2n− 1)!!

(2n)!!(n = 1, 2, 3, . . .).

(Am introdus aceasta notatie ın manuscrisul din 1987 al cartii [11], am publicat-opentru prima data ın articolul [6], am folosit-o ın toate editiile culegerii de probleme[7], ca si ıntr-o serie de lucrari: [8], [9], [11] si [13].)

Tot ın scopul scrierii mai simple a formulelor si calculelor care vor urma, estenecesar sa definim Ωn si pentru n = 0. Pentru aceasta, ın cazul n ≥ 1, sa amplificam

fractia din partea dreapta a egalitatii (1) cu (2n)!! = 2nn!. Obtinem Ωn =(2n)!

(2nn!)2.

Ca aceasta formula sa poata fi prelungita si pentru n = 0, convenim sa punem prindefinitie

(2) Ω0 = 1.

Cu aceasta notatie, identitatea care formeaza rezultatul principal al articoluluicitat [1] se scrie concentrat astfel:

(3)nX

k=0

(−1)kCknΩk = Ωn.

Ea poate fi considerata din doua puncte de vedere: ca formula de ınsumare com-binatoriala, precum si ca identitate referitoare la expresiile Ωk.

2. Obtinerea identitatii (3) se efectueaza ın [1] utilizand integralele:

(4) In =

Z π/2

0cosn x dx =

Z π/2

0sinn x dx,

1Conf. dr., Departamentul de Stiinte, Univ. ”Valahia”, Targoviste

101

Page 20: Revista (format .pdf, 5.8 MB)

pentru care I0 =π

2, I1 = 1 si pe care, folosind notatia (1) ımpreuna cu definitia (2),

putem sa le mai scriem sub forma:

(5) In =

8><>:π

2Ωk, daca n = 2k

1

Ωk· 1

2k + 1, daca n = 2k + 1.

(Gasirea, pe baza formulei de recurenta Ik =k − 1

kIk−2, k ≥ 2, a expresiilor integralei

In – scrise ınsa fara a folosi expresia Ωk – este reamintita ın [1].)Astfel, cu utilizarea notatiei mentionate pentru Ωn si a simbolului de ınsumareP, putem sintetiza obtinerea rezultatului principal din [1] astfel:

π

2Ωn = I2n =

Z π/2

0cos2n x dx =

Z π/2

0

1− sin2 x

ndx =

=

Z π/2

0

nX

k=0

(−1)kCkn sin2k x

!dx =

nXk=0

(−1)kCkn

Z π/2

0sin2k x dx

=

=nX

k=0

(−1)kCknI2k =

π

2

nXk=0

(−1)nCknΩk,

adicaπ

2Ωn =

π

2

nXk=0

(−1)kCknΩk,

de unde (3).Putem aplica acum exact aceeasi idee de calcul pornind de la integrala (4) de ordin

impar, I2n+1. Vom obtine:

1

Ωn· 1

2n+ 1= I2n+1 =

Z π/2

0cos2n+1 x dx =

Z π/2

0

1− sin2 x

ncosx dx

t=sin x===

t=sin x===

Z 1

0

1− t2

ndt =

Z 1

0

nX

k=0

(−1)kCknt

2k

!dt =

=nX

k=0

(−1)kCkn

Z 1

0t2k dt

=

nXk=0

(−1)kCk

n

2k + 1,

adica

(6)nX

k=0

(−1)kCk

n

2k + 1=

1

Ωn· 1

2n+ 1.

Aceasta identitate, care constituie rezultatul principal al lucrarii noastre, nu semai refera la expresiile Ωk ın interiorul sumei, ci contine pe Ωn doar la rezultat, fiind

102

Page 21: Revista (format .pdf, 5.8 MB)

(spre deosebire de (3), care admite doua interpretari) doar o identitate combinatoriala.Dar, din punct de vedere al procedeului de deducere folosit, ea constituie o identitateduala celei din [1], adica (3).

Insa, ca de multe ori ın domeniul identitatilor combinatoriale, identitatea gasitanu este noua! Intr-adevar, ın cartea lui H. W. Gould [2], la pag. 6, este prezentataidentitatea

nXk=0

(−1)kn

k

x

x+ k=

1x+nn

(x ∈ R)

ın care notatia consacrata

α

k

, pentru α ∈ R si k ∈ N∗, are semnificatia:

α

k

=

α(α− 1)(α− 2) · . . . · (α− k + 1)

1 · 2 · 3 · . . . · k,

cu

α

0

def== 1, iar daca α = n ∈ N si k ≤ n se obtin combinarile,

n

k

= Ck

n.

Introducand ın identitatea lui Gould x =1

2, se obtine (6).

3. Prin prisma identitatilor combinatoriale, egalitatea (6) ocupa pozitia a patradin urmatoarea succesiune de formule:

Sndef===

nXk=0

Ckn

k + 1=

2n+1 − 1

n+ 1; S(a)

ndef===

nXk=0

(−1)kCk

n

k + 1=

1

n+ 1;

Tndef===

nXk=0

Ckn

2k + 1= f(n); T (a)

ndef===

nXk=0

(−1)kCk

n

2k + 1=

1

Ωn· 1

2n+ 1,

unde a desemneaza o suma alternata, iar f(n) este o expresie neprecizata.

Gasirea sumelor Sn si S(a)n este binecunoscuta (o cale fiind integrarea identitatilor

(1+t)n =nX

k=0

Cknt

k, respectiv (1−t)n =nX

k=0

(−1)kCknt

k, pe intervalul [0, 1]). Obtinerea

sumei T(a)n a format obiectul prezentei note, expus ın sectiunea 2 si astfel, se mai pune

problema calculului sumei Tn. O alta problema de studiu ar putea-o constitui calculul

unei sume asemanatoare cu cea din (3), dar nealternata, anumenX

k=0

CknΩk.

Bibliografie

1. A. Corduneanu, Gh. Costovici – Un sir strans legat de sirul lui Wallis, RecreatiiMatematice, Anul IX, Nr. 2 iulie-decembrie 2007, 95-96.

2. H. W. Gould – Combinatorial identities, Morgantown Printing and Binding, Co.,1972.

3. R. L. Graham, D. E. Knuth, O. Patashnik – Concrete Mathematics, AddisonWesley Longman, Reading MA, 1994.

103

Page 22: Revista (format .pdf, 5.8 MB)

4. D. E. Knuth – The art of Computer Programming, Vol. 1, Addison Wesley Longman,Reading, MA 1977 (traducerea la Ed. Teora, Bucuresti).

5. J. Riordan – Combinatorial Identities, J. Wiley & Sons, New York, 1968.

6. L. Toth, A. Vernescu – Dezvoltarea asimptotica a sirului lui Wallis, G. M.-A, 11(1990), Nr. 1, 26-29.

7. A. Vernescu – Analiza matematica. Probleme. Calcul diferential, Editura Pantheon,Bucuresti, 1991.

8. A. Vernescu – Ordinul de convergenta al sirului lui Wallis, G. M.-A, 12 (1991), 7-8.

9. A. Vernescu – Asupra unui tip de relatie de recurenta, Revista Matematica dinTimisoara (seria a IV-a), nr. 4/2003, 8-14.

10. A. Vernescu – Siruri de Numere Reale, Ed. Univ. Bucuresti, 2004.

11. A. Vernescu – Numarul e si matematica exponentialei, Ed. Univ. Bucuresti, 2004.

12. A. Vernescu – The natural proof of the inequalities of Wallis type, Libertas Mathe-matica, 24 (2004), 183-190.

13. A. Vernescu, C. Mortici – New results in discrete asymptotic analysis, GeneralMathematics, to appear.

(continuare de la pagina 9)

Ma aflu ın posesia unei demonstratii minunate a acestei afirmatii,dar marginea paginii este prea stramta pentru a o cuprinde.

Care avea sa fie o provocare timp de peste 350 de ani pentru multe generatii dematematicieni.

Abia ın 1995 Andrew Wiles a demonstrat afirmatia, punand capat provocarii luiFermat. Drumul parcurs de lumea matematica pana la acest final a fost presarat detentative de demonstratie, esecuri, descoperiri epocale, drame si tragedii individuale,dezvaluiri senzationale ın mass-media etc.

N-au lipsit nici glumele pe marginea provocarii lui Fermat:

In statia de metrou de pe Eighth Street, New York a aparut inscriptia:

xn + yn = zn : nu exista solutie. Am descoperit o demonstratie cuadevarat remarcabila a acestui fapt dar nu pot s-o scriu fiindca-mivine trenul.

Sau catrenul:

”Pe untul meu sunt multe litere scrise”,Suparat un client la o masa racnise;”N-am avut loc, raspunse piccolo-ul Pierre,Nici pe margine, nici pe rafturile din frigider.”

(Simon Singh–Marea Teorema a lui Fermat, Humanitas, Bucuresti, 2005.)

104

Page 23: Revista (format .pdf, 5.8 MB)

Asupra problemei C.O: 5004 din G.M.Dan Mihai MOCANU 1

Abstract. In this Note, the problem C.O.: 5004 of the journal G.M. – 2/2009, p.103 is gene-ralized and a couple of simple results in a triangle are established; they deal with isogonal lines andangle trisectors.

Keywords: altitudes, angle bisector, isogonal lines, angle trisectors, symmedians.

MSC 2000: 51F20.

A

B C

O H

A1

A2

. .

Scopul prezentei note este de a stabili cateva rezultateıntr-un triunghi, legate de cevienele izogonale care sunt ınacelasi timp si trisectoare.

Fie ABC un triunghi oarecare si A1, A2 ∈ (BC).Se spune ca cevienele (AA1, (AA2 sunt izogonale dacaÖBAA1 ≡ ÖCAA2 si ca sunt trisectoare ale unghiului bA dacaÖBAA1 ≡ ×A1AA2 ≡ ÖA2AC.

Afirmatia urmatoare are o demonstratie imediata.

Propozitie. Daca (AA1 si (AA2 sunt ceviene izogonale si una din ele este bisec-

toarea unghiului dintre o latura si cealalta, atunci ele sunt trisectoarele unghiului bA.Demonstratie. Sa presupunem ca (AA1 este bisectoarea unghiului ÖBAA2 (la

fel demonstram daca (AA2 ar fi bisectoarea unghiului ÖA1AC). Avem, deci, ÖBAA1 ≡×A1AA2. Cum ÖBAA1 ≡ ÖA2AC (AA1 si AA2 sunt izogonale), rezulta ca ÖBAA1 ≡×A1AA2 ≡ ÖA2AC, ceea ce trebuia demonstrat.

Consecinta 1. (C.O.: 5004 din G.M.-2/2009, p.102). In triunghiul ascutitunghic

ABC se considera ınaltimea AH. Sa se arate ca, daca bisectoarea unghiului ÕBAHtrece prin centrul cercului circumscris triunghiului ABC, atunci semidreapta (AH

este trisectoare a unghiului ÕBAC.Laura Constantinescu

Demonstratie. Se stie ca ınaltimea coborata din A si diametrul ce are A ca oextremitate a sa sunt izogonale. Dar, prin ipoteza, acest diametru este bisectoarea

unghiului ÕBAH. Conform propozitiei precedente (AH este trisectoare a unghiului bA).Consecinta 2. Daca AH este ınaltime, O este centrul cercului circumscris tri-

unghiului ascutitunghic ABC, iar (AH este bisectoarea unghiului ÕOAC, atunci (AO

este trisectoare a unghiului ÕBAC.Demonstratie. Se procedeaza ca ın Consecinta 1.

Observatie. Daca una dintre (AH sau (AO este trisectoare, atunci si cealaltaeste trisectoare.

1Elev, cl. a IX-a, Colegiul National, Iasi

105

Page 24: Revista (format .pdf, 5.8 MB)

Urmatoarele rezultate pot fi privite ca reciproce ale consecintelor de mai sus.

Reciproca (a Consecintei 1). Daca (AA2 este o trisectoare a triunghiului ascu-

tit-unghic ABC si (AO este bisectoarea unghiului ÖBAA2, atunci A2 este ınaltime.Demonstratie. Din conditiile ipotezei, rezulta ca (AA2 si (AO fac cu laturile

triunghiului unghiuri de masuraA

3. Atunci si (AH, ca izogonala a lui (AO, face cu

AC un unghi de masuraA

3. Ca urmare (AA2 coincide cu (AH.

Reciproca (a Consecintei 2). Daca (AA1 este o trisectoare a triunghiului ascutit-

unghic ABC si (AH este bisectoarea unghiului ÖA1AC, atunci O ∈ (AA1.Demonstratie. La fel ca pentru reciproca precedenta.

Observatie. In loc de ”ınaltime-diametru”, putem lua o alta pereche de cevieneizogonale; de exemplu, ”mediana-simediana”. Trecerea rezultatelor de mai sus lanoua pereche se face cu usurinta.

In ıncheiere, vom rezolva o problema strans legata de cele de mai sus.

Problema. Fie un triunghi ABC, AB > AC. Fie (AM) mediana relativ lalatura (BC) si (AN) simediana corespunzatoare. Aratati ca, daca (AM sau (AN

este trisectoare a unghiului ÕBAC, atunciAB

AC= 2 cos

A

3.

Demonstratie. Ambele vor fi trisectoare. Cu ajutorul ariilor, avem

AB ·AM sinÖBAM = AC ·AM sinÖCAM,

de undeAB

AC=

sinÖCAM

sinÖBAM=

sin 2A3

sin A3

= 2 cosA

3.

In toamna anului 1999 a aparut primul numar al revisteiRecreatii Matematice.In anul acesta se ımplinesc 10 ani de la aparitia acesteia.

Scrieti numarul 2009 cu ajutorul numarului 10 si folosind numai operatiile deadunare si ımpartire !

Care este numarul maxim de operatii cu care puteti face acest lucru ?Dar cel minim ?

(Nu se accepta termeni nuli !)

Nota. Raspunsurile la aceste ıntrebari sunt date la pag.

106

Page 25: Revista (format .pdf, 5.8 MB)

Matrices a coefficients dans un corps finiAdrien REISNER1

Abstract. It is considered the set Ap of matrices of order 2 with their entries in Zp, defined byAp = a = λM + µI;λ, µ ∈ Zp, and some properties of this set are presented (Theorems 1,3,6,7,9).

Keywords: unitary ring, the order of an element, the field Zp, isomorphism.

MSC 2000: 15A33.

A etant un anneau unitaire d′elements neutre e on appelle ordre d′un elementinversible a de A le plus petit entier positif n del que an = e; dans ce cas l′ensembleGa = e, a, a2, . . . , an−1 forme un sous-groupe du groupe G des elements inversiblesde l′anneau A: l′ordre n de l′element a ∈ G est le cardinal du ce sous-groupe Ga.

Pour toute matrice carree M a coefficients dans un corps on designe par T et ∆respectivement les deux applications suivantes: T :M→T (M)=trace de la matrice M,∆ : M → ∆(M)=determinant de la matrice M .

p designant un nombre premier strictement superieur a 3, on considere le corpsfini Zp des classes residuelles modulo p. M et I etant les deux matrices suivantes acoefficients dans Zp:

M =

b4 b1−b1 b0 , I =

b1 b0b0 b1 ,

on considere l′ensemble Ap des matrices d′ordre 2 a coefficinets dans Zp defini par:

Ap = a = λM + µI;λ, µ ∈ Zp.

Theoreme 1. L′ensemble Ap est un anneau commutatif unitaire pour les operationsusuelles. De plus: Card Ap = p2.

Demonstration. On a immediatement M2 = b4M − I; on en deduit compte tenude la structure de l′ensemble M2(Zp) que Ap est un algebre associative et unitaire.Enfin, la commutativite se verifie directement. D′autre part, (I,M) etant une basede l′algebre Ap, on a Ap ≃ Z2

p et par suite: Card Ap = (Card Zp)2 = p2.

La proposition suivante se verifie immediatement par le calcul.

Proposition 2. Pour toute matrice A = λM + µI ∈ Ap, on a:

a) T (A) = b2(b2λ+ µ), ∆(A) = λ2 + µ2 + b4λµ;b) T (A2) = T 2(A)− 2∆(A).En particulier, T (A2) = b2(b7λ2 + µ2 + b4λµ).Theoreme 3. Pour A ∈ Ap, deux quelconques des conditions suivantes impliquent

la troisieme: a) T (A) = b0, b) ∆(A) = b1, c) A est une matrice d′ordre 4.

1Centre de Calcul E.N.S.T., Paris; e-mail: [email protected]

107

Page 26: Revista (format .pdf, 5.8 MB)

Demonstration. a) + b) ⇒ c) Le theoreme de Cayley-Hamilton appliquee a lamatrice A donne:

A2 − T (A)A+∆(A)I = O.

Il vient alors avec les hypotheses a) et b): A2 = −I et A4 = I. L′ordre de la matriceA devise donc 4 et comme A2 = I (p etant impair), 2 n′est pas multiple de cet ordre.Finalement, A est d′ordre 4.

a) + c) ⇒ b) Supposant les conditions a) et c) verifiees, on a:

A2 = −∆(A)I et I = A4 = ∆2(A)I.

On en deduit: ∆2(A) = b1, ∆(A) = −b1 soit ∆(A) = b1.b) + c) ⇒ a) Compte tenu de b) et c), A2 = T (A)A− I d′ou

I = A4 = T 2(A)A2 − b2T (A)A+ I = T (A)[T 2(A)− b2]A+ [b1− T 2(A)]I.

On peut distinguer deux cas: I A et I sout lies, i.e. A = µI. Dans ce cas, A2 = µ2I,A4 = µ4I, d′ou, A etant d′ordre 4: µ2 = b1, µ4 = b1, µ2 = −b1 et A2 = −I, T (A)A = 0.Si T (A) = b0, alors A = O d′ou T (A) = b0, impossible. Par suite T (A) = b0. II A, Iest une famille libre. Dans ce cas l′egalite I = T (A)[T 2(A) − b2]A + [b1 − T 2(A)]Iimplique: b1 = b1− T 2(A) d′ou T (A) = b0. Le theoreme est ainsi demontre.

On considere la suite des entiers Ykk≥0 definie par Y0 = 2 et Yk+1 = 2Y 2k − 1,

k ≥ 0, dont les premiers terms sont: Y0 = 2, Y1 = 7, Y2 = 97, Y3 = 18817, . . .

Theoreme 4. Pour tout k ≥ 0, on a 2Yk ∈ T (M2k).Demonstration. Par recurrence sur k. Pour k = 0 la propriete se verifie trivi-

alment puisque 2Y0 = 4 ∈ b4 = T (M). Supposant la propriete verifiee a l′ordre k,demontrons-la pour k + 1. On a immediatment d′une part: 2Yk+1 = 4Y 2

k − 2 =

(2Yk)2 − 2 et d′autre part T (M2k+1

) = T [(M2k)2] = T 2(M2k) − b2, compte tenude l′assertion b) de la Proposition 2 et puisque ∆(M) = b1. Donc, compte tenu de

l′hypothese de recurrence: 2Yk+1 = (2Yk)2 − 2 ∈ T (M2k+1

).

Theoreme 5. La matrice M est d′ordre 2k si et seulement si p|Yk−2.

Demonstration. Supposons la matrice M d′ordre 2k, i.e. M2k = I. On a:

M2 =

c15 b4−b4 −b1 et par suite l′ordre de M ne divise pas 2 soit k ≥ 2. En posant

alors A = M2k−2

on a, M etant d′ordre 2k : A2 = M2k−1 = I, A4 = M2k = I.A ∈ Ap est donc une matrice d′ordre 4 verifiant aussi ∆(A) = b1. Compte tenu du

theoreme 3, b) + c) ⇒ a), il vient alors: T (A) = b0 soit d′apres le theoreme precedent:2Yk−2 ∈ T (A) = b0, i.e. p divise 2Yk−2 et finalement p etant impair p|Yk−2.

Reciproquement, avec les memes notations si p|Yk−2 i.e. Yk−2 ≡ 0 (mod p), alorsT (A) = b0. Mais, comme ∆(A) = b1, le theoreme 3, a)+b) ⇒ c), montre que la matrice

A est d′ordre 4 soit M2k = I et M2k−1

= A2 = I. L′ordre de M divisant 2k mais non2k−1 est donc egal a 2k.

108

Page 27: Revista (format .pdf, 5.8 MB)

Theoreme 6. Les deux assertions suivantes sont equivalentes:

a) b3 n′est pas le carre d′un element de Zp;

b) Ap est un corps.

Demonstration. a) ⇒ b) Si b3 n′est pas le carre d′un element de Zp, alors pourA ∈ Ap :

∆(A) = b0 ⇒ A = O.

En effet, supposons ∆(A) = b0. Pour A ∈ Ap on a: ∆(A) = b0 = (λ + b2µ)2 − b3µ2.

µ = b0 impliquerait b3 = (λµ−1 + b2)2 ce qui est exclut par hypothese. Donc µ = b0 et∆(A) = b0 = λ2 soit: λ = b0 et finalement: A = O. Donc l′ensemble Ap est forme dela matrice nulle et de matrices inversibles dans M2(Zp).

A = O etant une matrice de Ap, l′application Ap → Ap definie par X → AX est

lineaire et injective, donc surjective (dimAp = 2). Pour A = O il existe B ∈ Ap telleque: AB = 1 soit A−1 = B ∈ Ap: les inverse des matrices non nulles de Ap sont dansAp.

b) ⇒ a) Nous allons montrer que, non a) ⇒ non b). Supposons qu′il existe a ∈ Zp

tel que a2 = b3. Dans ce cas: ∆(A) = [λ + (b2 − a)µ][λ + (b2 + a)µ]. Pour la matriceA = (a − b2)M + I(a = b2 puisque b22 − b3 = b1) on a ∆(A) = b0 et (Cayley-Hamilton)A(A−T (A)I) = O avec A = O et A = T (A)I. On en deduit que Ap n′est pas integreet par suite que Ap n′est pas un corps.

Theoreme 7. En supposant que b3 est un carre dans Zp:

a) M est semblable a une matrice diagonale;

b) Ap est isomorphe a l′anneau produit Zp × Zp;

c) dans Ap il y a p− 1 elements de determinant b1 et (p− 1)2 elements inversibles.

Demonstration. a) L′equation caracteristique de la matrice M s′ecrit X2−b4X+b1 = (x−b2)2 − a2 = 0 ou a est tel que a2 = b3. M ayant deux valeurs propre distinctesλ1 = b2 + a et λ2 = b2 − a, on en deduit que la matrice M est diagonalisable i.e. il

existe P ∈ GL2(Zp) telle que M =

b2 + a b0b0 b2− a

P−1.

b) L′applicationR → P−1RP qui est un automorphisme pour la structure d′anneau,transforme toute matrice A = λM+µI de Ap en une matrice diagonale. Or l′ensembleDp des matrices diagonales de M2(Zp) ayant comme cardinal p2 (je rappelle que CardAp = p2), l′application Ap → Dp, A → P−1AP est donc un isomorphisme.

D′autre part, l′application Zp × Zp → Dp, (α, β) →α b0b0 β

, etant de facon

immediate un isomorphism, on en deduit que: Ap ≃ Zp × Zp (cet isomorphisme estmeme un isomorphisme d′algebre).

c) Compte tenu de l′isomorphisme precedent, comme ∆(A) = ∆(P−1AP ), lenombre de matrices A ∈ Ap telles que ∆(A) = b1 est egal aux nombre de couple (α, β)

verifiant αβ = b1. Il y en a p− 1 (choisir d′abord α).

La demonstration de l′implication a) ⇒ b) du Theoreme 6 a montre que ∆(A) =b0 ⇒ A−1 ∈ Ap. On en deduit que le nombre de matrices A inversibles de Ap (i.e. le

109

Page 28: Revista (format .pdf, 5.8 MB)

nombre des matrices A ∈ Ap telles que ∆(A) = b0) est egal aux nombre des couples

(α, β) ∈ Zp × Zp verifiant αβ = b0 soit (p− 1)2.

Corollaire 8. Dans le cas ou b3 est un carre dans Zp:a) l′ordre de la matrice M divise p− 1;b) si p|Yk−2, alors 2k|p− 1.Demonstration. a) L′ensemble des matrices de Ap de determinant b1 forment un

sous-groupe multiplicatif de A∗p, groupe des matrices inversibles de Ap (si A ∈ Ap et

∆(A) = b0, alors A−1 ∈ Ap–voir a) ⇒ b) du theoreme 6). Compte tenu de l′assertionc) du theoreme precedent ce sous-groupe est fini de cardinal p − 1. Or la matriceM appartient a ce sous-groupe. L′ordre de M divise par suite p − 1 (ordre de cesous-groupe).

b) Si p divise Yk−2, alors la matrice M est d′ordre 2k d′apres le Theoreme 5.L′assertion a) de ce corollaire permet alors de conclure.

Theoreme 9. En supposant que b3 n′est pas un carre dans Zp:a) ∆ est un homomorphisme du groupe multiplicatif des elements non nuls de Ap

dans celui des elements non nuls de Zp;b) il existe k tel que p− 1 = Card (Im∆) et Card Ker∆ = k(p + 1);c) il existent p+ 1 elements de determinant b1 dans Ap.Demonstration. a) Ap etant un corps d′apres le Theoreme 6, l′assertion a) est

evidente.b) On en deduit l′isomorphisme Im∆ ≃ A∗

p/Ker∆ et par suite: Card A∗p = p2−1 =

(Card Im ∆)(Card Ker ∆). De plus, Im∆ est un sous-groupe de Z∗p: il existe k tel

que p− 1 = k Card (Im ∆), d′ou Card Ker ∆ = k(p+1). Notons que 1 ≤ k ≤ p− 1.c) Les matrices A de Ker ∆ verifient ∆(A) = b1. Il s′agit de montrer que Card

Ker ∆ = p + 1. L′egalite ∆(A) = b1 entraıne, compte tenu de l′assertion a) de laProposition 2: λ2 + µ2 + b4λµ− b1 = b0 (1). λ ∈ Zp etant donne, il existe donc 0, 1 ou

2 elements µ ∈ Zp tels que ∆(A) = b1 (Zp etant un corps), donc il existe au plus 2pcouples (λ, µ) verifiant l′equation (1). D′autre part, le nombre de tels couples est egala-voir b)-k(p + 1) = Card Ker ∆. On en deduit finalement que k = 1 et par suiteCard Ker ∆ = p+ 1.

Corollaire 10. Dans le cas ou b3 n′est pas un carre dans Zp:a) l′ordre de la matrice M divise p+ 1;b) si p|Yk−2, alors 2k|p+ 1.Demonstration. a) La matrice M de determinant b1 appartient au sous-groupe

Ker ∆. L′ordre de M divise par suite l′ordre p + 1 de ce sous-groupe Ker ∆ -voirl′assertion c) du Theoreme 9.

b) De meme qu′au Corollaire 8, si p divise Yk−2, alors la matrice M est d′ordre2k d′apres le Theoreme 5. L′assertion a) de ce corollaire permet alors de conclure.

Remarque. Dans le cas ou b3 n′est pas un carre dans Zp, l′ordre de toute matrice

A de Ap verifiant ∆(A) = b1 divise p + 1 - ordre de sous-groupe Ker ∆. Par suite∀A ∈ Ap : Ap+1 = I.

110

Page 29: Revista (format .pdf, 5.8 MB)

O metoda de rezolvare a problemelorMaria MIHET1

In aceasta nota vom evidentia o strategie importanta de rezolvare a problemelor:demonstratia prin exprimarea ın doua moduri a unor marimi. Aceasta metoda s-afolosit ınca ın primele clase de gimnaziu, pentru obtinerea unor ecuatii. Am ıntalnit-o,de asemenea, ın demonstratia formulei pentru suma unghiurilor unui poligon convex:cu ajutorul unui punct O din interiorul poligonului se trianguleaza poligonul si seexprima ın doua moduri suma unghiurilor triunghiurilor obtinute. Prin exprimareaın doua moduri a ariei sau volumului se pot afla anumite distante (ne amintim, deexemplu, cum se poate afla ınaltimea din varful unghiului drept ıntr-un triunghidreptunghic prin exprimarea ın doua moduri a ariei triunghiului), iar multe identitaticombinatoriale pot fi demonstrate folosind numararea ın doua moduri.

Am exemplificat metoda prin cateva probleme tip, iar la sfarsitul lucrarii amıntocmit o lista de probleme ınsotite de indicatii de rezolvare. Am dori ca cititorii saıncerce mai ıntıi sa le rezolve fara a apela la indicatii.

Aceasta nota se adreseaza ın special elevilor din clasele VI-VIII. Ea poate fi ınsacompletata cu multe exemple de nivel liceal.

Problema 1. Fie q un numar real, iar n ∈ N, n ≥ 2. Sa se calculeze, ın functiede q, suma Sn = 1 + q + q2 + . . .+ qn.

Solutie. Exprimam Sn+1 ın doua moduri: mai ıntai Sn+1 = Sn + qn+1, iar, pede alta parte, Sn+1 = qSn + 1. Rezulta ca Sn + qn+1 = qSn + 1, de unde obtinem ca

(∗) (q − 1)Sn = qn+1 − 1.

Astfel, daca q = 1, atunci Sn =qn+1 − 1

q − 1. Daca q = 1, prin ınlocuire ın Sn se

obtine direct ca Sn = n+ 1. Asadar,

Sn =

8<:n+ 1, daca q = 1;qn+1 − 1

q − 1, daca q = 1.

Problema 2. Fie n ≥ 2 un numar natural si Dn = d1, . . . , dk multimea divizo-

rilor naturali ai lui n. Demonstrati cad1 + . . .+ dk1d1

+ . . .+ 1dk

este un numar natural.

Solutie. Observam ca d este un divizor al lui n daca si numai daca nd este divizor

al lui n.Mai mult, multimea

§n

d1, . . . ,

n

dk

ªeste egala cu multimeaDn. Scriind ca suma

elementelor ın aceste multimi este aceeasi, obtinem egalitatead1 + . . .+ dk1d1

+ . . .+ 1dk

= n.

1Profesor, Scoala cu clasele I-VIII nr. 24, Timisoara

111

Page 30: Revista (format .pdf, 5.8 MB)

Problema 3. Sa se arate ca ın orice triunghi ABC au loc relatiile:

a)a

sinA=

b

sinB=

c

sinC(teorema sinusurilor);

b)BD

CD=

AB

ACunde D este piciorul bisectoarei unghiului A (teorema bisectoarei).

Solutie. a) Vom arata cab

sinB=

c

sinC; pentru aceasta, fie M mijlocul laturii

[BC]. Considerand triunghiurile ABM si ACM cu ınaltimile din A, obtinem caSABM = SACM .

Pe de alta parte, 2SABM = AB · BM · sin∠(ABC), iar 2SACM = AC · CM ·

sin∠(ACB). Cum CM = BM , rezulta ca b sinC = c sinB, adicab

sinB=

c

sinC.

b) Consideram triunghiurile ABD si ACD. AtunciSABD

SACD=

BD · ha

CD · ha=

BD

CD.

Pe de alta parte, deoarece D se afla pe bisectoarea unghiului A, ınaltimile din D

ale triunghiurilor ABD si ACD sunt egale; prin urmareSABD

SACD=

AB

AC. Egaland

rapoartele, obtinem teorema bisectoarei.

In cele ce urmeaza, propunem celor interesati o serie de probleme care pot firezolvate folosind aceasta strategie.

1) Vasul A contine apa, iar vasul B contine alcool pur. Se toarna ın A un paharplin de alcool din B, apoi se scoate din A un pahar de amestec, care se toarna ın B.Ce relatie exista ıntre cantitatea de alcool din A si cantitatea de apa din B?

Indicatie. Exprimati ın doua moduri cantitatea de apa care lipseste din A dupacele doua operatii.

2) Doi calatori au plecat ın acelasi moment din localitatile A si B, fiecare de-plasandu-se spre localitatea celuilalt cu viteza constanta. Ei s-au ıntalnit la ora 13 si,continuandu-si drumul, primul a ajuns ın B la ora 17, iar cel de-al doilea ın A la ora21. La ce ora au plecat cei doi ın calatorie?

Indicatie. Calculati ın doua moduri distantele parcurse de cei doi calatori pana laıntalnire.

3) O motonava a plecat ıntr-o cursa pe mare. Cand motonava s-a ındepartat cu180 mile de tarm, a fost trimis dupa ea un hidroavion cu un mesaj urgent. Vitezahidroavionului este de zece ori mai mare decat viteza motonavei. La ce distanta detarm a fost ajunsa motonava?

Indicatie. Exprimati ın doua moduri timpul pana la ıntalnire.4) 98 de numere naturale consecutive a1 < a2 < . . . < a98 au suma 137. Aflati

a2 + a4 + a6 + . . .+ a98.Indicatie. Exprimati ın doua moduri a1 + a2 + a3 + . . .+ a98.5) In fiecare din patratelele unui tabel dreptunghiular cu 4 linii si 5 coloane scriem

cate un numar natural, astfel ıncat suma numerelor de pe fiecare linie sa fie aceeasi sisuma numerelor de pe fiecare coloana sa fie aceeasi (nu neaparat egala cu cea de pelinii). Fie S suma numerelor din tabel. a) Putem avea S = 30? b) Dati doua exemplede tabele cu S = 20. c) Aflati toate tabelele cu S < 20.

Indicatie. Calculati S ın doua moduri.6) Exista poligoane convexe cu mai mult de trei unghiuri ascutite?

112

Page 31: Revista (format .pdf, 5.8 MB)

Indicatie. Calculati ın doua moduri suma unghiurilor poligonului.7) Un numar natural n ≥ 2 cu numar impar de divizori este patrat perfect.Indicatie. Scrieti ın doua moduri multimea divizorilor lui n si grupati divizorii ın

perechi de formand,

n

d

o.

8) Fie d1, . . . , dk multimea divizorilor naturali ai numarului natural n ≥ 2.Demonstrati ca (d1d2 . . . dk)

2 = nk.Indicatie. Scrieti ın doua moduri multimea divizorilor lui n.9) Demonstrati ca daca m ≥ 2, n ≥ 2 sunt numere naturale, iar m divide n, atunci

2m − 1 divide pe 2n − 1.Indicatie. Calculati ın doua moduri 1 + 2 + . . .+ 2n−1.

10) Aflati cea mai mica valoare pentru max

§a+

1

b, b+

1

a

ª, cand a, b ∈ (0,∞).

Indicatie. Evaluati ın doua moduri suma dintre a+1

bsi b+

1

a.

11) Pe latura (BC) a triunghiului ABC, se considera punctele D,E astfel ıncat

∠BAD ≡ ∠CAE. Demonstrati caBD

DC· BE

EC=

AB2

AC2(Steiner).

Indicatie. Exprimati ın doua moduriSABD

SACDsi

SABE

SACE.

12) In triunghiul ascutitunghic ABC, ınaltimea AA′ si mediana CM au aceeasilungime. Aflati m(∠MCB).

Indicatie. Exprimati ın doua moduri aria triunghiului MCB.13) Fie n ∈ N. Demonstrati ca

1− 1

2+1

3− 1

4+. . .+

1

2n− 1− 1

2n=

1

n+ 1+

1

n+ 2+. . .+

1

2n(Botez-Catalan).

Indicatie. Calculati ın doua moduri suma: 1+ 12+

13+. . .+ 1

n+1

n+1+1

n+2+. . .+ 12n .

14) Produsul a doua numere de forma x2− 2y2, x, y ∈ N, este un numar de aceesiforma.

Indicatie. Exprimati ın doua moduri (a+√2b)(c+

√2d)(a−

√2b)(c−

√2d).

15) Fie k ≥ 3 un numar natural impar. Aratati ca daca x1, x2, . . . , xk suntnumere ıntregi astfel ıncat |x1 − x2| = |x2 − x3| = . . . = |xk−1 − xk| = |xk − x1|, iarx1 + x2 + . . .+ xk = m, atunci k ıl divide pe m.

Indicatie. Exprimati ın doua moduri suma (x1 − x2) + (x2 − x3) + . . .+ (xk−1 −xk) + (xk − x1).

16) Aflati numerele N = a0a1 . . . a9 de 10 cifre (ın baza 10) cu proprietatea ca a0este numarul cifrelor de 0 ale lui N , a1 este numarul de 1-uri din scrierea lui N , . . . ,a9 este numarul cifrelor de 9 ale lui N.

Indicatie. Calculati ın doua moduri suma cifrelor lui N .17) Numerele a1, a2, . . . , an apartin multimii −1, 1, iar a1a2 + a2a3 + . . . +

an−1an + ana1 = 0. Aratati ca n este multiplu de 4.Indicatie. Calculati ın doua moduri produsul (a1a2)(a2a3) . . . (ana1).18) Un tablou cu m linii si n coloane are toate elementele egale cu 1 sau cu −1,

iar produsul elementelor de pe fiecare linie si fiecare coloana este −1. Demonstrati cam si n au aceeasi paritate.

Indicatie. Calculati ın doua moduri produsul elementelor din tablou.

113

Page 32: Revista (format .pdf, 5.8 MB)

19) Intr-un tablou cu 25 de linii si 25 de coloane pentru fiecare i, j ∈ 1, . . . , 25scriem la intersectia dintre linia i si coloana j numarul 1, daca i divide pe j si 0, dacai nu divide pe j. Demonstrati ca numarul zerourilor din tablou este par.

Indicatie. Calculati ın doua moduri suma elementelor tabloului.20) La un turneu de sah au participat 14 sahisti, fiecare jucand cate o partida cu

fiecare din ceilalti 13. La sfarsitul turneului s-a constatat ca suma punctelor obtinutede primii trei clasati a fost egala cu suma punctelor ultimilor 9. Stiind ca meciuldintre sahistii de pe locurile 4 si 5 nu s-a terminat remiza, aflati cate puncte a obtinutsahistul clasat pe locul 4 (pentru victorie se primeste 1p, pentru remiza 1

2p, iar pentruınfrangere niciun punct).

Indicatie. Calculati ın doua moduri suma punctelor primilor trei sahisti.21) Putem scrie pe tabla 17 numere reale astfel ıncat suma oricaror 7 dintre ele

sa fie strict pozitiva, iar suma oricaror 11 sa fie strict negativa?Indicatie. Daca x1, . . . , x17 sunt cele 17 numere, calculati ın doua moduri suma

elementelor din tabloul (cu 11 linii si 7 coloane):

x1 x2 . . . x7

x2 x3 . . . x8

. . . . . . . . . . . . . . . . . .x11 x12 . . . x17.

22) La o adunare a zeilor ın Olimp, fiecare zeu a primit ın cupa sa aceeasi cantitatede ambrozie si are voie sa toarne din cupa sa ın cupa altui zeu, ınsa doar o cantitateade ambrozie egala cu cea pe care o are celalalt zeu. La sfarsitul adunarii, toataambrozia a ajuns ın cupa lui Zeus. Demonstrati ca numarul participantilor a fost oputere a lui 2.

Indicatie. Scrieti ın doua moduri cantitatea initiala de ambrozie din cupa lui Zeus.

Bibliografie1. A. Engel - Probleme de matematica – strategii de rezolvare, GIL, Zalau, 2006.

2. M. Mihet, E. Obadeanu - Teste si probleme comentate pentru Concursul ”TraianLalescu” (gimnaziu), 19 (1988).

3. L. Niculescu - Teme de algebra pentru gimnaziu, Ed. Cardinal, 1993.4. G. Polya - Cum rezolvam o problema?, Ed. Stiintifica, 1965.5. *** - Colectia RMT, 1996-2009.

Vizitati noua pagina web a revistei:

http://www.recreatiimatematice.ro

114

Page 33: Revista (format .pdf, 5.8 MB)

Metoda identificarii

Silviu BOGA1

Ne ocupam ın cele ce urmeaza de demonstrarea unor identitati ce permit calcululunor sume sau al unor produse, prin formule de tipul

nXk=1

ak = bn, ∀n ∈ N∗, respectivnY

k=1

ak = bn, ∀n ∈ N∗.

In ambele situatii, cele mai populare strategii de abordare sunt urmatoarele:• demonstrarea relatiei prin rationamente sintetic-constructive; consideram ca a-

ceasta metoda este superioara calitativ fata de oricare alta, ınsa presupunem o anumedexteritate din partea rezolvitorilor ın utilizarea unor artificii de calcul si rationament;

• demonstrarea relatiei prin inductie matematica – este metoda aleasa de majori-tatea rezolvitorilor, alegere motivata de gradul de accesibilitate al rationamentului sicalculelor ın acest caz, comparativ cu aplicarea strategiilor de tip sintetic-constructiv.

Vom oferi cititorului spre comparare un procedeu mai putin cunoscut, pe carel-am denumit metoda identificarii, exprimat prin urmatoarea

Propozitie. a) Daca a1 = b1, bk−bk−1 = ak, ∀k ≥ 2, atuncinX

k=1

ak = bn, ∀n ≥ 1.

b) Daca a1 = b1 sibk

bk−1= ak, ∀k ≥ 2, atunci

nYk=1

ak = bn, ∀n ≥ 1.

Demonstratie. Intr-adevar,nX

k=1

ak = b1+nX

k=2

(bk − bk−1) = b1+(b2− b1)+ (b3−

b2) + ...+ (bn − bn−1) = bn, la sumare efectuand asa-numitele reduceri telescopice.

Faptul ca metoda identificarii nu apare printre caile frecvent aplicate pentru veri-ficarea unor identitati este explicabil prin aceea ca celelalte metode au o arie de apli-cabilitate incomparabil mai vasta. In ceea ce priveste ınsa eleganta rationamentuluisi simplitatea, urmatorul exemplu cu rezolvari comparative va convinge, probabil, deavantajele metodei identificarii.

Problema 1. Demonstrati canX

k=1

(k2 + 1)k! = n(n+ 1)!, ∀n ∈ N∗.

Solutia 1 (prin rationamentele sintetic-constructive). Daca ak = (k2 + 1)k! estetermenul general al sumei, cautam a, b, c astfel ıncat ak = a · k!+ b(k+1)!+ c(k+2)!;dupa calcule similare celor din metoda coeficientilor nedeterminati, gasim a = 2, b =−3, c = 1, prin urmare ak = 2 · k! − 3 · (k + 1)! + (k + 2)! Sumand, obtinem canX

k=1

ak = 2 · 1! + (2− 3) · 2! + (1− 3)(n+ 1)! + (n+ 2)! = n · (n+ 1)! si identitaea este

astfel demonstrata.

1Profesor, Colegiul Tehnic ”I.C. Stefanescu”, Iasi

115

Page 34: Revista (format .pdf, 5.8 MB)

Solutia 2 (prin inductie matematica). Identitatea se verifica pentru n = 1(obtinem 2 = 2, afirmatie adevarata). Presupunem ca egalitatea din enunt are locpentru n (oarecare) si demonstam ca este adevarata si pentru n+ 1. Avem

n+1Xk=1

(k2 + 1)k! =nX

k=1

(k2 + 1)k! + ((n+ 1)2 + 1)(n+ 1)! =

= n(n+ 1)! + (n2 + 2n+ 2)(n+ 1)! =

= (n+ 1)!(n2 + 3n+ 2) = (n+ 1)!(n+ 2)(n+ 1) = (n+ 1)(n+ 2)!

Conform principiului inductiei complete, identitatea are loc pentru orice n ∈ N∗.Solutia 3 (prin metoda identificarii). Identitatea este de tipul celei din Propozitia

1, ak = (k2 + 1)k! si bn = n(n + 1)!. Observam ca a1 = b1(= 2), iar bk − bk−1 =

k(k + 1)!− (k − 1)k! = (k2 + 1)k! = ak, ∀k ≥ 2. Rezulta canX

k=1

ak = bn = n(n+ 1)!

Prezentam ınca doua probleme, ın rezolvarea carora vom folosi direct metodaidentificarii:

Problema 2. Demonstrati canY

k=1

1− 2

k2 + 3k + 2

=

n+ 3

3(n+ 1),∀n ∈ N∗.

Solutie. Identitatea este de tipulnY

k=1

ak = bn, ∀n ≥ 1, cu ak = 1− 2

k2 + 3k + 2si

bn =n+ 3

3(n+ 1). Se observa ca a1 = b1, iar

bkbk−1

=k + 3

3(k + 1):k + 2

3k=

k2 + 3k

(k + 1)(k + 2)=

1− 2

k2 + 3k + 2= ak, ∀k ≥ 2. Conform Propozitiei 1, identitatea este demonstrata.

Problema 3. Demonstrati canX

k=1

sin kx =sin nx

2 · sin (n+ 1)x

2

sinx

2

, ∀n ∈ N∗.

Solutie. Cu notatiile din Propozitia 1, avem evident ca a1 = b1, iar bk − bk−1 =1

sin x2

sin

kx

2sin

(k + 1)x

2− sin

(k − 1)x

2sin

kx

2

=

1

sin x2

sin kx2 · 2 · sin x

2cos

kx

2=

2 sinkx

2cos

kx

2= sin kx = ak, ∀k ≥ 2, prin urmare are loc cerinta problemei.

Propunem cititorului stabilirea urmatoarelor identitati valabile pentru ∀n ∈ N∗:

4.nX

k=1

(2k − 1)2=n(4n2 − 1)

3, 5.

nXk=1

k2

(2k − 1)(2k + 1)=

n(n+ 1)

2(2n+ 1),

6.nY

k=1

1 +

p− 1

k(k + p)

=

p(n+ 1)

n+ p, 7.

nYk=1

cos 2kx =sin 2n+1x

2n · sin 2x.

Bibliografie

1. L. Panaitopol - Inductia matematica, Gil, Zalau, 2005.

2. Gh. Rizescu - Sume si produse, Ed. Sigma, Bucuresti, 1999.

116

Page 35: Revista (format .pdf, 5.8 MB)

Bisectoarele exterioarenu sunt ca bisectoarele interioare

Dumitru MIHALACHE si Marian TETIVA1

Abstract. It is known the fact that an analogous result to the Steiner-Lehmus Theorem for theexterior bisectors does not hold. The authors of this article present this result in a convenient wayfor the reader who wants to see how a case study should be performed.

Keywords: angle bisector, exterior angle bisector, Steiner-Lehmus Theorem.

MSC 2000: 97C20.

Inca din clasa a sasea elevii ınvata despre triunghiul isoscel. Mai precis, ei ınvataın primul rınd ca daca un triunghi are doua laturi congruente, atunci si unghiurileopuse acelor laturi sunt congruente. Mai afla ca, ıntr-un asemenea triunghi, medianelecorespunzatoare laturilor congruente sunt congruente si acelasi lucru se ıntampla cuınaltimile corespunzatoare acestor laturi, si tot congruente sunt si bisectoarele unghiu-rilor congruente. De prisos sa mai spunem ca privim toate aceste linii importante aletriunghiului ca pe niste segmente, toata lumea ıntelege la ce ne referim. De asemenea,este vorba de bisectoarele unghiurilor interioare ale triunghiului, deoarece despre celeale unghiurilor exterioare nu prea se mai vorbeste ın noile programe. Totusi n-ar figreu de demonstrat si congruenta a doua dintre ele daca le definim corespunzator sitriunghiul este isoscel, dar nu echilateral. Vom avea nevoie de aceasta notiune, asa caexplicam acum la ce ne referim. Sa spunem ca triunghiul ABC are laturile [AB] si[AC] de lungimi diferite si sa consideram bisectoarele unghiurilor sale exterioare careau varful ın A (acestea fiind unghiurile adiacente si suplementare cu unghiul interiorcare are varful ın A), deocamdata ca semidrepte (se stie ca sunt semidrepte opuse).Una din ele intersecteaza dreapta BC; daca AB > AC vedeti usor ca intersectia Meste astfel ıncat C se afla ıntre B si M . Vom numi atunci bisectoare exterioara din A atriunghiului ABC segmentul [AM ]. E clar ca, daca AB = AC, bisectoarea exterioaranu poate fi considerata ın sensul acestei definitii.

Nu mult dupa aceea, sau chiar deodata, elevii afla ca si reciprocele acestor afirmatiisunt adevarate. Dar, gasindu-se la ınceputul studiului geometriei, le va fi mult maigreu sa le justifice. Faptul ca un triunghi cu doua mediane congruente (sau cu douaınaltimi congruente) este isoscel nu prea poate fi demonstrat cu una, cu doua, deoarecenecesita cazuri speciale de congruenta. Iar despre a arata ca un triunghi cu doua bi-sectoare congruente este isoscel nici nu poate fi vorba atata vreme cat el face obiectulunei renumite teoreme care a pus la grea ıncercare chiar mari matematicieni. Dar esteadevarat, ceea ce nu se mai poate afirma ın cazul bisectoarelor exterioare (si despreasta vom vorbi mai departe). Profesorul C.L. Lehmus, ın 1840, a cerut pentru primadata o solutie sintetica a problemei, care a fost gasita, printre altii, de marele geometruJacob Steiner (se pare ca Lehmus ınsusi avea sa gaseasca ulterior o demonstratie);

1Profesori, Colegiul National ”Gheorghe Rosca Codreanu”, Barlad

117

Page 36: Revista (format .pdf, 5.8 MB)

teorema a ramas cunoscuta cu aceste nume: teorema Steiner-Lehmus. Mai multedespre istoria ei si demonstratia ın cadrul geometriei euclidiene clasice se gasesc, de ex-emplu, pe internet la adresa http://forumgeom.fau.edu/FG2005volume5/FG200525.pdf[2] (ca sa dam un exemplu la ındemana cititorilor mai tineri; altminteri, orice carteserioasa de geometrie euclidiana elementara cuprinde si aceasta teorema).

Noi vom ıncepe cu o demonstratie prin calcul a acestui renumit rezultat, probabilfoarte cunoscuta cititorilor. Vom considera un triunghi oarecare ABC ın care vomfolosi notatiile obisnuite: a, b, c sunt lungimile laturilor (opuse varfurilor A, B, re-spectiv C), la este lungimea bisectoarei obisnuite (interioare) a unghiului cu varful ınA. Formula

l2a = bc− a2bc

(b+ c)2

care furnizeaza lungimea acestei bisectoare este notorie, nu mai insistam asupra ei.Ca si asupra urmatorului calcul, pe care-l recomandam celor ce nu-l cunosc ınca.

Exercitiul 1. Sa se arate ca, cu notatiile de mai sus, avem

l2a − l2bc

= (b− a)

1 +

ab(a2 + b2 + c2 + ab+ 2ac+ 2bc)

(a+ c)2(b+ c)2

.

Evident, de aici se obtine imediat teorema Steiner-Lehmus si chiar mai mult, sevede ca avem la < lb ⇔ a > b (deoarece expresia din paranteza este strict pozitiva).Aceasta proprietate este valabila si pentru mediane sau ınaltimi, adica ma < mb ⇔a > b si ha < hb ⇔ a > b (daca notam ma si ha lungimea medianei, respectivınaltimii din A), dar vom vedea ca pentru bisectoarele exterioare nu este catusi deputin adevarata. Colateral, va puteti ocupa si de

Exercitiul 2. Demonstrati ca ma < mb ⇔ a > b si ha < hb ⇔ a > b.Acum sa vorbim si despre bisectoarele exterioare. Vom nota cu ea lungimea bi-

sectoarei exterioare din A (daca b = c). Formula

e2a =a2bc

(b− c)2− bc

este si ea destul de cunoscuta (daca n-o stiti, o puteti obtine fie din relatia lui Stewart,fie dintr-un calcul cu arii, folosind teorema bisectoarei exterioare pentru a exprima ınfunctie de a, b, c lungimile segmentelor determinate pe [BC] de bisectoare, respectivfolosind expresia lui cos(A/2)). Cu ajutorul ei obtinem imediat, la fel ca mai sus

e2a − e2bc

= (a− b)

1 +

ab(a2 + b2 + c2 + ab− 2ac− 2bc)

(a− c)2(b− c)2

.

Desigur, putem considera ea si eb (si calcula aceasta expresie) numai ıntr-un triunghineisoscel, deci mai departe presupunem ca oricare doua dintre a, b, c sunt distincte.

Exercitiul 3. Continuati acest calcul pentru a obtine

e2a − e2bc

=(b− a)(a+ b− c)(c3 − (a+ b)c2 + 3abc− ab(a+ b))

(a− c)2(b− c)2.

118

Page 37: Revista (format .pdf, 5.8 MB)

E clar acum ca nu avem cum sa obtinem egalitatea laturilor din egalitatea bisec-toarelor exterioare, desi acesta este unul din cazuri: daca presupunem ea = eb rezultaca unul din factorii de la numarator trebuie sa fie zero. Dintre acesti factori doar dea+ b− c putem fi siguri ca este nenul (pozitiv pentru a, b, c lungimile laturilor unuitriunghi). Ramane totusi al treilea factor, care ne permite sa demonstram urmatoarea

Propozitie. Pentru orice numere reale pozitive si distincte a si b exista c pozitivsi diferit de a si de b astfel ıncat a, b, c pot fi laturile unui triunghi, iar ın acesttriunghi ea = eb.

Demonstratie. Sa presupunem, de exemplu, ca a < b. Pentru functia continuaf definita prin

f(x) = x3 − (a+ b)x2 + 3abx− ab(a+ b), ∀x ∈ R,

avem f(b − a) = −2a3 < 0 si f(b) = ab(b − a) > 0, deci exista c ∈ (b − a, b) astfelıncat f(c) = 0. Acest c este diferit si de a, deoarece f(a) = ab(a− b) < 0; pentru cab > a, b > c si c > b− a implica b < a+ c, triunghiul cu laturile a, b si c exista si esteneisoscel. Exprimarea de mai sus a diferentei e2a − e2b ne arata ca, ın acest triunghi,ea = eb, ceea ce ıncheie demonstratia.

Se mai observa ca semnul acestei diferente nu poate fi stabilit numai ın functie desemnul lui b − a, de aceea un analog al proprietatii la < lb ⇒ a > b nu functioneazapentru bisectoarele exterioare. In plus, desi tocmai am aratat ca exista triunghiurineisoscele cu doua bisectoare exterioare congruente, cititorul va vedea usor ca nu sepoate ca toate bisectoarele exterioare sa aiba lungimi egale.

Exercitiul 4. Sa se arate ca ıntr-un triunghi neisoscel, nu se poate ca toate celetrei bisectoare exterioare sa fie congruente.

In sfarsit, sa ne mai punem o ıntrebare fireasca: putem da un exemplu concretde asemenea triunghi (neisoscel, dar care are doua bisectoare exterioare congruente)?Caci una e sa spui: exista un astfel de triunghi, si alta e daca afirmi raspicat catriunghiul cu laturile de anumite lungimi (bine precizate) are respectiva proprietate.Avem un asemenea exemplu!

Exercitiul 5. Sa se arate ca ın triunghiul cu laturile a = 5 −√7, b = 5 +

√7 si

c = 6 avem ea = eb.Mai mult, putem gasi toate triunghiurile cu aceasta proprietate.

Propozitie. Un triunghi neisoscel ABC are proprietatea ea = eb daca si numaidaca exista un numar p din intervalul (0, 1) si un numar pozitiv c astfel ıncat lungimilelaturilor sale sa fie date de

BC =c

2(p+ 1)

3p+ 1−

È(1− p)(p2 + 3p+ 4)

,

AC =c

2(p+ 1)

3p+ 1 +

È(1− p)(p2 + 3p+ 4)

si AB = c.

Demonstratie. Desigur, ca sa avem ea = eb ıntr-un triunghi neisoscel trebuie caultimul factor de la numaratorul expresiei e2a− e2b sa fie zero, adica trebuie sa aiba locegalitatea

c3 − (a+ b)c2 + 3abc− ab(a+ b) = 0.

119

Page 38: Revista (format .pdf, 5.8 MB)

Cu notatiile x = a/c si y = b/c aceasta se mai scrie (dupa ımpartirea cu c3):

1− (x+ y) + 3xy − xy(x+ y) = 0 ⇔ 1− s+ 3p− sp = 0,

daca punem s = x + y si p = xy. Desigur, x si y sunt solutiile ecuatiei de gradul aldoilea t2 − st + p = 0, iar daca stim pe x si pe y, cunoastem si laturile triunghiului:BC = cx, AC = cy si BC = c. Relatia de mai sus ne furnizeaza s = (3p+ 1)/(p+ 1)si un calcul rapid ne arata ca discriminantul ecuatiei de gradul al doilea este atunci

∆ = s2 − 4p =

3p+ 1

p+ 1

2

− 4p =(1− p)(4p2 + 3p+ 1)

(p+ 1)2

care va fi pozitiv daca si numai daca p ∈ (0, 1) (de la bun ınceput e clar ca p trebuiesa fie pozitiv, iar din ∆ > 0 rezulta ca p < 1). Daca ∆ > 0, ecuatia va aveadoua solutii pozitive si distincte care conduc exact la formulele din enunt pentrulungimile laturilor. Interesant este ca aceste valori ale lui x si y produc ıntotdeauna(fara sa fie nevoie de vreo conditie suplimentara) lungimile laturilor unui triunghi(ın care, credem ca e foarte clar, ea = eb). Intr-adevar, ordinea lor se vede ca estea < c < b, ceea ce corespunde inegalitatilor x < 1 < y (care rezulta repede din(x − 1)(y − 1) = p(p − 1)/(p + 1) < 0), caz ın care mai trebuie sa verificam doarb < a+ c ⇔ b− a < c ⇔ y − x < 1. Dar

y − x =√∆ = 1− 4p3

(p+ 1)2< 1

si demonstratia este ıncheiata.

Noi am considerat p = 1/2 si c = 6 pentru a obtine exemplul din Exercitiul 5, darcititorul are acum la dispozitie formulele generale care dau lungimile laturilor acestortriunghiuri si ısi poate construi oricate exemple doreste. Intrebare (inevitabila): gasimtriunghiuri cu toate lungimile laturilor numere ıntregi ın care ea = eb? (Nu cunoastemraspunsul la acesta ıntrebare.)

Probabil ati observat ca titlul, asa glumet-aluziv cum este el spune exact ceam vrut noi sa evidentiem ın aceasta nota: proprietatile bisectoarelor interioare(la = lb ⇒ a = b, respectiv la < lb ⇒ a > b care au loc si pentru celelalte liniiimportante obisnuite ale triunghiului) nu se pastreaza deloc pentru bisectoarele ex-terioare. Lasam ın seama cititorului interesat (si mai bun prieten cu geometria eu-clidiana clasica) sa explice pur geometric aceste fapte aparent stranii. Mentionam caexistenta triunghiurilor neisoscele cu doua bisectoare exterioare congruente e cunos-cuta de mult: de exemplu, se vorbeste despre ele ın [1], la pagina 235 (unde suntnumite triunghiuri pseudoisoscele).

Bibliografie

1. F. G.-M. – Exercices de geometrie, 1912.

2. K.R.S. Sastry – A Gergonne Analogue of the Steiner - Lehmus Theorem, ForumGeometricorum, 5 (2005), 191-195.

120

Page 39: Revista (format .pdf, 5.8 MB)

Metoda falsei ipoteze - variante de utilizare

D.M. BATINETU-GIURGIU 1

In clasele primare elevii ıntalnesc marimi care sunt dependente una de alta. Deexemplu, daca ın produsul F1 · F2 creste (descreste) unul din factori de un numar deori, acelasi lucru se ıntampla si cu produsul, iar daca unul din factori creste (descreste)de un numar de ori si produsul ramane neschimbat, atunci celalalt factor descreste(creste) de acelasi numar de ori.

Problemele ın care apar astfel de marimi se rezolva uneori cu metoda falsei ipoteze,care comporta parcurgerea urmatoarelor etape: 1) se face o ipoteza arbitrara asupramarimilor de aflat, ce va fi, ın general, ın contradictie cu datele problemei; 2) seanalizeaza sursele esecului si erorii comise si se trag concluziile corespunzatoare; 3) semodifica ipoteza (adica valorile date arbitrar marimilor) pe baza concluziilor punctuluiprecedent, ın scopul obtinerii solutiei; 4) se verifica corectitudinea solutiei gasite.

Deoarece ın programa claselor primare nu mai este inclusa aceasta metoda, damun exempul de problema pe care vom ilustra aceasta cale de rezolvare.

Problema. Un producator a vandut la piata 1200 kg mere de 2 lei/kg, 3 lei/kg si4 lei/kg, pe care a ıncasat 3300 lei. Stiind ca producatorul a avut cantitati egale demere de 2 lei/kg si 3 lei/kg, sa se afle cate kilograme de mere de 2 lei/kg, 3 lei/kg si4 lei/kg a vandut producatorul.

Prezentam sase variante de rezolvare, utilizand de fiecare data metoda falseiipoteze, cu o ipoteza sau cu doua ipoteze arbitrare.

Varianta 1. Presupunem ca toate merele sunt de 4 lei/kg. In aceasta ipotezase ıncaseaza 4 × 1200 = 4800 lei. Suma reala este depasita cu 4800 − 3300 = 1500lei. Aceasta depasire apare din faptul ca am ınlocuit x kilograme mere de 2 lei/kg six kilograme mere de 3 lei/kg cu 2x kilograme mere de 4 lei/kg. Inlocuind 1 kg demere de 2 lei/kg si 1 kg de mere de 3 lei/kg cu 2 kg mere de 4 lei/kg, suma realase mareste cu 8 − (2 + 3) = 3 lei. Avem un numar de 1500 : 3 = 500 ınlocuiri, ceeace ınseamna ca avem 500 kg mere de 2 lei/kg, 500 kg de mere de 3 lei/kg si 200 kgmere de 4 lei/kg. Se verifica usor ca aceste cantitati ındeplinesc conditiile problemei:500× 2 + 500× 2 + 200× 4 = 3300 lei.

Varianta 2. Presupunem ca avem numai mere de 2 lei/kg si de 3 lei/kg. In aceastaipoteza suma ıncasata de producator este 600 × 2 + 600 × 3 = 3000 lei. Diferentadintre suma reala si cea din presupunere este 3300−3000 = 300 lei. Aceasta diferenta

1Profesor, Colegiul National ”Matei Basarab”, Bucuresti

121

Page 40: Revista (format .pdf, 5.8 MB)

provine din faptul ca, de x ori am ınlocuit cate 2 kg de mere de 4 lei/kg cu 1 kg demere de 2 lei/kg si 1 kg de mere de 3 lei/kg, adica de cate ori se cuprinde (8−2−3) ın300, de unde aflam ca x = 100. Inseamna ca producatorul a vandut 2× 100 = 200 kgmere de 4 lei/kg, 500 kg de mere de 2 lei/kg si 500 kg de mere de 3 lei/kg. Verificarease face la fel.

Varianta 3. Presupunem ca cele trei categorii de mere sunt ın cantitati egale. Inaceasta ipoteza suma ıncasata este 400× 2 + 400× 3 + 400× 4 = 3600 lei. Diferentadintre suma presupusa si cea reala este 3600− 3300 = 300 lei. Evident, cantitatile nupot fi egale. Trebuie sa ınlocuim de x ori cate 2 kg mere de 4 lei/kg cu 1 kg de merede 2 lei/kg si 1 kg de mere de 3 lei/kg. Obtinem x = 300 : (8− 2− 3) = 100, ceea ceınseamna ca trebuie sa micsoram cantitatea de mere de 4 lei/kg cu 2× 100 = 200 kg.Am aflat ca producatorul a vandut 200 kg mere de 4 lei/kg, 500 kg mere de 2 lei/kgsi 500 kg mere de 3 lei/kg.

Varianta 4. Observam ıntai ca producatorul a vandut cantitati de mere expri-mate prin numere naturale pare. Sa presupunem ca producatorul a vandut 500 kgmere de 4 lei/kg. In acest caz suma ıncasata este 350× 2 + 350× 3 + 500× 4 = 3750lei. Diferenta dintre suma din presupunere si cea reala este 3750 − 3300 = 450 lei.Evident, cantitatea de mere de 4 lei/kg trebuie micsorata. Inlocuim de x ori cate 2 kgde mere de 4 lei/kg cu 1 kg de mere de 2 lei/kg si 1 kg mere de 3 lei/kg, diminuandsuma presupusa cu 8−(2+3) = 3 lei. Cantitatea de mere de 4 lei/kg trebuie micsoratacu 450 : 3 × 2 = 300 kg, ceea ce ınseamna ca producatorul a vandut 200 kg mere de4 lei/kg, 500 kg mere de 2 lei/kg si 500 kg mere de 3 lei/kg.

Varianta 5. Presupunem ca producatorul a vandut 300 kg mere de 2 lei/kg,300 kg mere de 3 lei/kg si 600 kg mere de 4 lei/kg. In aceasta ipoteza producatorulıncaseaza 300× 2 + 300× 3 + 600× 4 = 3900 lei. Diferenta dintre suma presupusa sicea reala este de 3900− 3300 = 600. Trebuie sa ınlocuim de x ori cate 2 kg de merede 4 lei/kg cu 1 kg mere de 2 lei/kg si 1 kg mere de 3 lei/kg, iar suma ıncasata ınaceasta ipoteza se micsoreaza cu 8 − (2 + 3) = 3 lei. Cantitatea de mere de 4 lei/kgtrebuie micsorata cu 600 : 3×2 = 400 kg, ceea ce ınseamna ca producatorul a vandut200 kg mere de 4 lei/kg, 500 kg mere de 2 lei/kg si 500 kg mere de 3 lei/kg.

Varianta 6. Prima ipoteza: producatorul a vandut 320 kg mere de 2 lei/kg, 320kg mere de 3 lei/kg si 560 kg mere de 4 lei/kg. Suma ıncasata ın prima ipoteza este320× 2+320× 3+560× 4 = 3840 lei. Ipoteza a doua: producatorul a vandut 340 kgmere de 2 lei/kg, 340 kg mere de 3 lei/kg si 520 kg mere de 4 lei/kg. In aceasta ipotezasuma ıncasata este 340× 2+ 340× 3+ 520× 4 = 3780 lei. Se observa ca la o cresterede 40 kg mere de 4 lei/kg, prima suma presupusa se micsoreaza cu 3840− 3780 = 60lei. Prima suma presupusa trebuie sa scada cu 3840− 3300 = 540 lei. Cantitatea de560 kg mere de 4 lei/kg trebuie sa scada cu 540 : 60× 40 = 360 kg, ceea ce ınseamnaca producatorul a vandut 200 kg mere de 4 lei/kg, 500 kg mere de 2 lei/kg si 500 kgmere de 3 lei/kg. Verificarea este imediata.

122

Page 41: Revista (format .pdf, 5.8 MB)

Liceul Teoretic ”Garabet Ibraileanu”1

La 110 ani de la ınfiintare

La prima vedere Liceul Teoretic ”Garabet Ibraileanu”, cu arhitectura sa usorconfundabila cu aceea a scolilor ridicate ın ”epoca de aur”, pare o institutie scolarade data recenta. Nimic mai fals. Liceul acesta are mai bine de 100 de ani de laınfiintare si istoria sa reflecta convulsiile prin care a trecut societatea romaneasca,drama intelectualitatii si chiar ”drama psihologiei” pentru a prelua titlul unei cartide Vasile Pavelcu, fost profesor la aceasta scoala.

Legea ınvatamantului secundar promulgata de Spiru Haret cerea candidatilor laprofesorat sa fi trecut un examen de capacitate. Dar pentru a se putea prezenta laacest examen candidatii trebuiau sa frecventeze un Seminar Pedagogic, sa parcurgamodulul pedagogic, am spune noi astazi, si implicit sa faca un numar de ore depractica pedagogica. De aceea s-a creat Seminarul Pedagogic Universitar cupropria scoala de aplicatie. Aceasta se ıntampla la 1 noiembrie 1899, localul scoliifiind casa Aslan de la vale de Universitatea noua din Copou. Scoala de aplicatie aveaun dublu rol: pentru studenti sau alsolventi era locul unde ısi desfasurau practicapedagogica, pentru profesorii universitari era locul ın care ısi puneau ın practicametodele moderne de predare, un laborator de pedagogie experimentala. Se observadin capul locului coerenta care exista ıntre activitatea didactica, pregatirea pedagogicaa viitorilor profesori si activitatea de cercetare a universitarilor. Nu este de mirare cade aici au plecat majoritatea cadrelor didactice din liceele Moldovei. Multi absolventi

1N. R. Pentru informatii mai ample recomandam cartea Magda Negrea – Liceul de Aplicatieal Semninarului Pedagogic Universitar – Iasi cu un studiu introductiv de prof.dr. AdrianNeculau.

123

Page 42: Revista (format .pdf, 5.8 MB)

ai scolii de aplicatie au revenit la SPU ca practicanti, iar unii chiar ca profesori. Deasemenea unii profesori ai SPU au pus bazele unor scoli asemanatoare la Cernauti(Constantin Narly)) si Cluj sau a Universitatii din Chisinau (Constantin Fedeles).

Liceul de Aplicatie al SPU a functionat cu succes pana la reforma ınva-tamantului din 1948. O istorie de jumatate de veac marcata de personalitatea celordoi directori, ambii pedagogi de frunte ai scolii iesene: Ioan Gavanescul si StefanBarsanescu. Ioan Gavanescul, pedagog cu studii ın Germania, a pus bazele unuiregulament organic al institutiei. ınainte de deschiderea scolii a convocat profesorii ınsase conferinte ın care a fixat normele si principiile generale didactice si de ordine sidisciplina. Ideea calauzitoare a fost ”educatia ca scop, instructia ca mijloc”. Scoalatrebuie sa dezvolte simtul practic si spontaneitatea mentala. De aceea ın predare se vafolosi metoda activa, intuitia si corelatia ıntre obiectele de studiu. Educatia nu trebuiesa fie teoretica si intelectuala ci ”practica si voluntara”, elevul, asemeni lui Robin-son Crusoe, sa stie ın orice ımprejurare ”a gasi mijloacele practice pentru a ınvingeorice greutate”. Pe langa activitatea teoretica, elevul era antrenat ın multe activitatipractice: cartonaj, modelaj, tamplarie. La SPU din Iasi s-a introdus pentru primadata lucrul manual la o scoala de cultura generala. Ioan Gavanescul a fost directorpana ın 1932, cand a iesit la pensie, dar a continuat sa tina conferinte, era membrual partidului conservator, a avut convingeri de dreapta. La venirea comunistilor laputere, desi se apropia de 90 de ani a fost arestat si moare ın ınchisoare ın 1951.

Intre 1932 si 1938 director este Paul Nicorescu, istoric, fiind ajutat de VasilePavelcu, responsabil cu practica pedagogica si de E. Briul care raspundea de ac-tivitatea dirigintilor si disciplina scolara. Sub directoratul lui Paul Nicorescu se con-struieste aripa de vest a scolii, care exista si azi la adresa din Toma Cozma, nr. 2.Stefan Barsanescu, care ıi urmeaza ca director, continua sa construiasca ın ciudavremurilor de razboi. Sub coordonarea lui Stefan Barsanescu se construieste laturadin centru si se toarna fundatia pentru aripa est care ar fi ınchis acest amsamblu.ın aceasta activitate, Stefan Barsanescu este secundat de directorul administrativI. Rick si de directorul adjunct D. Gafitanu.

Odata cu venirea lui Stefan Barsanescu la conducerea Seminarului Pedagogic Uni-versitar, ıncepe o perioada de dezvoltare distincta a acestei institutii. Cu studii laUniversitatea din Iasi si doua doctorate ın Germania, Stefan Barsanescu are toateatributele unui mare pedagog. Spre deosebire de Ioan Gavanescul, care punea accentpe formarea caracterului si pregatirea practica, Stefan Barsanescu pune accent pelectie, pe transmiterea de valori. Totul graviteaza ın jurul ideii de cultura. PentruStefan Barsanescu un ”om cultivat” este acela care se conduce ın activitatea sa dupa”valorile vesnice” ımbogatindu-si si ordonandu-si nazuintele dupa adevar (stiinta),bine(morala) si frumos(arta). In privinta studentilor practicanti, Stefan Barsanescuıi roaga pe profesori ”sa-i castige ca pe niste prieteni, sa nu fie descurajati, sa se simtaca ıntr-o mare familie. Practicantii asistau la un numar impresionant de ore panaerau lasati sa faca lectii de proba. Toate lectiile erau discutate. Fiecare practicant ısiıntocmea un dosar consistent cu observatii, rapoarte, planuri de lectii, un adevaratghid cu care pornea ın activitatea la catedra.

Dar ın martie 1944 tot Iasul pleaca ın refugiu. Tot ce s-a putut demonta si ambalaa fost pus ın 14 vagoane si transportat ın Banat. Iasul est bombardat, trei bombe

124

Page 43: Revista (format .pdf, 5.8 MB)

cazand si pe Liceul de aplicatie. In martie 1945, Liceul revine din refugiu si pe 9 apriliese ıncep cursurile. Zece vagoane din cele 14 sunt recuperate, se fac reparatii, ıncatla 1 oct. 1945 cladirea adaposteste atat Liceul de aplicatie cat si clase de la LiceulInternat si de la ”Oltea Doamna”. Se fac demersuri pentru a continua constructiaaripei de est la care de multi ani era turnata fundatia. Totul parea ınca posibil,dar reforma ınvatamantului din 1948 face totul imposibil. Seminarul Pedagogic sedesfiinteaza, Liceul de aplicatie ısi schimba aceasta menire si devine Liceul Clasic.Se poate spune ca ıncepe drama psihologiei si pedagogiei. Disciplinile acestea aproapeca dispar din ınvatamantul universitar, practica pedagogica se reduce ca numar deore si ca importanta, dar, ın mod paradoxal, se face cu precadere tot la fostul Liceu deaplicatie. Stefan Barsanescu este ındepartat atat de la liceu cat si de la Universitate.Abia daca ocupa un post de bibliotecar. Este reprimit la catedra ın 1957. Aveanu numai cultura, ci si har ısi aminteste studentul de atunci Adrian Neculau, acumprofesor de psihologia grupului social si fervent admirator al S.P.U. de altadata.

Liceul de aplicatie fusese doar de baieti, Liceul Clasic este mixt si are 12 clase. Din1953 se numeste Scoala Medie Clasica, apoi din anul urmator se numeste ScoalaMedie nr. 2 de fete. Toate aceste schimbari, adevarate convulsii pentru ınvatamantın general si pentru liceul de care ne ocupam ın special, precum si politizarea ın excesa lectiilor au drept scop ruperea de istoria scolii, uitarea bunei traditii. Din anul 1956,scolile medii devin mixte si se numara din nou, liceul nostru semicentenar devenindScoala Medie nr. 4. Pentru ca uitarea sa se astearna si mai bine pe istoria scolii, ınloc sa se termine de construit aripa est, se ıncepe construirea unui nou local pe stradaOastei nr. 1. Scoala Medie nr. 4, care ıntre timp a capatat si numele de Liceul”Garabet Ibraileanu”, se muta ın noul local ın 1962. In localul vechi continua safunctioneze internatul pentru mai multi ani, apoi aici se instaleaza dispensarul pentruelevi si studenti. Vis-a-vis, ın vechea scoala de ınvatatoare, functioneaza Facultateade psihologie si pedagogie, adica fostul SPU. E doar ironia sortii sau contratimpultragic care ne marcheaza pe noi, romanii? Vechea cladire din Toma Cozma nr. 2asteapta si acum sa fie terminata si sa serveasca drept scoala ıntre marile scoli aleIasului: Universitate, Facultatea de psihologie, Liceul Internat si nu departe LiceulNational si fosta ”Oltea Doamna”.

In noul local din dealul Copoului, liceul are profilul real - umanist si din 1967devine scoala pilot pentru ınvatarea limbilor straine, ın speta limba franceza, dinciclul primar. In 1977 ısi schimba iar numele si profilul, devenind Liceul IndustrialHidrotehnic. Dar pe fiecare an de studiu se pastreaza o clasa de reala si una defilologie. In 1990 revine la profilul real - umanist. Pe baza traditiei de studii clasice,la fiecare an de studiu a existat pana ın 2004 o clasa de limbi clasice (limba latina sigreaca veche). Actualmente liceul este teoretic si are clase de informatica, stiinte alenaturii, filologie.

Povestea Liceului ”Garabet Ibraileanu” e povestea noastra a tuturor, mereu”sub vremuri”. Si totusi spiritul Liceului de aplicatie, samanta buna semanata deacei dascali de demult, cu dragoste de tara si de profesie nu a pierit si aducerea ınmemorie a unui secol de scoala ne ıncurajeaza si ne obliga.

Prof. Magda NEGREA

125

Page 44: Revista (format .pdf, 5.8 MB)

LIDIA COHAL

(1930-2009)

Evenimente a caror ımpletire este mai presus de dorinte si ratiuni ne obliga laoarece zabava. Pe langa numeroasele, nobilele si acaparantele obligatii ale profesorilorde matematica, ni se releva una mai rar evidentiata, dar pe care o gasim plina desemnificatii: activitatea ın familia substantial extinsa a matematicienilor.

Lidia s-a nascut la 30 octombrie 1930 ın satul Ciutulesti, judetul Balti, de parteamai zbuciumata a Prutului. Absolva scoala primara ın satul natal si continua primelepatru clase de liceu la Liceul ”Regina Maria” din orasul Balti. Nu consideram nece-sar sa explicam fracturile sociale care au determinat ca ultimele patru clase sa si ledesfasoare la Liceul ”Ecaterina Teodoroiu” din Targu Jiu.

Lidia devine studenta la Facultatea de matematica si fizica la Universitatea ”Al.I. Cuza” Iasi. Ca recunoastere a meritelor de la ınceputul anului IV este numitapreparator. La absolvirea facultatii, ın anul 1952, a fost ıncadrata ca asistent univer-sitar la Catedra de geometrie, bine reputata inclusiv la nivel international. Impreunacu colegele si prietenele Alice Corduneanu si Ligia Papuc (deja casatorite cu distinsimatematicieni) se lanseaza ın activitati stiintifice si didactice, devenind si o harnicasecretara a Filialei Iasi a SSM.

In septembrie 1959 i se desface contractul de munca: originea sociala, fiica depreot, era privita drept ”necorespunzatoare”. Conducerea SSM se dovedeste maiputin influentabila si decide retribuirea fructuoasei sale activitati de secretara defiliala. Este locul sa spunem ca Societatea preluase aproape integral sarcinile deperfectionare a profesorilor de matematica prin scoli de vara; filiala ieseana activaın aceasta directie extrem de sustinut si materialele respective au fost incorporate ınpatru volume de ”Probleme actuale de matematica”.

In septembrie 1960 este ıncadrata si ın ınvatamantul liceal din Iasi, pastrandu-sisi atributiile de ”secretara de filiala model”: extrem de precisa gasea mereu zambetulprietenesc stimulator.

A functionat la liceele iesene ”National”, ”Emil Racovita” si ”Mihai Eminescu”.Din perioada ın care era ıncadrata ca asistent universitar, Lidia ıncepuse docu-

mentarea si lucrul la o carte destinata ”modernizarii predarii geometriei”. Coordonaeforturile profesorul Adolf Haimovici, presedinte al filialei Iasi si mai colaborau pri-etenele mai sus numite Alice si Ligia. Cartea Elemente de geometrie a planului aaparut abia ın 1968 la Editura Didactica si Pedagogica.

Lidia s-a pensionat ın septembrie 1987.De multe ori se omite amintirea unor realizari ın pretentioasa activitate de sotie;

nu vom face aici aceasta greseala. Prin optimismul ei molipsitor si prin tonusul eiexcelent, Lidia a potentat neıntrerupt activitatea sotului ei, distinsul profesor ieseanTraian Cohal.

Unii oameni nu mor de tot.Prof.dr. Dan BRANZEI

Presedintele Filialei Iasi a SSM

126

Page 45: Revista (format .pdf, 5.8 MB)

Concursul de matematica “Al. Myller”Editia a VII-a, Iasi, 28 martie 2009

Clasa a VII-a

1. Determinati n ∈ N∗ cu proprietatea ca n!+3 ·2n = 6n−2, unde n! = 1 ·2 ·3 . . . n.Artur Balauca

2. Fie triunghiul ABC si punctul D situat pe latura [BC]. Aratati caAB ·DC +AC ·BD ≥ AD ·BC.

***3. Fie p un numar natural impar. Se stie ca oricare divizor al lui p are ultima

cifra diferita de 3 si 7. Sa se arate ca numarul 5p+ 1 nu este patrat perfect.Mircea Fianu

4. Fie triunghiul echilateral ABC si punctul D situat pe latura (AC). Bisectoareaunghiului ∠ABD intersecteaza paralela prin A la dreapta BC ın punctul E. Aratatica AE +DC = BD.

Cristian Lazar

Clasa a VIII-a

1. Fie un tetraedru regulat cu muchia de lungime 3. Pe suprafata acestuia seconsidera 37 de puncte. Aratati ca printre aceste puncte exista doua astfel ıncatdistanta dintre ele este cel mult egala cu 1.

***

2. Determinati perechile de numere (a, b) ∈ Z× Z care verifica egalitatea2(a+ b)2 + 3(a+ b) + ab+ 4 = 0.

Petru Raducanu

3. Fie a ≥ b ≥ c ≥ d ≥ 0 astfel ıncat a2 + b2 + c2 + d2 = 1. Sa se arate caa+ b ≥ 1 ≥ c+ d.

Gheorghe Iurea

4. Numim piramida Myller o piramida SABCD cu baza ABCD, care are SA =SB = SC = SD, ∠ASB ≡ ∠ASD si ∠BSC ≡ ∠DSC, iar lungimile SA,AB,BC,CD, DA, AC, BD sunt numere naturale nenule. Aflati piramida Myller de volumminim.

Cristian Lazar

Clasa a IX-a

1. Determinati n ∈ N∗ pentru care exista o multime A ⊂ R cu n elemente, avandproprietatea a(b3 + 6) ≤ b(a3 + 6), ∀a, b ∈ A.

Gheorghe Iurea

127

Page 46: Revista (format .pdf, 5.8 MB)

2. Care este numarul minim de elemente care trebuie eliminate din multimea1, 2, 3, . . . , 100 astfel ıncat ın multimea ramasa sa nu existe trei elemente x, y, zpentru care xy = z?

***

3. Fie triunghiul ABC si punctele M ∈ (AB), N ∈ (AC) astfel ıncat BM+CN =MN +BC. Notam ρ raza cercului ınscris ın triunghiul AMN . Aratati ca

ρ(√bc+

È(p− b)(p− c)) ≤ r(

√bc−

È(p− b)(p− c)).

Dan Branzei

4. Numarul ıntreg m are proprietatea ca, pentru orice numar natural n, existaan ∈ Z astfel ıncat |nm− 80an + 1| < 20. Aratati ca 80 divide m.

Dinu Serbanescu

Clasa a X-a

1. Determinati valorile lui n ∈ N∗ pentru care 41 are un multiplu de formaa00 . . . 0b| z n+2 cifre

unde a, b sunt cifre zecimale nenule.

Mihai Baluna

2. Fie ABC un triunghi, k ∈ R\1 si punctele M ∈ BC, N ∈ CA, P ∈ AB

astfel ıncatMB

MC=

NC

NA=

PA

PB= k. Se stie ca AM = BN = CP. Demonstrati ca

triunghiul ABC este echilateral.I. V. Maftei

3. Pentru x, y ∈ R definim f(x; y) = distanta de la |x − y| la cel mai apropiatıntreg, iar pentru o multime finita M ⊂ [0, 1] definim

s(M) =X

x,y∈M,x<y

f(x, y).

Determinati valoarea maxima pentru s(M), cand M parcurge familia multimilor cu4 elemente.

Dinu Serbanescu4. Fie a un numar real. Aratati ca sirul de termen general xn = (−1)[na], n ≥ 0,

este periodic daca si numai daca a este rational.Gheorghe Iurea

Clasa a XI-a

1. Fie sirul (xn)n≥1 de numere reale definit prin x1 = 1 si xn+1 =xn − n

n+ 1

,oricare ar fi n ≥ 1. Sa se arate ca sirul este divergent.

Paul Georgescu, Gabriel Popa

2. a) Sa se determine doua matrice A,B ∈ M2(C) astfel ıncat A2 + B2 = I2 simatricea AB −BA este inversabila.

128

Page 47: Revista (format .pdf, 5.8 MB)

b) Fie n un numar natural impar si matricele A,B ∈ Mn(C) astfel ıncat A2+B2 =In. Sa se arate ca det(AB −BA) = 0.

Andrei Ciupan

3. Fie P,Q : R → R doua functii polinomiale neconstante cu proprietatea caexista si este finita lim

x→∞([P (x)] − [Q(x)]). Sa se arate ca exista n ∈ Z astfel ıncat

P (x)−Q(x) = n, oricare ar fi x ∈ R.Gheorghe Iurea

4. Fie A ∈ Mn(R) o matrice simetrica cu elementele de pe diagonala principalaegale cu 1 si cu suma modulelor elementelor de pe fiecare linie mai mica sau egala cu2. Sa se arate ca detA ≤ 1.

Cosmin Pohoata

Clasa a XII-a

1. Fie f, g doua functii polinomiale reale, de acelasi grad, ambele avand coeficien-tul dominant egal cu 1. Daca g nu are radacini reale pozitive, calculati

limn→∞

n

Z 1

0x

f(nx)

g(nx)− 1

dx.

Radu Gologan

2. Fie A un inel. a) Aratati ca daca x ∈ A este nilpotent (exista k ∈ N∗ cuxk = 0), atunci 1 + x este inversabil.

b) Daca A este finit, numarul elementelor inversabile este un numar prim iarelementele neinversabile sunt nilpotente si numarul elementelor neinversabile este maimare sau egal cu numarul elementelor inversabile, aratati ca A are 4 elemente.

***

3. Sa se determine functiile continue f : R → R care verifica egalitatea f(arctg x) =(1 + x2)f(x), pentru orice x ∈ R.

Al. Gabriel Mırsanu

4. Fie p, p > 2, un numar prim si f un polinom cu coeficienti ıntregi, de grad p−1,cu proprietatea ca pentru orice a, b numere ıntregi pentru care p divide f(a) − f(b),rezulta ca p divide a− b. Aratati ca f are coeficientul dominant divizibil cu p.

Marian Andronache

Raspunsul la ”recreatia” de la pag. ... este:

Numarul maxim de operatii cu care se poate scrie 2009 este 4017 :

2009 = 10 : 10 + 10 : 10 + · · ·+ 10 : 10 (2009 termeni).Numarul minim de operatii este 209:

2009 = (10 + 10 + · · ·+ 10) + (10 : 10 + 10 : 10 + · · ·+ 10 : 10)( 200 termeni ın prima paranteza si 9 ın cea de-a doua).

129

Page 48: Revista (format .pdf, 5.8 MB)

Concursul de matematica ”Florica T. Campan”Etapa judeteana, 21 februarie 2009

Clasa a IV-a (Subiect elaborat de Dumitru Paraiala si Catalin Budeanu)

1. Fiica, tatal si bunica au ımpreuna 90 de ani. Peste doi ani, tatal va avea de optori varsta fiicei, iar bunica de doua ori varsta actuala a tatalui. Aflati varsta fiecaruiaın prezent.

2. Un blocnotes are 100 de pagini, numerotate de la 1 la 100. Se rup din acesta,la ıntamplare, 30 de foi, apoi se face suma numerelor ce marcheaza paginile ramase.Este posibil ca aceasta suma sa fie egala cu 800? Justificati!

3. O cantitate de 120 kg de prune a fost ımpartita ın mod egal ın mai multelazi. In vederea transportului, pentru a se evita pierderile, s-a transferat cate un sfertdin cantitatea de prune din fiecare lada, obtinadu-se astfel o noua lada cu tot atateakilograme de prune cate au ramas ın fiecare din lazile initiale.

a) Cate lazi au fost la ınceput?Justificati!b) Cate kilograme de prune erau initial ın fiecare lada?c) Cati lei s-au ıncasat din vanzarea prunelor din lada nou formata, daca un

kilogram de prune de calitatea ıntai costa 4 lei, un kilogram de prune de calitatea adoua costa 3 lei, iar la fiecare 2 kilograme de prune de calitatatea ıntai exista cate3kg de prune de calitatea a doua?

Clasa a V-a (Subiect elaborat de Gheorghe Iurea si Andrei Nedelcu)

1. Notam cu a1, a2, ....., a2009 numerele 1, 2, . . . , 2009, scrise ın alta ordine.Aratati ca numarul N = (1 + a1) · (2 + a2) · . . . · (2009 + a2009) este par. Este posibilca N sa nu fie divizibil cu 4 ?

2. Intr-un patrat 7 × 7 sunt scrise 49 numere naturale diferite. De pe fiecarelinie se considera cel mai mic numar, iar cel mai mare dintre aceste sapte numere senoteaza cu x. De pe fiecare coloana se considera cel mai mare numar, iar cel mai micdintre aceste sapte numere se noteaza cu y. Justificati ca x ≤ y.

3. a) Aratati ca numarul 3 · 102009 − 8 nu este patrat perfect.b) Aratati ca numarul 221506 are cel putin 2009 cifre.

Clasa a VI-a (Subiect elaborat de Claudiu Stefan Popa si Ciprian Baghiu)

1. a) Fie proportiaa

b=

c

d, cu toti termenii numere naturale, astfel ıncat d este

divizibil si cu b, si cu c, iar b si c sunt prime ıntre ele. Demonstrati ca bcd este patratperfect.

b) Daca un extrem al unei proportii cu toti termenii numere naturale nenule estedivizibil cu fiecare din mezii ei, aratati ca suma tuturor termenilor este divizibila cucelalalt extrem.

Claudiu Stefan Popa

130

Page 49: Revista (format .pdf, 5.8 MB)

2. Din cauza crizei economice, pretul unei marfi se reduce saptamanal cu 50%,timp de n saptamani, unde n ∈ N, n ≥ 2.

a) Consideram ca pretul initial al marfii este de 32 de lei. Stiind ca, atunci candeste cazul, se procedeaza la rotunjirea pretului dupa o anumita reducere, pentru cael sa poata fi platit cu unitatile monetare existente si ca ın a n-a saptamana pretulafisat este acelasi cu cel din saptamana n – 1, sa se afle n.

b) Presupunand acum ca preturile succesiv obtinute sunt exprimate fara rotunjiresi ca pretul initial al marfii este a lei, aratati ca suma ıntre preturile marfii din primelen – 1 saptamani si dublul pretului marfii din a n-a saptamana este a lei.

Claudiu Stefan Popa

3. Consideram triunghiul isoscel ABC, cu AB = AC sim(]BAC) < 900. PuncteleM si N sunt ın asa fel ıncat AB separa C si M, AC separa B si N, m(]MAB) =m(]NAC) = 900, iar AM = AN = AB. Aratati ca:

a) MC = NB;b) MB = NB;c) daca, ın plus, triunghiul MNB este isoscel, calculati m(]BAC).

Clasa a VII-a (Subiect elaborat de Vasile Nechita si Ionel Nechifor)

1. a) Fie a, b, c numere reale strict pozitive astfel ıncat 5a+3c = 4b si 2b=√15ac.

Sa se arate ca numerele a, b, c sunt direct proportionale cu numerele 6, 15 si 10.b) Determinati x ∈ N pentru care

√x2 + 9x+ 13 ∈ N.

2. a) Pe o masa sunt 20 monede. Radu si Bogdan joaca urmatorul joc: ia fiecare,pe rand, de pe masa, cate 1, 2 sau 3 monede; castigatorul este cel care ia ultimamoneda. Cine va castiga jocul? (justificati!) Dar daca numarul monedelor de pemasa este 21?

b) Fiind dat un paralelogram ABCD si o dreapta d, care taie dreptele AB, BC,

CD, DA ın punctele M, N, P, respectiv Q, demonstrati caMA

MB· NB

NC· PC

PD· QD

QA= 1.

3. Andrei, Bogdan, Costel si Dan cumpara trei torturi de ınghetata de formapatrata, care au fiecare aceeasi grosime si compozitie. Stim ca perimetrele torturilor

sunt invers proportionale cu numerele1

36,

1

160,

1

164, iar suma laturilor celor trei

torturi este 90cm. Explicati cum vor ımparti copiii cele trei torturi, pentru a luafiecare portii egale (fara cantarire).

Clasa a VIII-a (Subiect elaborat de Gabriel Popa si Cristian Lazar)

1. a) Determinati numerele ıntregi a si b cu proprietatea caa2b+ ab2 + a+ b+ 1 = 0.

Recreatii Matematice, 1/2000b) Fie n un numar natural dat. Determinati numerele reale strict pozitive x, y, z

pentru care xn = yz, yn = zx si zn = xy.

Neculai Hartan , Recreatii Matematice, 2/2003

2. Despre un numar natural n vom spune ca are proprietatea (P) daca√24n+ 1 ∈

Q si vom spune ca are proprietatea (Q) daca√24n+ k ∈ R\Q,∀k ∈ 2, 3, 4, ..., 99 .

131

Page 50: Revista (format .pdf, 5.8 MB)

a) Dati exemplu de un numar natural care are proprietatea (P), dar nu are pro-prietatea (Q).

b) Dati exemplu de un numar natural care are proprietatea (Q), dar nu are pro-prietatea (P).

c) Dati exemplu de un numar natural care nu are nici proprietatea (P), nici pro-prietatea (Q).

d) Determinati (cu justificare) cel mai mic numar natural care are atat proprietatea(P), cat si proprietatea (Q).

e) Gasiti toate numerele naturale care au atat proprietatea (P), cat si proprietatea(Q).

Cristian Lazar

3. Un zmeu din carton, care are forma unui triunghi ABC, lasa pe pamant oumbra avand forma unui triunghi A′B′C ′, care este asemenea cu triunghiul ABC.

a) Daca zmeul se afla sub razele soarelui la amiaza, demonstrati ca ∆ABC ≡∆A′B′C ′.

b) Daca zmeul se afla sub becul unui stalp de iluminat, mai rezulta ın mod necesarca ∆ABC ≡ ∆A′B′C ′?

Constantin Cocea, Gabriel Popa

Etapa interjudeteana, 21 martie 2009

Clasa a IV-a (Subiect elaborat de Dan Branzei si Catalin Budeanu)

1. Aflati suma resturilor ımpartirii a 2009 numere naturale consecutive la 15,stiind ca ultimul se ımparte exact la 15.

2. Suma a doua numere naturale este 26. Impartind primul numar la al doilea siapoi pe al doilea la primul se obtine, de fiecare data, aceeasi suma dintre cat si rest,aceasta fiind cu 8 mai mica decat unul dintre numere. Aflati numerele.

3. Fie numarul N = 510152025 . . . 725730735 . . . 20020052010.a) Cate cifre are numarul N?b) Care este a 1000-a cifra a lui N?

4. (facultativ) Centrul unei piete are forma unui patrat de latura 7 si este ocupatde o statuie cu un soclu patrat de latura 1. Se poate acoperi suprafata ramasa cupatru dreptunghiuri avand dimensiunile 3 si 4?

Clasa a V-a (Subiect elaborat de Gheorghe Iurea si Andrei Nedelcu)

1. La un turneu de fotbal ın sala participa patru echipe. Se acorda 2 punctepentru victorie, 1 punct pentru egal si 0 puncte la ınfrangere. Fiecare echipa joaca cufiecare cate un singur meci. In clasamentul final nu sunt doua echipe la egalitate depuncte. Care este numarul minim de puncte pe care ıl poate avea echipa castigatoare?Realizati o distributie a rezultatului meciurilor care sa corespunda situatiei de maisus.

132

Page 51: Revista (format .pdf, 5.8 MB)

2. Consideram multimea A = 2a · 3b · 5c|a, b, c ∈ N. Aratati ca printre oricare 9elemente ale multimii A exista cel putin doua a caror produs este patrat perfect.

3. Un patrat cu latura 5 se ımparte ın patrate cu latura 1, care se numeroteaza cunumere de la 1 la 25. Se calculeaza sumele de pe fiecare linie si de pe fiecare coloana.Exista o numerotare astfel ıncat exact o suma sa fie numar par?

4. (facultativ) Numarul 31 organizeaza o petrecere ın ımparatia Numerelor Nat-urale. Daca x ∈ N, atunci 9x – 2 si 9x + 2 sunt parintii lui x, iar 9x + 4 este bunicullui x. Se stie ca, daca bunicul unui numar se afla la petrecere, atunci este invitatsi nepotul sau. De asemenea, daca un numar este invitat la petrecere, atunci suntinvitati si parintii sai. Aratati ca numarul 2009 este invitat la petrecere.

Clasa a VI-a (Subiect elaborat de Ionel Nechifor si Ciprian Baghiu)

1. Sa se arate ca oricare ar fi m,n ∈ N, atuncim+ 2267

m+ 3178<

n+ 5388

n+ 7291.

2. Pe un teren de forma unui triunghi echilateral cu latura de 8m se planteaza 5brazi.

a) Demonstrati ca oricum am planta brazii, vor exista cel putin doi avand ıntreei o distanta nu mai mare de 4 metri.

b) In cate moduri se pot planta cei 5 brazi, astfel ıncat sa nu existe doi la distantastrict mai mica de 4 metri?

3. Fie a un numar natural nenul. Aratati ca exista b ∈ N∗ astfel ıncat 9b − 3b safie divizibil cu a.

Ciprian Baghiu

4.(facultativ) Determinati toate valorile numarului natural n, pentru care fractia2n2 + 5

3n2 + 5n+ 10este reductibila.

Clasa a VII-a (Subiect elaborat de Vasile Nechita si Claudiu Stefan Popa)

1. Luandu-se dupa harta care a apartinut piratului Supernegru, pentru a gasi co-moara fabuloasa (adica o supercomoara) ascunsa de acesta candva, undeva ın Caraibe,patru supercautatori trebuie sa caute ın interiorului patrulaterului NESV situat peun teren plat si obtinut astfel: din acelasi punct O, fiecare dintre cei 4 merge, ın liniedreapta; primul face b pasi spre nord pana ın N, al doilea c pasi spre est pana ın E, altreilea a pasi spre sud pana ın S si ultimul a pasi spre vest pana ın V (a,b,c numerenaturale). Stiind ca ıntre N si E sunt exact a pasi si ca lungimea pasului fiecaruiadintre cei 4 este aceeasi si constanta, aratati ca suprafata patrulaterului NESV arearia exprimata printr-un numar natural (unitatea de masura este patratul cu laturade 1 pas). Este posibil ca aria patrulaterului sa fie egala cu aria unui patrat de latura6 pasi ?

Claudiu Stefan Popa

2. a) Fie a, b, cnumere reale pozitive. Demonstrati ca a este media geometrica a

lui b si c daca si numai daca1

a=

1

a+ b+

1

a+ c.

133

Page 52: Revista (format .pdf, 5.8 MB)

b) Fie O intersectia diagonalelor patrulaterului convex ABCD. Demonstrati ca

AB|| CD daca si numai daca1

AAOD=

1

AACD+

1

AABD.

Claudiu Stefan Popa

3. Rezolvati ın multimea numerelor ıntregi ecuatia x2(y − 1) + y2(x− 1) = 1.Gabriel Mırsanu

Clasa a VIII-a (Subiect elaborat de Gabriel Popa si Cristian Lazar)

1. Se da multimea M = 1, 2, 3, 4 . Se formeaza toate sumele cu termeni distinctidin M, luand ın seama inclusiv sumele cu un singur termen. Doua sume se consideraa fi distincte daca difera fie prin cel putin un termen, fie prin ordinea termenilor (deexemplu, sumele 4, 1 + 3, 3 + 1, 1 + 2 + 3 sunt distincte).

a) Cate sume diferite se pot forma?b) Aratati ca orice numar cuprins ıntre 1 si 10 poate fi obtinut ca rezultat ın urma

calcularii unei astfel de sume.c) Doua numere cuprinse ıntre 1 si 10 vor fi numite ınrudite daca se obtin de acelasi

numar de ori ca rezultat al unor sume ca cele din enunt. Determinati perechile denumere ınrudite.

Gabriel Popa

2. Fie VABCD si SABCD doua piramide patrulatere regulate, avand ca bazacomuna patratul ABCD de latura 12

√2cm, varfurile V si S de o parte si de alta a

planului (ABCD) si ın care ınaltimile si muchiile laterale se exprima (ın centimetri)prin numere ıntregi.

a) Demonstrati ca punctele V, A, S si C sunt coplanare.b) Aflati valorile posibile ale ınaltimii VO a piramidei VABCD.c) Aratati ca patrulaterul VASC este circumscriptibil.d) Daca patrulaterul VASC este inscriptibil, determinati lungimea segmentului

VS.Cristian Lazar

3. Trei greieri ”sar capra”: un greiere, aflat ın punctul A, sare peste un alt greiere,aflat ın B, si ajunge ın C, unde C este simetricul lui A fata de B. Apoi, acelasi greieresau un altul sare peste un partener de joaca si tot asa. Daca initial cei trei greierise aflau ın trei dintre varfurile unui patrat, se poate ca la un moment dat, ın cursuljocului, unul dintre greieri sa ajunga ın cel de-al patrulea varf al acelui patrat?

Mircea Ganga

Vizitati noua pagina web a revistei:

http://www.recreatiimatematice.ro

134

Page 53: Revista (format .pdf, 5.8 MB)

Solutiile problemelor propuse ın nr. 2/2008

Clasele primareP.154. Dorina are 15 baloane rosii si albastre. Cate baloane rosii poate avea, daca

numarul acestora este mai mic decat numarul baloanelor albastre si este cel putin egalcu 3?(Clasa I ) Inst. Maria Racu, Iasi

Solutie. Numarul baloanelor rosii poate fi 3, 4, 5, 6 sau 7.

P.155. Dintr-o carte lipsesc cateva pagini, de la numarul 71 la numarul 94. Catefoi lipsesc din aceasta carte?(Clasa I ) Ionela Baragan, eleva, Iasi

Solutie. Prima foaie care lipseste are paginile 71 si 72, iar ultima are paginile 93si 94. In total lipsesc 12 foi.

P.156. La concursul ”Desene pe asfalt”, elevii claselor I-IV de la Scoala ”OtiliaCazimir” au acumulat 50 de puncte si cel putin 2 premii din fiecare categorie. Careeste cel mai mare numar de premii pe care-l pot obtine elevii, daca pentru premiulI s-au acordat 10 puncte, pentru premiul al II-lea s-au acordat 6 puncte, iar pentrupremiul al III-lea s-au acordat 4 puncte?(Clasa a II-a) Inv. Elena Porfir, Iasi

Solutie. Numarul maxim de premii se obtine ın cazul: 2 premii I, 3 premii II si3 premii III, ın total 8 premii.

P.157. Prin golirea unui singur vas, ales dintre cele de mai jos, putem face carestul vaselor sa aiba cantitati egale de lichid. Care vas trebuie golit?

(Clasa a II-a) Amalia Cantemir, eleva, IasiSolutie. Distribuim lichidul din vasul de 15 litri ın primele 6 vase astfel: 5 l, 4 l,

3 l, 2 l, 1 l, 0 l. In acest mod, ın fiecare vas vom avea 14 litri.

P.158. Aflati trei numere naturale stiind ca, adunandu-le doua cate doua, obtinem100, 89, respectiv 141.(Clasa a III-a) Inst. Maria Racu, Iasi

Solutie. Obtinem ca dublul sumei celor trei numere este 330, deci suma celor treinumere este 330 : 2 = 165. Astfel, primul numar este 165 − 100 = 65, al doilea este165− 89 = 76, iar al treilea este 165− 141 = 24.

P.159. Se considera numerele: a = 1 + 4 + 7 + 10 + · · · + 2008, b = 2 + 5 + 8++ · · ·+ 2009, c = 3 + 6 + 9 + · · ·+ 2010. Aratati ca suma a+ b+ c se ımparte exactla 3, fara sa calculati aceasta suma.(Clasa a III-a) Iuliana Moldovan, eleva, Iasi

135

Page 54: Revista (format .pdf, 5.8 MB)

Solutie. Putem scrie a + b + c = (1 + 2 + 3) + (4 + 5 + 6) + (7 + 8 + 9) + . . . +(2008+2009+2010). In fiecare paranteza avem suma a trei numere consecutive, carese ımparte exact la 3, deci a+ b+ c se ımparte exact la 3.

P.160. Numarul a este de forma xy0, iar numarul b este de forma uv. Sa se aflea si b stiind ca a+ b = 22 zeci.(Clasa a III-a) Dragos Toma, elev, Iasi

Solutie. Conditia din problema se scrie xy0 + uv = 220, din care se deducev = 0, iar xy+ u = 22. Pentru perechea (a, b) avem posibilitatile: (210, 10); (200, 20);(190, 30) . . . (130, 90).

P.161. Fie a si b doua numere naturale astfel ıncat diferenta lor este de 5 orimai mica decat suma lor. Sa se arate ca numarul cel mai mare se ımparte exact la3, iar cel mai mic se ımparte exact la 2.(Clasa a IV-a) Diana Tanasoaie, eleva, Iasi

Solutie. Fie a si b cele doua numere. Din a+ b = 5(a− b) obtinem 2a = 3b, decib trebuie sa fie par. Inlocuind b = 2c, rezulta ca a = 3c, prin urmare a se ımparteexact la 3.

P.162. Maria are 9 saculeti cu monede. Cel putin un saculet cantareste unkilogram. In orice grupare de 5 saculeti, cel putin 3 saculeti au aceeasi masa, iar ınorice grupare de 6 saculeti, cel mult 5 saculeti au aceeasi masa. Care este cel maimare numar de saculeti de 1 kg pe care ıl poate avea Maria?(Clasa a IV-a) Petru Asaftei, Iasi

Solutie. Nu putem avea mai mult de 5 saculeti de 1 kg, deoarece am gasi ogrupare de 6 saculeti de aceeasi masa. Numarul maxim posibil de saculeti de 1 kgeste 5 si poate fi atins, de exemplu, daca Maria are 5 saculeti de 1 kg si ınca 4 de oalta masa, aceeasi, caz ın care ar fi ındeplinita si prima conditie (conform principiuluicutiei).

P.163. Jumatatea produsului a doua numere naturale consecutive, ımpartita cu3, nu poate da niciodata restul 2.

Recreatii Stiintifice, Anul I (1883)- nr. 4, p.119Solutie. Daca unul dintre cele doua numere consecutive se ımparte exact la

3, restul ımpartirii din enunt este 0. In caz contrar, produsul celor doua numereconsecutive, care este par, va da restul 2 la ımpartirea prin 3, iar jumatatea se va dala ımpartirea prin 3 restul 1.

Clasa a V-aV.95. Doua numere naturale se scriu ın baza 10 folosind doar cifrele 1, 4, 6 si 9.

Poate fi unul dintre numere de 2008 ori mai mare decat celalalt?Catalin Budeanu, Iasi

Solutie. Daca numerele A si B se scriu doar cu ajutorul cifrelor 1, 4, 6 si 9, atunciU(2008A) ∈ 2, 8, prin urmare 2008A = B. Raspunsul la ıntrebarea din enunt estenegativ.

V.96. Determinati k, n ∈ N∗ astfel ıncat (1 + 1 · n)+(2 + 2 · n)+· · ·+(k + k · n) =3 · 4 · 5 · 6.

Petru Asaftei, Iasi

136

Page 55: Revista (format .pdf, 5.8 MB)

Solutie. Deoarece (1 + 1 · n) + (2 + 2 · n) + . . . + (k + k · n) = (1 + 2 + . . . +

k) + n(1 + 2 + . . . + k) = (1 + 2 + . . . + k)(n + 1) si 1 + 2 + . . . + k =k(k + 1)

2,

ecuatia data este echivalenta cu k(k + 1)(n + 1) = 2 · 3 · 4 · 5 · 6. Atunci k si k + 1sunt divizori ai numarului 2 · 3 · 4 · 5 · 6, iar k(k + 1) ≤ 2 · 3 · 4 · 5 · 6. Gasim solutiile(k, n) ∈ (1, 359); (2, 119); (3, 59); (4, 35); (5, 23); (8, 9); (9, 7); (15, 2).

V.97. Aratati ca numarul N = 17n+21n+25n, n ∈ N, nu poate fi patrat perfect.

Virginia Grigorescu, Craiova

Solutie. Deoarece (Ma+1)n = Ma+1, deducem ca 17n = M4+1, 21n = M4+1,25n = M4 + 1, deci N = M4 + 3, prin urmare N nu poate fi patrat perfect.

V.98. Fie n ∈ N∗. Sa se demonstreze ca numarul N = 5050 . . . 505 (2n+1 cifre)se scrie ca suma a 4n+ 2 patrate perfecte distincte.

Veronica Plaesu si Dan Plaesu , Iasi

Solutie. Observam ca N = 5 ·102n+5 ·102n−2+ . . .+5 ·102+5. Cum 5 = 22+12,iar 5 · 102k = 500 · 102k−2 = (162 + 122 + 82 + 62) · (10k−1)2 = (16 · 10k−1)2 + (12 ·10k−1)2 + (8 · 10k−1)2 + (6 · 10k−1)2, cerinta problemei este demonstrata.

V.99. Se considera numarul N = 1 + 11 + 101 + 1001 + · · ·+ 100 . . . 01| z n cifre

.

a) Pentru n ∈ N, n ≥ 5, aratati ca 5 | N ⇔ 5 | n.b) Precizati care dintre propozitiile ”3 | n ⇒ 3 | N” si ”3 | N ⇒ 3 | n” este

adevarata pentru orice n ≥ 3.

Temistocle Bırsan, Iasi

Solutie. a) CumN = 1+10+100+. . .+100 . . . 0| z ncifre

+1 + 1 + . . .+ 1| z n−1 termeni

= 11 . . . 11| z ncifre

+(n−

1) = 11 . . . 10| z ncifre

+n. Cum 5| 11 . . . 10| z n

, rezulta cerinta a).

b) Niciuna dintre implicatii nu este adevarata. Intr-adevar, pentru n = 3 avem ca

N = 1 + 11 + 101 = 113.../ 3. Apoi, pentru n = 5 avem ca N = 1 + 11 + 101 + 1001 +

10001 = 11115...3, ın timp ce 5

.../ 3.

V.100. Determinati numerele naturale nenule a si b pentru care exista n ∈ Nastfel ıncat

a

b=

3n+ 2

7n+ 5si 3a+ 2b < 100.

Gheorghe Iurea, Iasi

Solutie. Daca d = (3n+2, 7n+5), atunci d|3n+2 si d|7n+5, de unde d|3(7n+

5)− 7(3n+ 2), cu alte cuvinte d = 1, deci fractia3n+ 2

7n+ 5este ireductibila. Deoarece

a

b=

3n+ 2

7n+ 5, deducem ca a = k(3n+2) si b = k(7n+5), unde k ∈ N∗, iar inegalitatea

3a + 2b < 100 devine k(23n + 16) < 100. Pentru n = 0 obtinem k ∈ 1, 2, 3, 4, 5, 6,a = 2k si b = 5k; pentru n = 1 avem k ∈ 1, 2, a = 5k si b = 12k. Daca n = 2,atunci k = 1, a = 8, b = 19, iar pentru n = 3, k = 1, a = 11, b = 26. Pentru n ≥ 4,problema nu admite solutii.

137

Page 56: Revista (format .pdf, 5.8 MB)

V.101. Consideram fractiaan+ b

cn+ d, unde n, a, b, c, d ∈ N∗, astfel ıncat b si d au

paritati diferite, iar a si c au aceeasi paritate. Aratati ca, daca ad− bc = 2k, k ∈ N∗,atunci fractia este ireductibila.

Cosmin Manea si Dragos Petrica, PitestiSolutie. Presupunem prin absurd ca fractia este reductibila. Fie p un divizor

comun al numerelor an+b si cn+d, p ∈ N∗, p ≥ 2; rezulta ca p|(a ·n+b) si p|(c ·n+d),de unde p|ad− bc, deci p|2k si atunci p este par. Totodata, p|n(a+ c) + b+ d si cumn(a+ c) + b+ d este numar impar, rezulta ca p este impar, contradictie.

Clasa a VI-aVI.95. Determinati numerele naturale nenule a1, a2,. . . , a2008, stiind ca

a1a21 · 2

=a2a32 · 3

= · · · = a2007a20882007 · 2008

, iar a1 + a2008 = 2009.

Gheorghe Iurea, Iasi

Solutie. Considerand rapoartele doua cate doua, obtinem caa11

=a33

= . . . =a20072007

, iara22

=a44

= . . . =a20082008

. Astfel, relatia a1 + a2008 = 2009 devine a1 +

1004a2 = 2009 si cum a1, a2 ∈ N∗, atunci a1 = 1, a2 = 2. Deducem ca ai = i,∀i = 1, 2008.

VI.96. Determinati p ∈ N pentru care numerele p, p+12, p+22, p+52, p+72,p+ 102 si p+ 132 sunt prime.

Damian Marinescu, TargovisteSolutie. Pentru p = 7, numerele sunt 7, 19, 29, 59, 79, 109, 139 si sunt toate prime.

Daca p ∈ 2, 3, 5, obtinem numerele compuse 2 + 12, 3 + 12, respectiv 5 + 22. Dacap ≥ 11, considerand p = M7 + r, cu r ∈ 0, 1, . . . , 6, gasim ın fiecare caz cate unnumar compus printre cele date. In concluzie, singura valoare convenabila este p = 7.

VI.97. a) Daca a, b, c, d, e, f ∈ N∗ sunt astfel ıncat (a, b) = (c, d) = (e, f) =

(b, d) = 1, iar t =a

b− c

d+

e

f∈ N, aratati ca f = bd.

b) Determinati a, b ∈ N pentru care4

2a+ 1− 1

2b+

7

6∈ N.

Cosmin Manea si Dragos Petrica, Pitesti

Solutie. a) Deoarece t · bd = ad − bc+ bd · ef

∈ N, rezulta ca bd · ef

∈ N si, cum

(e, f) = 1, deducem ca f |bd. Pe de alta parte, din t · bf = af − bf · cd+ ed ∈ N si

t · df = df · ab− cf + de ∈ N, rezulta ca bf · c

dsi df · a

bsunt numere naturale. Folosind

faptul ca (b, a) = (d, b) = (d, c) = 1, obtinem ca d|f si b|f si, cum (b, d) = 1, atuncibd|f . Din f |bd si bd|f , rezulta ca f = bd.

b) Suntem ın conditiile punctului precedent; deducem ca 6 = (2a+1) ·2b, de undea = b = 1. Pentru aceste valori, vom avea t = 2 ∈ N.

VI.98. Determinati cel mai mic numar natural n cu proprietatea ca numarulzerourilor ın care se termina numarul (n+ 10)! este cu 2008 mai mare decat numarulzerourilor ın care se termina n! (unde n! = 1 · 2 · 3 · · ·n).

Catalin Budeanu, Iasi

138

Page 57: Revista (format .pdf, 5.8 MB)

Solutie. Ipoteza problemei revine la faptul ca numarul (n+1)(n+2) . . . (n+10)se termina ın 2008 zerouri, ceea ce se ıntampla atunci cand produsul (n + 1)(n +2) . . . (n + 10) se divide cu 52008 si nu se divide cu 52009. Printre factorii produsuluiprecedent, exista exact doi care sunt divizibili cu 5, iar dintre acestia unul nu mai estedivizibil cu nicio alta putere a lui 5, deci celalalt se va divide cu 52007. Cum dorim nminim, atunci n+ 10 = 52007, prin urmare n = 52007 − 10.

VI.99. Un patrulater convex are doua laturi opuse congruente si diagonalelecongruente. Aratati ca patrulaterul este trapez isoscel sau dreptunghi.

Ioan Sacaleanu, HarlauSolutie. Presupunem ca (AB) ≡ (CD) si (AC) ≡ (BD). Din congruenta tri-

unghiurilor ABC si DCB, rezulta ca ÕABC ≡ ÕDCB, iar din congruenta triunghiurilor

ABD si DCA, rezulta ca ÕBAD ≡ ÕCDA. Cum suma unghiurilor unui patrulater este

360, deducem ca m(ÕABC) +m(ÕBAD) = 180, deci AD∥BC.Daca AB∥CD, atunci ABCD este un paralelogram cu diagonalele congruente,

deci este un dreptunghi. Daca AB ∦ CD, atunc ABCD este un trapez cu diagonalelecongruente, adica un trapez isoscel.

VI.100. Fie ABC cu m( bA) ≥ 90. Sa se arate ca m(ÒB) = 2m(ÒC) daca sinumai daca exista M ∈ [BC] astfel ıncat AB = AM = MC.

Petru Asaftei, IasiSolutie. Presupunem ca m(ÒB) = 2m(ÒC) si fie M intersectia mediatoarei laturii

B C

A

M

[AC] cu BC; cum m( bA) ≥ 90, vom avea ca M ∈ [BC].Evident atunci ca MAC este isoscel cu MA = MC si

cum ÖAMB este unghi exterior, deducem ca m(ÖAMB) =

2m(ÒC) = m(ÒB). Astfel, ABM va fi isoscel cu AB =AM, ceea ce ıncheie justficarea afirmatiei directe.

Reciproc, daca exista M ∈ [BC] cu AB = AM = MC, din triunghiurile isoscele

ABM si MAC obtinem ca m(ÒB) = m(ÖAMB) = 2m(ÒC).

VI.101. Fie ABC un triunghi dreptunghic cu m( bA) = 90 si CD bisectoarea

unghiului ÒC, D ∈ (AB). Perpendiculara din D pe bisectoarea unghiului ÒB inter-secteaza ipotenuza BC ın E. Daca P este punctul de intersectie a bisectoarelor unghi-urilor triunghiului ABC, iar M este punctul de intersectie dintre EP si AC, aratati

ca ÖMPA ≡ ÕPBE.Nela Ciceu, Bacau si Titu Zvonaru, Comanesti

Solutie. Din enunt rezulta ca triunghiul BDE este isoscel si ca BP este medi-

A B

C

D

E

M

P

atoarea segmentului DE. Totodata, m(ÕCDE) = m(ÕCDB) −m(ÕEDB) = 180 − 1

2m(ÒC) − m(ÒB) − 1

2[180 − m(ÒB)] = 45.

Prin urmare, triunghiul DPE este dreptunghic isoscel. Astfel,CP ⊥ ME si cum CP este bisectoarea unghiului ÒC, rezulta

ca triunghiul CME este isoscel. Deducem ca m(ÖMPA) =

180 − m(ÖPMA) − 45 = 135 − 180 + m(ÖCME) = −45 +1

2[180 −m(ÒC)] =

1

2[90 −m(ÒC)] =

1

2m(ÒB) = m(ÕPBE).

139

Page 58: Revista (format .pdf, 5.8 MB)

Clasa a VII-aA B

CD

N

M

VII.95. Fie ABCD patrat, M un punct oarecare pe (AB),iar N ∈ (BC) este astfel ıncat MN ⊥ MD. Aratati ca AM ·AB + CN · CB = DM2.

Ovidiu Pop, Satu MareGh. Szollosy, Sighetul Marmatiei

Solutie. Fie a latura patratului x = AM si y = CN ; atunciDM2+MN2 = DN2 ⇔ a2+x2+(a−x)2+(a−y)2 = a2+y2 ⇔2a2 − 2ax+ 2x2 − 2ay = 0 ⇔ a2 + x2 = ax+ ay ⇔ DM2 = AM ·AB + CN · CB.

A

B CD

E

F

P

M

.

.

.

VII.96. Fie [AD] mediana ın ABC, M mijlocul lui [AD],E = BM ∩ AC, iar punctul F pe dreapta AB este astfel ıncatCF ∥ AD. Demonstrati ca punctele D, E si F sunt coliniare.

Mirela Marin, IasiSolutie. Cum ADCF este trapez, M este mijlocul bazei mici,

iar B este punctul de intersectie a laturilor neparalele, ınseamnaca P = BM ∩CF este mijlocul lui [CF ]. Cu reciproca teoremeiliniei mijlocii se arata ca A este mijlocul lui [BF ], prin urmareCA si BP sunt mediane ın BCF , iar E va fi centrul de greutateal acestui triunghi. Coliniaritatea dorita rezulta observand ca a treia mediana atriunghiului este FD.

VII.97. Fie C1 (O1, r1) si C2 (O2, r2), r1 < r2, doua cercuri tangente exterior.Consideram punctele A′ ∈ C1, B′ ∈ C2, de aceeasi parte a dreptei O1O2, astfel ıncatA′O1 ∥ B′O2. Daca AB este tangenta comuna exterioara a cercurilor (A ∈ C1,B ∈ C2), demonstrati ca dreptele AB, A′B′ si O1O2 sunt concurente.

Romanta Ghita si Ioan Ghita, BlajSolutie. Notam P = AB ∩ O1O2, P ′ = A′B′ ∩ O1O2. Din asemanarile

PO1A ∼ PO2B si P ′O1A′ ∼ P ′O2B

′ obtinem caPO1

PO2=

P ′O1

P ′O2=

r1r2

< 1.

A

B

B A PP

O1

O2

.

.

Astfel, punctele P si P ′ se afla de aceeasi parte cu O1 pe dreapta O1O2 si ımpartsegmentul [O1O2] ın acelasi raport, prin urmare P = P ′ si de aici rezulta concurentadorita.

VII.98. Sa se determine numerele naturale nenule a si b, stiind ca sunt directproportionale cu b− 6 si a si invers proportionale cu a+ 12 si b.

Constantin Apostol, Rm. Sarat

Solutie. Din datele problemei obtinem caa

b− 6=

b

asi a(a+12) = b2. Din prima

ecuatie rezulta ca a2 − b2 = −6b, iar din a doua a2 − b2 = −12a, deci −6b = −12a.

140

Page 59: Revista (format .pdf, 5.8 MB)

Atunci b = 2a si, folosind prima ecuatie, gasim ca a = 4 si apoi b = 8.

VII.99. Fie a, b ∈ Z si numerele A = 119a5 + 5b3 − 4a si B = 119b5 + 5a3 − 4b.Sa se arate ca A se divide cu 120 daca si numai daca B se divide cu 120.

Dan Nedeianu, Drobeta-Tr. SeverinSolutie. Avem ca A+B = 120(a5 + b5)− (a5 − 5a3 +4a)− (b5 − 5b3 +4b). Cum

a5 − 5a2 + 4a = (a− 2)(a− 1)a(a+ 1)(a+ 2) se divide la 120, rezulta ca A+ B...120

si concluzia de impune.

VII.100. Aratati ca 2a2 + 15b2 + 7c2 ≥ 10ab− 6ac+ 20bc, ∀a, b, c ∈ R.Alexandru Negrescu, student, Iasi

Solutie. Inegalitatea se scrie: 4a2 − 4a(5b − 3c) + 30b2 + 14c2 − 40bc ≥ 0 ⇔(2a−5b+3c)2+30b2+14c2−40bc− (5b−3c)2 ≥ 0 ⇔ (2a−5b+3c)2+5(b− c)2 ≥ 0,evident adevarat. Egalitate avem pentru a = b = c.

VII.101. Pentru n ∈ N∗, notam cu d (n) numarul divizorilor primi ai lui n.a) Determinati cardinalul multimii A = n ∈ N∗ | n ≤ 208, d (n) = 3.b) Aflati cel mai mic si cel mai mare element al multimii

B = k ∈ N | ∃n ∈ N∗, n ≤ 2008, a.ı. d (n) = k .Gabriel Popa, Iasi

Solutie. a) Observam ca n ∈ A daca si numai daca n = pα11 · pα2

2 · pα33 , cu

p1 < p2 < p3 numere prime, iar α1, α2, α3 ∈ N∗. Cum 5 ·7 ·11 = 385 > 208, ınseamnaca p1 ∈ 2, 3, prin urmare

(p1, p2, p3) ∈ (2, 3, 5); (2, 3, 7); (2, 3, 11); (2, 3, 13); (2, 3, 17); (2, 3, 19); (2, 3, 23);(2, 3, 29); (2, 3, 31); (2, 5, 7); (2, 5, 11); (2, 5, 13); (2, 5, 17); (2, 5, 19);

(2, 7, 11); (2, 7, 13); (3, 5, 7); (3, 5, 11); (3, 5, 13).Pentru tripletele subliniate, ın produsul n = pα1

1 · pα22 · pα3

3 vom avea obligatoriuα1 = α2 = α3 = 1; obtinem astfel 13 elemente ale lui A.

Daca (p1, p2, p3) = (2, 3, 5), ın n = 2α1 · 3α2 · 5α3 putem avea α3 = 1, (α1, α2) ∈(1, 1); (2, 1); (3, 1); (1, 2); (2, 2) sau α3 = 2, α1 = α2 = 1; gasim ınca 6 elemente dinA. Daca (p1, p2, p3) = (2, 3, 7), ın n = 2α1 · 3α2 · 7α3 este obligatoriu ca α3 = 1, iar(α1, α2) ∈ (1, 1); (2, 1); (3, 1); (1, 2); obtinem 4 noi elemente ale lui A.

Daca (p1, p2, p3) = (2, 3, 11), atunci α3 = 1, (α1, α2) ∈ (1, 1); (2, 1); (1, 2), deciınca 3 elemente. Daca (p1, p2, p3) ∈ (2, 3, 13); (2, 3, 17); (2, 5, 7), vom avea ın fiecarecaz (α1, α2, α3) ∈ (1, 1, 1); (2, 1, 1), adica ınca 3× 2 = 6 elemente.

In total, |A| = 13 + 6 + 4 + 3 + 6 = 32.b) Evident ca minB = 0, atins pentru n = 1.Aratam ca maxB = 4: daca ar exista n ≤ 2008 cu d(n) ≥ 5, atunci n ≥

p1p2p3p4p5 ≥ 2 · 3 · 5 · 7 · 11 = 2310 > 2008, absurd, iar pentru n = 2 · 3 · 5 · 7avem d(n) = 4.

Clasa a VIII-a

VIII.95. Pentru a, b, c ∈ R∗, notam α =a

b+

b

c+

c

a, β =

a

c+

c

b+

b

a. Calculati

numarul x =a3

b3+

b3

c3+

c3

a3ın functie α si β.

Elena Nicu, Malu-Mare (Dolj)

141

Page 60: Revista (format .pdf, 5.8 MB)

Solutie. Fie x =a

b, y =

b

c, z =

c

a; atunci xyz = 1, x + y + z = α, iar

xy + yz + zx

xyz=

1

x+

1

y+

1

z= β, de unde xy + xz + yz = β. Folosind identitatea

x3 + y3 + z3 − 3xyz = (x+ y + z)3 − 3(x+ y + z)(xy + xz + yz),

rezulta ca x3 + y3 + z3 = α3 − 3αβ + 3.

VIII.96. Rezolvati ın numere naturale ecuatia x2 + y2 + xy = x2y2.Mihail Bencze, Brasov

Solutie. Putem presupune ca x ≤ y. Daca x = 0, obtinem ca y = 0. Daca x = 1,deducem ca y = −1 /∈ N. Pentru x = 2, gasim ca 3y2−2y−4 = 0, ecuatie care nu aresolutii naturale. In cazul ın care x ≥ 3, atunci y ≥ 3 si vom avea ca x2 − 1 > x+ 1,y2−1 > y+1, inegalitati care, ınmultite, conduc la (x2−1)(y2−1) > (x+1)(y+1). Insa(x+1)(y+1) > xy+1, prin urmare (x2−1)(y2−1) > xy+1, adica x2+y2+xy < x2y2.In concluzie, singura solutie a ecuatiei date este (x, y) = (0, 0).

Solutia 2. Vom rezolva ecuatia ın multimea numerelor ıntregi. Fie (x, y) o solutie.Scriind ecuatia sub forma (x+ y)2 = xy(xy + 1), deducem ca xy(xy + 1) este patratperfect. Cum pentru xy ≥ 1, (xy)2 < xy(xy + 1) < (xy + 1)2, iar pentru xy ≤ −2(xy + 1)2 < xy(xy + 1) < (xy)2, nu avem solutii ın aceste cazuri. Rezulta ca xy = 0sau xy = −1. Obtinem solutiile: (0, 0); (1,−1) si (−1, 1).

Solutia 3 (Gheorghe Iurea). Fie (x, y) o solutie cu x, y ∈ Z. Ecuatia esteechivalenta cu (x+ y)2 = (xy)2+xy ⇔ 4(x+ y)2 = (2xy+1)2− 1 ⇔ [2xy+1− 2(x+y)][2xy + 1 + 2(x+ y)] = 1, deci 2xy + 1− 2(x+ y) = 2xy + 1 + 2(x+ y) ∈ −1, 1.Obtinem solutiile (0, 0); (1,−1) si (−1, 1).

VIII.97. Fie d1, d2, d3, d lungimile diagonalelor fetelor, respectiv diagonalei

unui paralelipiped dreptunghic. Daca d21 =2d22d

23

d22 + d23, sa se arate ca paralelipipedul are

o muchie de lungime cel putin egala cud√3

3.

Gheorghe Molea, Curtea de ArgesSolutie. Fie d21 = a2+ b2, d22 = b2+ c2, d23 = a2+ c2, d2 = a2+ b2+ c2, unde a, b, c

sunt lungimile muchiilor paralelipipedului. Relatia din ipoteza se scrie succesiv:

d21 =2d22d

23

d22 + d23⇔ a2 + b2 =

2(b2 + c2)(c2 + a2)

b2 + c2 + c2 + a2⇔ a4 + b4 = 2c4.

Insa 2(a4 + b4) ≥ (a2 + b2)2, deci 4c4 ≥ (a2 + b2)2 ⇔ 2c2 ≥ a2 + b2 ⇔ 3c2 ≥

a2+ b2+ c2 ⇔ 3c2 ≥ d2, deci c ≥ d√3

3. Avem egalitate pentru a = b = c, deci ıntr-un

cub.

VIII.98. Fie VABCD piramida patrulatera regulata. Notam u=m( Û(V BC),(ABC)),

v = m( Û(V BC) , (V CD)) si t = m( Û(V BC) , (V AD)). Aratati ca u+ v + t > 180.Claudiu Stefan Popa, Iasi

Solutie. Fie M si N mijloacele laturilor [BC], respectiv [AD], iar P proiectia

142

Page 61: Revista (format .pdf, 5.8 MB)

A B

CD

N O

P

V

.

.

..M

lui B pe V C; se arata ca u = m(ÖVMN), v = m(ÕBPD), iar

t = m(ÖMVN). In VMN avem ca u + u + t = 180

si atunci concluzia problemei rezulta daca am arata cav > u. Evident ca u < 90, prin urmare daca v ≥ 90,demonstratia este ıncheiata. Presupunem ca v < 90;

atunci v > u ⇔ sin v > sinu ⇔ BD · PO

BP 2>

V O

VM. Cum

PO =V O ·BD

2V C, iar BP =

VM ·BC

V C, ultima inegali-

tate revine laBD2 · V C

2BC2 · VM> 1. Insa BD2 = 2BC2, iar

V C > VM si astfel solutia problemei este completa.

VIII.99. Pentru n ∈ N∗, consideram A =12, 22, 32, . . . , n2

. Determinati n,

stiind ca exista o functie f : A → A astfel ıncat f (x)− f (y) =√x−√

y, ∀x, y ∈ A.Cristian Lazar, Iasi

Solutie. Avem ca f(x) −√x = f(y) − √

y, ∀x, y ∈ A, deci ∃k ∈ R astfel ıncatf(x) −

√x = k, ∀x ∈ A, de unde f(x) =

√x + k, ∀x ∈ A. Deducem ca, pentru

x < y din A, avem ca f(x) < f(y) si atunci, cum A este finita, rezulta ca f(12) = 12,f(22) = 22, . . . , f(n2) = n2, de unde 1 + k = 1 si n+ k = n2. Deci, k = 0 si n = 1.

VIII.100. Rezolvati ın ın N2 ecuatia x2 − 8n + 1287 = 0.Mihai Craciun, Pascani

Solutie. Daca n este impar, atunci 8n = (9 − 1)n = M9 − 1. Cum x2 da la

ımpartirea prin 9 unul dintre resturile 0, 1, 4 sau 7, iar 1287...9, cantitatea din membrul

stang al ecuatiei da la ımpartirea prin 9 unul dintre resturile 1, 2, 5 sau 8 si se ajungela o contradictie.

Daca n = 2k, ecuatia se scrie: (8k − x)(8k + x) = 32 · 11 · 13. Analizand pe randcazurile care se obtin, gasim solutie doar cand 8k−x = 11, 8k+x = 32 ·13, cand vomavea k = 2, x = 53. Deci, singura solutie a ecuatiei date este (n, x) = (4, 53).

VIII.101. Se calculeaza suma cifrelor pentru fiecare dintre numerele de la 1la n, n > 10. Pentru fiecare suma dintre cele n se calculeaza din nou suma cifrelor,repetandu-se aceasta operatie pana cand obtinem n numere formate din cate o singuracifra. Sa se afle n, stiind ca ın multimea astfel obtinuta cifrele 1, 2, 3 si 4 se repetade cate 101 ori fiecare, iar cifrele 5, 6, 7, 8 si 9 de cate 100 ori fiecare.

Mihai Haivas, IasiSolutie. Restul ımpartirii unui numar prin 9 este egal cu restul ımpartirii sumei

cifrelor sale prin 9. Atunci, cifra 1 se obtine din acele numere care dau restul 1la ımpartirea prin 9, adica din numerele 1, 10, 19 . . . . Pentru a obtine 1 de 101 ori,

trebuie sa avemhn9

i+ 1 = 101, deci n ∈ 900, 901, . . . , 904. Se verifica faptul ca

doar n = 904 satisface si celelalte conditii din ipoteza.

Clasa a IX-aIX.91. Fie a, b, c, p ∈ R, p > 0. Daca

ax2 + bx+ c ≤ p, ∀x ∈ [−1, 1], atuncicx2 + bx+ a

≤ 2p, ∀x ∈ [−1, 1].Dorin Marghidanu, Corabia

143

Page 62: Revista (format .pdf, 5.8 MB)

Solutie. Fie f(x) = ax2 + bx + c, g(x) = cx2 + bx + a, x ∈ [−1, 1]. Din ipotezase obtine ca |a + b + c| = |f(1)| ≤ p, |a − b + c| = |f(−1)| ≤ p, iar |c| = |f(0)| ≤ p.Folosind inegalitatea modulului, avem:

|g(x)| = |cx2 + bx+ a| = |c(x2 − 1) + (a+ b+ c) · x+ 1

2+ (a− b+ c) · 1− x

2| ≤

≤ |c| · |x2 − 1|+ |a+ b+ c| · |x+ 1|2

+ |a− b+ c| · |1− x|2

≤ p ·|x2 − 1|+ |x+ 1|

2+

|1− x|2

= p

1− x2 +

x+ 1

2+

1− x

2

=

= p(2− x2) ≤ 2p, ∀x ∈ [−1, 1].

IX.92. Fie n ∈ N, n ≥ 3, iar α, β ∈ R astfel ıncat n+ α+ β = 0. Aratati ca

(1 + α) · · · (n+ α)

n+ α+ β− (1 + α) · · · (n− 1 + α) +

n−2Xi=1

(−1)i+1

(1 + α) · · · (n− i− 1 + α)×

×β (β + 1) · · · (β + i− 1) + (−1)nβ · · · (β + n− 2) = (−1)

n β (β + 1) · · · (β + n− 1)

n+ α+ β.

Gheorghe Costovici, IasiSolutie. Notam cu Sn membrul stang al egalitatii. Pentru n = 3, avem ca

S3 =(1 + α)(2 + α)(3 + α)

3 + α+ β− (1 + α)(2 + α) + (1 + α) · β −

−β(β + 1) = . . . = −β(β + 1)(β + 2)

3 + α+ β,

calculele fiind de rutina. Fie n ≥ 4, iar σn−2 = Sn − (−1)nβ(β + 1) . . . (β + n − 2).Ar fi suficient sa dovedim ca

(∗) σk = (−1)k−1(1+α) . . . (n−k−1+α)β(β+1) . . . (β+k)· 1

n+ α+ β, ∀k ∈ 1, n− 2

odata demonstrata (∗), vom obtine ca

Sn = σn−2 + (−1)nβ(β + 1) . . . (β + n− 2) =

= (−1)nβ(β + 1) . . . (β + n− 2)

1− 1 + α

n+ α+ β

=

= (−1)n · β(β + 1) . . . (β + n− 1)

n+ α+ β,

adica tocmai ceea ce trebuia demonstrat.Justficam (∗) prin inductie dupa k:

σ1 =(1 + α) . . . (n+ α)

n+ α+ β− (1 + α) . . . (n− 1 + α) + (1 + α) . . . (n− 2 + α)β =

= (1 + α) . . . (n− 1 + α) · −β

n+ α+ β+ (1 + α) . . . (n− 2 + β) · β =

=(1 + α) . . . (n− 2 + α)β(β + 1)

n+ α+ β,

144

Page 63: Revista (format .pdf, 5.8 MB)

deci (∗) este adevarata pentru k = 1. Presupunem (∗) adevarata pentru k ∈ 1, 2, . . . , n−3; atunci

σk+1 = σk + (−1)k(1 + α) . . . (n− k − 2 + α)β(β + 1) · . . . · (β + k) =

= (−1)k(1 + α) . . . (n− k − 2 + α)β · . . . · (β + k)

1− n− k − 1 + α

n+ α+ β

=

= (−1)k(1 + α) . . . (n− k − 2 + α)β · . . . · (β + k + 1) · 1

n+ α+ β,

deci (∗) este adevarata si pentru k + 1 si astfel solutia problemei este completa.

IX.93. Fie ABC dreptunghic cu m( bA) = 90 siAB

AC=

3

2, iar D mijlocul lui

[AC]. Notam cu E punctul de intersectie a cercurilor C1 (A,AD) si C2 (B,BC), aflat

de aceeasi parte a dreptei AB ca si punctul C. Determinati masura unghiului ÕCAE.Catalin Tigaeru, Suceava

Solutie. Fie M,N = AB ∩ C1, cu N ∈ (AB), iar a = AD; atunci AC = 2a,

A B

C

DE

M N. .

..

AB = 3a, BC = a√13, BD = a

√10, iar BM = 4a.

Cum BD < BC < BM, ınseamna ca cercurile C1 si

C2 sunt secante, iar 90 < m(ÕBAE) < 180. Aplicamteorema cosinusului ın ABE : BE2 = AB2+AE2−2AB ·AE · cos(ÕBAE) ⇔ 13a2 = 9a2 + a2 − 2 · 3a · a ·cos(ÕBAE), de unde obtinem cos(ÕBAE) = −1

2, adica

m(ÕBAE) = 120. Deducem ca m(ÕCAE) = 120 − 90 = 30.

IX.94. In ABC, I este centrul cercului ınscris, iar M = AI∩BC. Demonstrati

ca bisectoarea unghiului ÖAMC, BI si AC sunt trei drepte concurente daca si numaidaca m( bA) = 120.

Vlad Emanuel, student si Andrei Cozma, elev, BucuretiSolutie. Notam cu a, b, c lungimile laturilor si fie P = BI ∩ AC; avem ca

A

B C

I

P

M

AM =2bc

b+ c·cos A

2,MC =

ab

b+ c, iar

AP

PC=

c

a. Atunci:

BI, AC si bisectoarea unghiului ÖAMC sunt concurente

⇔ MP este bisectoarea lui ÖAMC ⇔ AP

PC=

AM

MC⇔

c

a=

2c

acos

A

2⇔ cos

A

2=

1

2⇔ m( bA) = 120.

IX.95. Daca xi ∈ [0, a], i = 1, n si xn+1 = x1, demonstrati ca

nXi=1

xi+1 (a− xi) <na2

4 sin2π

n

.

Gigel Buth, Satu MareSolutie. Consideram poligonul regulat de latura a, A1A2 . . . An, ınscris ın cercul

C(O, r). Luam pe fiecare latura [AiAi+1] cate un punct Pi astfel ıncat AiPi = xi, i =1, n (cu conventia An+1 = A1). Daca Si este aria triunghiului PiAi+1Pi+1, iar S este

145

Page 64: Revista (format .pdf, 5.8 MB)

aria poligonului, atuncinX

i=1

Si < S, iar Si =xi+1(a− xi)

2· sin 2π

n, S =

na2

4 sinπ

n

·cos πn.

Obtinem ca

2 sinπ

ncos

π

n· 12·

nXi=1

xi+1(a− xi) <na2

4 sinπ

n

· cos πn,

de unde rezulta concluzia problemei.

Clasa a X-aX.91. Aratati ca

arctg1

7

2+arctg

3

4

2+

π2

16

2= 2

arctg

1

7

4+arctg

3

4

4+

π4

256

.

D. M. Batinetu-Giurgiu, BucurestiSolutie. Se demonstreaza relativ simplu identitatea

[x2 + y2 + (x+ y)2]2 = 2[x4 + y4 + (x+ y)4],

cunoscuta sub numele de identitatea lui G. Candido. Concluzia problemei rezulta

observand ca, daca x = arctg1

7si y = arctg

3

4, atunci tg(x + y) =

tg x+ tg y

1− tg x tg y= 1,

deci x+ y =π

4(deoarece x+ y <

π

2).

X.92. Fie a, b ∈ C. Demonstrati ca ecuatia z2 − az + b = 0 are ambele solutii demodul 1 daca si numai daca |b| = 1 si |a|2 +

a2 − 4b = 4. (In legatura cu X.77 din

RecMat - 1/2007.)Marian Tetiva, Barlad

Solutie. Putem proceda ca ın solutia problemei X.77, care poate fi consul-tata ın RecMat 1/2008, pp. 65. In cele ce urmeaza, vom prezenta o solutie carefoloseste scrierea trigonometrica a numerelor complexe. Presupunem ıntai ca ecuatiaare solutiile z1, z2 de modul 1, deci zk = cos tk + i sin tk, tk ∈ [0, 2π), k = 1, 2. Atunci

a = z1 + z2 = 2 cost1 − t2

2

cos

t1 + t22

+ i sint1 + t2

2

, b = z1z2 = cos(t1 + t2) +

i sin(t1 + t2), iar a2 − 4b = −4 sin2t1 − t2

2(cos(t1 + t2) + i sin(t1 + t2)). Evident ca

|b| = 1, iar |a|2 + |a2 − 4b| = 4

cos2

t1 − t22

+ sin2t1 − t2

2

= 4.

Reciproc, fie |b| = 1 si |a|2 + |a2 − 4b| = 4. Atunci |a2| + |4b − a2| = 4 = 4|b| =|a2 +4b− a2|, prin urmare exista t ≥ 0 astfel ıncat 4b− a2 = ta2 sau avem a2 = 0. Inal doilea caz, ecuatia devine z2 + b = 0 si, cum |b| = 1, ambele solutii vor fi de modul

1. In primul caz, discriminantul este ∆ = a2 − 4b = −ta2, de unde z1,2 = a · 1± i√t

2,

cu modulul |z1| = |z2| = |a| ·É

1 + t

4=

Ê (1 + t)a2

4

= È|b| = 1, ceea ce trebuia

demonstrat.

146

Page 65: Revista (format .pdf, 5.8 MB)

X.93. Daca a1, a2, . . . , an∈(0, 1) sau a1, a2, . . . , an∈(1,∞), iar f, g : 1, 2, . . . , n→ 1, 2, . . . , n sunt functii injective, sa se arate ca nX

k=1

logakaf(k)

ag(k)

nXk=1

ak

≥ n2.

Dan Popescu, SuceavaSolutie. Se ınmultesc membru cu membru urmatoarele doua inegalitati, care

rezulta din inegalitatea mediilor:nX

k=1

ak ≥ n · n

ÌnY

k=1

ak sinX

k=1

logakaf(k)

agk≥

n · n

ÏnQ

k=1

logakaf(k)

nQl=1

ag(k)

=n

n

ÊnQ

k=1

ag(k)

=n

n

ÊnQ

k=1

ak

(am tinut seama de faptul ca functiile

f si g sunt chiar bijective).

X.94. a) Sa se arate capx2n + y2n + xnyn+

√x2n + z2n + xnzn≥

py2n + z2n + ynzn,∀x, y, z∈R∗, ∀n∈N.

b) Demonstrati ca, daca n este par, inegalitatea este stricta, iar daca n este impar,atunci exista x, y, z ∈ R pentru care se atinge egalitatea.

Bogdan Victor Grigoriu, Falticeni

Solutie. a) Considerand ın plan punctele A(xn, 0), B

−yn

2,yn

√3

2

si

C

−zn

2,−zn

√3

2

, observam caAB =

px2n + y2n + xnyn, AC =

√x2n + z2n + xnzn,

iar BC =p

y2n + z2n + ynzn. Concluzia rezulta aplicand inegalitatea triunghiului.b) Daca n este par, atunci numerele xn, yn si zn sunt strict pozitive, prin urmare

B se va situa ın cadranul II, iar C ın cadranul III, ın timp ce A se va afla pe semiaxapozitiva Ox. In acest caz, punctele A,B si C nu pot fi coliniare, prin urmare ine-galitatea de la a) este stricta. Daca n este impar, egalitatea se atinge, de exemplu,pentru x = 1, y = z = − n

√2.

X.95. Consideram functia f : R3 → R,

f (x, y, z) = sinx+ sin y + sin z + sin (x− y) + sin (y − z) + sin (z − x) .

Determinati maximul si minimul functiei f .Catalin Calistru, Iasi

Solutie. Cu substitutiile x = x1 + x2, y = x2 + x3, z = x3 + x1, avem:

f(x, y, z) = sin(x1 + x2) + sin(x2 + x3) + sin(x3 + x1) + sin(x1 − x3)++ sin(x2 − x1) + sin(x3 − x2) = 2 sinx1 cosx3 + 2 sinx2 cosx1 + 2 sinx3 cosx2.

Prin urmare, 3+f(x, y, z) = (sinx1+cosx3)2+(sinx2+cosx1)

2+(sinx2+cosx2)2 ≥ 0

si 3 − f(x, y, z) = (sinx1 − cosx3)2 + (sinx2 − cosx1)

2 + (sinx3 − cosx2)2 ≥ 0, deci

147

Page 66: Revista (format .pdf, 5.8 MB)

−3 ≤ f(x, y, z) ≤ 3, ∀(x, y, z) ∈ R3. Cum f(π

2,π

2,π

2) = 3, iar f(−π

2,−π

2,−π

2) = −3,

rezulta ca max f = 3 si min f = −3.

Clasa a XI-aXI.91. Fie matricele A,B,C,D ∈ Mn (R) astfel ıncat AC + BD = In, iar

AD = BC. Demonstrati ca CA+DB = In si DA = CB.

I. V. Maftei, Bucuresti si Mihai Haivas, Iasi

Solutie. Din datele problemei obtinem ca (A−iB)(C+iD) = In; atunci matriceleA − iB si C + iD sunt una inversa celeilalte, prin urmare (C + iD)(A − iB) = In.Deducem ca (CA+DB)+ i(DA−CB) = In si, cum CA+DB, DA−CB ∈ Mn(R),atunci CA+DB = In si DA− CB = On.

XI.92. Determinati matricele X ∈ M2 (R) pentru care X2 +X =

1 11 1

.

Adrian Reisner, Paris

Solutie. Fie X =

a bc d

∈ M2(R); atunci X2 = (a + d)X − (ad − bc)I2 si

ecuatia devine X(a+ d+ 1)− (ad− bc)I2 =

1 11 1

. Obtinem sistemul:8><>: a(a+ d+ 1)− (ad− bc) = 1 (1)

b(a+ d+ 1) = 1 (2)c(a+ d+ 1) = 1 (3)d(a+ d+ 1)− (ad− bc) = 1 (4)

Din ecuatiile (1) si (4) rezulta ca (a + d + 1)(a − d) = 0 si cum a + d + 1 = 0,

atunci d = a. Din ecuatiile (2) si (3) obtinem ca b = c =1

a+ d+ 1=

1

2a+ 1.

Folosind acum ecuatia (1), avem ca a(2a + 1) − (a2 − 1

(2a+ 1)2) = 1, echivalent cu

a(a+ 1)(4a2 + 4a− 3) = 0, deci a ∈ −3

2,1

2,−1, 0. Gasim solutiile

X1 =

0 11 0

, X2 =

−1 −1−1 −1

, X3 =

1

2

1 11 1

, X4 = −1

2

3 11 3

.

XI.93. Studiati convergenta sirului (un)n≥1 definit prin u1 ≥ 0, un+1 =un + 1

u2n + 1

,

∀n ∈ N∗.

Gheorghe Costovici si Adrian Corduneanu, Iasi

Solutie. Vom arata ca (un) este convergent catre 1. Daca u1 ∈ 0, 1, afirmatiaeste imediata. Cazurile u1 ∈ (0, 1) si u1 ∈ (1,∞) tratandu-se asemanator, ne vomfixa atentia asupra primului. Evident ca un > 0, ∀n ∈ N si se arata usor prininductie ca u2n−1 < 1 si u2n > 1, ∀n ∈ N∗. Vom demonstra acum ca u2n+2 < u2n si

148

Page 67: Revista (format .pdf, 5.8 MB)

u2n+1 > u2n−1, ∀n ∈ N∗; efectuam calculele doar pentru prima afirmatie:

u2n+2 < u2n ⇔ u2n+1 + 1

u22n+1 + 1

< u2n ⇔u2n + 1

u22n + 1

+ 1

Áu2n + 1

u22n + 1

2

+ 1

< u2n ⇔ u4

2n + u32n + 3u2

2n + u2n + 2

u42n + 3u2

2n + 2u2n + 2< u2n ⇔ u5

2n − u42n + 2u2

2n−

−u22n + u2n − 2 > 0 ⇔ (u2n − 1)(u4

2n + 2u22n + u2n + 2) > 0,

adevarat. In concluzie, subsirul (u2n−1)n≥1 este crescator si marginit, iar subsirul(u2n)n≥1 este descrescator si marginit inferior de 1; ınseamna ca ambele sunt con-vergente spre α, respectiv β. Trecand la limita ın relatia de recurenta, obtinem ca

α =β + 1

β2 + 1, iar β =

α+ 1

α2 + 1. Inlocuind, gasim pentru α ecuatia α5 −α4 +2α3 −α2 +

α − 2 = 0, deci (α − 1)(α4 + 2α2 + α + 2) = 0 si, cum α > 0, ınseamna ca α = 1.Deducem apoi ca β = 1, prin urmare lim

n→∞un = 1.

Nota. In aceeasi maniera, autorii problemei au stabilit ca pentru orice a, b ∈(0,∞), sirul (un)n≥1 definit prin u1 ≥ 0, un+1 =

uan + b

ua+1n + b

este convergent catre 1.

XI.94. Sa se demonstreze ca pentru orice n ∈ N∗, exista numerele distincte

x1, x2, . . . , xn ∈ (1, 2), asa ıncat x1x2 · · ·xn =4e

n.

Dan Plaesu , Iasi

Solutia 1 (a autorului). Consideram functia f : [1, 2] → R, f(x) = x lnx − x.

Aplicand teorema lui Lagrange functiei f pe intervalele

1 +

k − 1

n, 1 +

k

n

, k =

1, 2, . . . , n, determinam xk ∈1 +

k − 1

n, 1 +

k

n

, k = 1, 2, . . . , n, astfel ıncat f

1 +

k

n

f

1 +

k − 1

n

=

1

nf ′(xk), k = 1, 2, . . . , n. Rezulta ca f ′(x1)+ f ′(x2)+ . . .+ f ′(xn) =

n ·nP

k=1

f

1 +

k

n

− f

1 +

k − 1

n

= f(2)− f(1). Folosind faptul ca f ′(x) = lnx

si ca f(2)− f(1) = ln4

e, din ultima relatie obtinem concluza problemei.

Solutia 2 (Gabriel Popa). Constructia sirului (xn)n≥1 se poate face inductiv:

luam x1 =4

e∈ (1, 2), iar daca presupunem existenta numerelor x1, x2, . . . , xn ∈ (1, 2)

pentru care x1x2 . . . xn =

4

e

n

, atunci numerele x1, . . . , xn−1, knxn si4

kne, unde

(kn)n≥1 este un sir de numere din (1, 2) cu limita 1, au produsul

4

e

n+1

si pot fi

facute distincte, alegand convenabil kn (posibil, ıntrucat orice vecinatate la dreaptaa lui 1 este nenumarabila).

Solutia 3 (Gheorghe Iurea). Cautam numerele xk, k = 1, n de forma xk =4

e

ak

cu ak ∈0,

3

2

, k = 1, n. Conditia problemei devine a1 + a2 + . . .+ an = n.

149

Page 68: Revista (format .pdf, 5.8 MB)

Pentru n = 2p putem alege numerele 1− 1

3, 1+

1

3, 1− 1

4, . . . , 1− 1

p+ 2, 1+

1

p+ 2, iar

pentru n = 2p+1 putem alege numerele 1, 1− 1

3, 1+

1

3, 1− 1

4, . . . , 1− 1

p+ 2, 1+

1

p+ 2.

XI.95. Calculati limn→∞

1 +

1

+1 +

1

12α

+ · · ·+1 +

1

1nα

− n

, unde

α ≥ 1 este fixat. (In legatura cu L83 din RecMat-1/2005.)Marius Olteanu, Rm. Valcea

Solutie. Notam cu (xn)n≥1 sirul a carui limita o cautam. Cum

1 +

1

1kα

>

1,∀k = 1, n, deducem ca xn > 0, ∀n ∈ N∗. Din inegalitatea lui Bernoulli, obtinem ca1 +

1

1kα

< 1 +1

nα· 1

kα, deci xn <

1

1 +

1

2α+ . . .+

1

, ∀n ∈ N∗. Din teo-

rema Cesaro-Stolz, limn→∞

1

1 +

1

2α+ . . .+

1

= lim

n→∞

1

(n+ 1)α· 1

(n+ 1)α − nα.

Daca vom dovedi ca aceasta din urma limita este 0, conform criteriului clestelui rezultaca lim

n→∞xn = 0.

Aplicand teorema lui Lagrange functiei f : [n, n+1] → (0,∞), f(x) = xα, deducem

ca (n + 1)α − nα = α · cα−1, cu c ∈ (n, n + 1). Atunci1

α· 1

(n+ 1)2α−1<

1

(n+ 1)α·

1

(n+ 1)α − nα<

1

α· 1

(n+ 1)α · nα−1, ceea ce, prin trecere la limita, conduce la faptul

ca limn→∞

1

(n+ 1)α· 1

(n+ 1)α − nα= 0.

Clasa a XII-a

XII.91. Prove that

Z 1

0(1 + x) ex(1+ex)dx = ee − 1.

Zdravko Starc, Vrsac, Serbia

Solutie. Facem substitutia xex = t; atunci (x+1)exdx = dt si

Z 1

0(1+x)ex(1+ex)dx =Z e

0etdt = et

e0= ee − 1.

XII.92. Fie b > a > 0, iar f : [a, b] → R o functie continua pe [a, b] si derivabila

pe (a, b); sa se arate ca exista c ∈ (a, b) astfel ıncat b

Z c

af (x) dx = c (b− c) f (c).

Dan Nedeianu, Drobeta Tr. Severin

Solutie. Aplicam teorema lui Rolle functiei g : [a, b] → R, g(x) =x− b

x

Z x

af(t)dt

si tinem cont ca g′(x) =b

x2

Z x

af(t)dt+

x− b

xf(x).

XII.93. Demonstrati ca exista c ∈ (2, π) pentru care

Z π2

1

sinx

xdx ≤ 2 cos 1

c.

Constantin Micu, Melinesti (Dolj)

150

Page 69: Revista (format .pdf, 5.8 MB)

Solutie. Se constata usor ca functia f :h1,

π

2

i→ R, f(x) = sinx este strict

crescatoare, iar g :h1,

π

2

i→ R, g(x) =

1

xeste strict descrescatoare; atunci, conform

inegalitatii lui Cebısev,

1π2 − 1

·Z π

2

1

sinx

x≤ 1

π2 − 1

2 ·Z π

2

1sinxdx ·

Z π2

1

1

xdx,

de unde

Z π2

1

sinx

xdx ≤ 2 cos 1 · lnπ − ln 2

π − 2. Aplicand teorema lui Lagrange functiei

h : [2, π] → R, h(x) = lnx, gasim c ∈ (2, π) pentru care1

c=

lnπ − ln 2

π − 2si astfel

solutia problemei este completa.

XII.94. Calculati limn→∞

n

Z 2n

n

xa + b√x2a+4 + 1

dx, unde a ∈ (0,∞) si b ∈ R.

Liviu Smarandache, Craiova

Solutie. Fie f : [0,∞) → R, f(x) =xa + b√x2a+4 + 1

si F (x) =

Z x

0

ta + b√t2a+4 + 1

dt,

x ≥ 0. Cum 0 ≤ F (x) ≤Z x

0

ta + b

ta+2dt =

1

x− 1

2x+

b

a+ 1

1

xa+1− 1

(2x)a+1

, obtinem

ca limx→∞

F (x) = 0. Atunci

limx→∞

x

Z 2x

x

ta + b√t2a+4 + 1

dt = limx→∞

x(F (2x)− F (x)) = limx→∞

F (2x)− F (x)1x

=

= limx→∞

2f(2x)− f(x)

− 1x2

= limx→∞

(x2f(x)− 1

2(2x)2f(2x)) = 1− 1

2=

1

2,

deoarece limx→∞

x2f(x) = limx→∞

xa+2 + bx2

√x2a+4 + 1

= 1. Prin urmare, limita ceruta este1

2.

XII.95. Fie (A,+, ·) un inel ın care 0 = 1 si 1 + 1 + 1 + 1 + 1 = 0. Sa se arateca, daca x3y2 = y2x3, ∀x, y ∈ A, atunci inelul este comutativ.

I.V. Maftei, Bucuresti si Mihai Haivas, IasiSolutie. Fie x, y ∈ A, arbitrare. Din x3y2 = y2x3 si x3(y + 1)2 = (y + 1)2x3,

gasim ca x3(y + y) = (y + y)x3. Cum 1 + 1 + 1 + 1 + 1 = 0, rezulta ca 5y = 0, decix3(5y) = (5y)x3. Deducem ca x3(3y) = (3y)x3, prin urmare x3y = yx3. Folosindaceasta relatie pentru x si x + 1, obtinem x3y = yx3 si (x + 1)3y = y(x + 1)3, deunde (3x2 + 3x + 1)y = y(3x2 + 3x + 1). Inlocuind pe x cu x + 1 si tinand cont ca3(x+1)2+3(x+1)+1−(3x2+3x+1) = 6x+6 = x+1, rezulta ca (x+1)y = y(x+1),deci xy = yx. Cum x, y sunt arbitrare din A, concluzia problemei se impune.

151

Page 70: Revista (format .pdf, 5.8 MB)

Solutiile problemelor pentru pregatireaconcursurilor propuse ın nr. 2/2008

A. Nivel gimnazialG146. Fie x, y, z ∈ (0,∞) astfel ıncat xyz = 1. Aratati ca

xy3

x4 + y + z+

yz3

y4 + z + x+

zx3

z4 + x+ y≥ 1.

Liviu Smarandache si Lucian Tutescu, CraiovaSolutie. Avem ca x3 + y3 ≥ xy(x+ y), cu egalitate pentru x = y. AtunciX xy3

x4 + y + z=X xy3

x4 + xyz(y + z)=X y3

x3 + yz(y + z)≥X x3

x3 + y3 + z3= 1.

Egalitatea se atinge pentru x = y = z = 1.

G147. Fie n ∈ N, n ≥ 2, fixat, iar a, b, c sunt numere naturale astfel ıncat

na+ (n+ 1) b+ 2nc = n2 + 1. Aratati ca n−hn− 1

2

i≤ a+ b+ c ≤ n.

Gheorghe Iurea, Iasi

Solutie. Cum b − 1 = n(n − a − b − 2c), ınseamna ca b − 1...n. Insa b < n, altfel

na+(n+1)b+2nc > n2+1 si atunci b−1 = 0, deci b = 1. Conditia din enunt devinena + n + 1 + 2nc = n2 + 1 ⇔ a + 2c = n − 1 ⇔ a + b + c = n − c. Suma a + b + ceste maxima cand c este minim, adica pentru c = 0; deducem ca (a+ b+ c)max = n,maxim atins cand a = n − 1, b = 1, c = 0. Suma a + b + c este minima cand c este

maxim, deci pentru c =

n− 1

2

; obtinem ca (a + b + c)min = n −

n− 1

2

, minim

atins cand a = n− 1− 2

n− 1

2

, b = 1, c =

n− 1

2

.

G148. Fie a1a2 . . . ap ∈ N. Sa se arate ca orice numar natural are un multiplude forma a1a2 . . . apa1a2 . . . ap . . . a1a2 . . . ap0 . . . 0.

Marian Pantiruc, IasiSolutie. Fie n ∈ N; consideram numerele: a1a2 . . . ap, a1a2 . . . apa1a2 . . . ap, . . . ,

a1a2 . . . apa1a2 . . . ap . . . a1a2 . . . ap, ın numar de (n + 1). Prin ımpartirea acestora lan obtinem (n + 1) resturi si, cum resturile posibile sunt 0, 1, . . . , n − 1, rezulta cacel putin doua resturi sunt egale. Fie a = a1a2 . . . ap . . . a1a2 . . . ap (format din k1numere a1a2 . . . ap si b = a1a2 . . . ap . . . a1a2 . . . ap (format din k2 numere a1a2 . . . ap)doua dintre numerele de mai sus, care dau acelasi rest la ımpartirea cu n. Diferentaacestora se divide cu n, deci a−b = a1a2 . . . apa1a2 . . . ap . . . a1a2 . . . ap 00 . . . 0 verificacerinta problemei.

G149. a) Determinati doua numere prime p, q astfel ıncat p < q, iar p2 − 1 aremai multi divizori naturali decat q2 − 1.

b) Determinati toate numerele prime p pentru care p2 − 1 are exact opt divizorinaturali.

Dan Popescu, Suceava

152

Page 71: Revista (format .pdf, 5.8 MB)

Solutie. a) De exemplu, putem lua p = 19, q = 23.b) Daca p = 2, atunci p2 ≡ 1 (mod 8). Daca p = 3, atunci p2 ≡ 1 (mod 3).

Cum (3, 8) = 1, ınseamna ca daca p /∈ 2, 3, atunci p2 ≡ 1 (mod 24), prin urmare

p2 − 1...24. Pentru p ≥ 7, avem, ca p2 − 1 > 24 si, cum 24 are opt divizori naturali,

p2−1 va avea mai mult de opt divizori. Pentru p ∈ 2, 3, p2−1 are mai putin de optdivizori. Daca p = 5, atunci p2 − 1 = 24 are exact opt divizori, deci singurul numarcare satisface conditiile din enunt este 5.

G150. Fie m si n numere naturale nenule cu proprietatea ca m ≤ 1+2+ · · ·+n.Sa se arate ca m poate fi scris ca suma catorva numere distincte dintre 1, 2,. . . , n.

Marian Tetiva, Barlad

Solutia 1 (Dan Mocanu, elev, Iasi). Acoperim multimea

§1, 2, 3, . . . ,

n(n+ 1)

2

ªcu urmatoarele sume avand termenii distincti: 1, 2, 3, . . . , n, n+1, n+2, . . . , n+(n−1),n+ (n− 1) + 1, n+ (n− 1) + 2, . . . , n+ (n− 1) + (n− 2), . . . , n+ (n− 1) + . . .+ 1;concluzia problemei este acum imediata.

Solutia 2 (a autorului). Demonstram prin inductie dupa n. Pentru n = 1, decim = 1, nu avem nimic de demonstrat; de asemenea se verifica usor cazul n = 2 (decim = 1, 2 sau 3). Vom presupune mai departe n ≥ 2 si ca afirmatia este adevaratapentru n− 1 si o demonstram pentru n.

Daca m este unul dintre numerele 1, 2, . . . , n nu avem ce arata. Daca m ≥ n+ 1,avem ca 1 ≤ m− n ≤ 1 + 2 + . . . (n− 1) si, conform ipotezei de inductie, m− n estesuma unor termeni distincti din multimea 1, 2, . . . , (n− 1). Rezulta ca m este sumadintre acesti termeni si n, ceea ce ıncheie demonstratia.

G151. Bazele unei prisme sunt poligoane cu 2008 varfuri. Numerotam cu 1,2,. . . , 2008 varfurile bazei inferioare si, corespunzator, cu a1, a2,. . . , a2008 varfurilebazei superioare, unde a1, a2, . . . , a2008 = 1, 2, . . . , 2008.

a) Demonstrati ca putem gasi o numerotare pentru baza superioara astfel ıncat

i+ ai... 8, ∀i ∈ 1, 2, . . . , 2008.

b) Demonstrati ca nu putem gasi o numerotare pentru baza superioara astfel ıncat

i+ ai... 9, ∀i ∈ 1, 2, . . . , 2008.

Gabriel Popa si Gheorghe Iurea, IasiSolutie. a) De exemplu, putem lua a1 = 2007, a2 = 2006, . . . ,a2007 = 1, a2008 =

2008.

b) Daca ar exsta o numerotare pentru care i + ai...9, ∀i ∈ 1, 2, . . . , 2008, atunci

(1 + a1) + (2 + a2) + . . .+ (2008 + a2008)...9, prin urmare 2(1 + 2 + . . .+ 2008)

...9. Am

obtine astfel ca 2008 · 2009...9, contradictie.

G152. In triunghiul isoscel ABC (AB = AC) notam cu B′, C ′ picioarele ınalti-milor din B, respectiv C. Daca AB = 2B′C ′, sa se determine unghiurile triunghiului.

Nela Ciceu, Bacau si Titu Zvonaru, ComanestiSolutie. Vom analiza doua cazuri, dupa cum unghiul bA este ascutit sau obtuz.a) Fie M mijlocul laturii [AB]; atunci [B′M ] va fi mediana ın ABB′

153

Page 72: Revista (format .pdf, 5.8 MB)

A

B C

M

C B

dreptunghic si deducem ca B′M =1

2AB. Avem si ca B′C ′ =

1

2AB,

prin urmare B′C ′ = B′M , deci ÖB′MB ≡ ÖB′C ′A. Insa ÖB′C ′A ≡ÒB (deoarece B′C ′∥BC) si atunci m(ÒB) = m(×B′MC ′) = 180 −2m(ÖMBB′) = 180−2[90−m( bA)] = 2m( bA), relatie care ımpreuna

cu m( bA)+2m(ÒB) = 180 conduce la m( bA) = 36, m(ÒB) = m(ÒC) =72.

A

B C

M

B C

b) Daca bA este obtuz, atunci B′MA va fi isoscel sicu un rationament asemanator celui de mai sus obtinem

ca m(ÒB) = m(×B′C ′M) = 180 − 2m(ÖB′AB) = 180 − 2 ·2m(ÒB) = 180 − 4m(ÒB), prin urmare m(ÒB) = m(ÒC) = 36,

iar m( bA) = 108.

G153. In triunghiul ABC, M este mijlocul laturii [BC], m(ÕABC) = 30 si

m(ÕACB) = 105. Perpendiculara din C pe AM taie AB ın Q. Calculati valoarea

raportuluiQA

QB.

Neculai Roman, Mircesti (Iasi)Solutie. Ducem CE⊥AB, E ∈ (AB). Folosind triunghiul CEM echilateral

sitriunghiul AEC dreptunghic isoscel, gasim ca ME = EC = EA, deci triunghiul

AEM este isoscel cum(AEM) = 150. Atuncim(ÖEAM) = 15, decim(ÖMAC) = 30

si m(ÖAMC) = 45. Prin urmare, m(ÕBCQ) = 45 si m(ÕACQ) = 60. Obtinem caQA

QB=

AACQ

ABCQ=

BC · CQ sinBCQ

AC · CQ sinÕACQ=

AC sin 60

BC sin 45.

Dar AC = EC√2 =

BC√2

2si atunci

QA

QB=

√3

2.

G154. Fie D mijlocul laturii [BC] a triunghiului echilateral ABC de latura 1,iar P un punct mobil pe [CD]. Notam cu M si N proiectiile pe AP ale punctelor B,respectiv C. Aflati aria locului geometric descris de segmentul [MN ].

Marius Olteanu, Rm. Valcea

A

B CD

EF

M

N

P

C1

C2

Solutie. Observand ca patrulaterele ABDMsi ACND sunt inscriptibile, deducem ca puncteleM si N se afla pe cercurile C1 (circumscris tri-unghiului ABD), respectiv C2 (circumscris tri-unghiului ACD). Daca E este mijlocul laturii

[AC], se constata ca M parcurge arcul mic ÷DE al

cercului C1, ın timp ce N parcurge arcul mic ÷CDal cercului C2. Astfel, locul geometric maturat de[MN ] este suprafata hasurata ın figura. Observamca segmentele de disc marginite de C1 si [DE], re-spectiv de C2 si [CD], sunt congruente; atunci aria locului geometric va fi egala cu

aria triunghiului echilateral CDE, adica

√3

16.

154

Page 73: Revista (format .pdf, 5.8 MB)

G155. Fie C cercul circumscris ABC ascutitunghic. Notam cu P punctul deintersectie al tangentelor duse la cerc ın B si C, D = AP ∩ C, iar M si N sunt

mijloacele arcului mic

BC, respectiv arcului mare

BC. Sa se arate ca dreptele AM ,DN si BC sunt concurente.

Gabriel Popa, Iasi

Solutie. Fie T = AM ∩BC. Cum AM este bisectoarea unghiului ÕBAC, rezultaca

A

B C

D M

P

N

T

C

.

.

.

(1)BT

TC=

AB

AC.

Vom arata caDT este bisectoare pentru ÕBDC; atunciD,T,N vor fi coliniare si de aici concluzia problemei.

Cum ÕBAP ≡ ÕPBD, rezulta ca PAB ∼ PBD,

deciPB

PD=

AB

BD. Analog se arata ca PAC ∼ PCD,

de undePC

PD=

AC

CD. Insa PB = PC, prin urmare

AB

BD=

AC

CD⇔ AB

AC=

BD

CD. Tinand cont de (1), obtinem ca

BT

TC=

BD

DC, adica DT este bisectoarea lui ÕBDC, ceea ce

ıncheie rezolvarea.

B. Nivel liceal

L146. In plan se considera dreptele d1, d2,. . . , dn+1, oricare doua neparalele.

Notam cu αk = m(Ødk, dk+1), αk ≤ 90, k = 1, n. Pe d1 se considera un seg-ment de lungime 2 care se proiecteaza pe d2, apoi segmentul obtinut se proiecteazape d3 si tot asa, pana cand pe dn+1 se obtine un segment de lungime 1. Stiind ca

tgmin

¦αi | i = 1, n

©=È

n√4− 1, determinati unghiurile αk, k = 1, n.

Cristian Savescu, student, Bucuresti

Solutie. Fie A1B1 segmentul de lungime 2 de pe d1, iar Ak = prdkAk−1,

Bk = prdkBk−1, k = 2, n+ 1. Cum Ak+1Bk+1 = AkBk · cosαk, atunci An+1Bn+1 =

A1B1 cosα1 cosα2 . . . cosαn, de unde cosα1 cosα2 . . . cosαn =1

2. Fie αp = minαi|i =

1, n; cum cosinusul este descrescator pe0,

π

2

i, avem cosαp ≥ cosαk, ∀k = 1, n si

astfel cosn αp ≥ 1

2, deci cosαp ≥ 1

n√2, apoi sinαp =

È1− cos2 αp ≤

Èn√4− 1

n√2

, prin

urmare tgαp ≤È

n√4− 1. Conform ipotezei, rezulta ca se atinge egalitatea; acest

lucru are loc pentru cosα1 = cosα2 . . . = cosαn, deci cand α1 = α2 = . . . = αn =

arccos1n√2.

L147. Se considera un poligon convex cu n laturi, n ≥ 4, avand proprietatea caoricare doua diagonale nu sunt paralele si oricare trei nu sunt concurente ın punctediferite de varfurile poligonului. Se noteaza cu ni numarul punctelor de intersectie a

155

Page 74: Revista (format .pdf, 5.8 MB)

diagonalelor interioare poligonului si cu ne cel al punctelor de intersectie exterioarepoligonului.

a) Sa se arate ca exista exact opt poligoane care verifica relatia ni > ne.b) Sa se arate ca exista exact trei poligoane pentru care ni + ne = kn2, k ∈ N∗.

Mihai Haivas, IasiSolutie. Fiecare punct interior de intersectie a diagonalelor este unic determinat

de cele doua diagonale ce-l contin, deci de patru varfuri ale poligonului; rezulta ca

ni = C4n. Cum sunt

n(n− 3)

2diagonale, care se intersecteaza ın C2

n(n−3)2

puncte,

fiecare varf al poligonului fiind numarat de C2n−3 ori (se obtine ca intersectie a oricare

doua diagonale care trec prin acel varf), obtinem ca numarul total de intersectii, fara

varfuri, este ni + ne = C2n(n−3)

2

− nC2n−3 =

n(n− 3)(n2 − 7n+ 14)

8. Prin urmare,

nl =n(n− 3)(n− 4)(n− 5)

12.

a) Din conditia ni > ne rezulta ca n3−15n+38 < 0, cu solutiile n ∈ 4, 5, 6, . . . , 11.

b) Conditia ni + ne = kn2 este echivalenta cu n3 − 10n2 + (35 − 8k)n − 42 = 0,deci n ∈ 7, 14, 12, carora le corespund valorile k ∈ 1, 11, 33.

Nota. Intr-o maniera asemanatoare a rezolvat problema dl. Daniel Vacaru,Pitesti.

L148. Pe latura (AB) a triunghiului ABC consideram punctul D astfel ıncatAB = 4AD. De aceeasi parte a laturii AB ca si punctul C, luam un punct P

astfel ıncat ÕPDA ≡ ÕACB si PB = 2PD. Demonstrati ca patrulaterul ABCP esteinscriptibil.

Nela Ciceu, Bacau si Titu Zvonaru, ComanestiSolutia 1 (a autorilor). Consideram punctul Q astfel ıncat AB separa P

A

B C

Q

P

D

si Q, AB = 2AQ si ÕQAB ≡ ÕBPD. AtunciAQ

AB=

PD

PB

=

1

2

si obtinem ca AQB ∼ PDB, de undeÕAQB = ÕPDB. Rezulta ca m(ÕAQB) + m(ÕACB) =

m(ÕPDB)+m(ÕPDA) = 180, ceea ce asgura inscriptibil-itatea patrulaterului AQBC. Din AB = 2AQ si AB =

4AD deducem ca AQ2 = AD · AB, astfel scrisAQ

AB=

AD

AQ, prin urmare AQD ∼

ABQ. Obtinem ca ÕADQ ≡ ÕAQB si cum avem si ÕAQB ≡ ÕPDB, rezulta ca punctele

Q,D si P sunt coliniare. Ne amintim ca ÕQAB ≡ ÕQPB, deci patrulaterul AQBPeste inscriptibil. Inscriptibilitatea patrulaterelor AQBC si AQBP arata ca puncteleA,Q,B,C, P sunt conciclice, de unde concluzia problemei.

Solutia 2 (Daniel Vacaru, Pitesti). Folosind relatia lui Stewart ın PAB,

obtinem ca PA2 ·BD

AB−PD2+PB2 ·AD

AB= AD ·DB, deci PA2 · 3

4−PD2+PB2 · 1

4=

3

16AB2 si, cum PB = 2PD, deducem ca PA2 · 3

4=

3

16AB2, adica PA =

1

2AB.

Aplicam acum teorema cosinusului ın ADP : PA2 = AD2+DP 2−2AD ·DP ·cosC,

156

Page 75: Revista (format .pdf, 5.8 MB)

de unde DP 2− 1

2AB ·DP · cosC− 3

16AB2 = 0, relatie pe care o vom privi ca ecuatie

de gradul II ın necunoscuta DP. Singura solutie pozitiva a acestei ecuatii este DP =1

4· AB · (cosC +

√3 + cos2 C). Cum cosÕAPB =

PA2 + PB2 −AB2

2PA · PB, ınlocuind PA

si PD cu valorile gasite anterior si facand calculele, obtinem cosÕAPB = cos(ÕACB),

prin urmare ÕAPB ≡ ÕACB, ceea ce arata ca patrulaterul ABCP este inscriptibil.

L149. Sa se determine pozitia punctului P pe directoarea parabolei P, astfel ıncataria triunghiului PT1T2 sa fie minima, unde T1 si T2 sunt punctele de contact cuparabola ale tangentelor duse din P la P.

Adrian Corduneanu, IasiSolutia 1 (a autorului). Raportam parabola la un reper canonic, avand originea

ın varful O al parabolei si drept axa a absciselor, perpendiculara dusa din O pedirectoare. Ecuatia parabolei este P : y2 = 2px(p > 0), iar cea a directoarei d : x =

−p

2. Se stie (sau se verifica!) faptul ca tangentele PT1 si PT2, duse la parabola din

punctul P ∈ d, sunt ortogonale; prin urmare, SPT1T2 =1

2PT1 · PT2.

Fie T1(x0, y0), cu y20 = 2px0, y0 > 0. Ecuatia tangentei PT1 este yy0 = p(x+x0) si

cum xp = −p

2, obtinem yp = − p2

2y0+y02, deci PT 2

1 =x0 +

p

2

2+

y0 +

p2

2y0− y20

2

2

=

1

4p2y20(y20 + p2)3. Tangenta PT2 trece prin P si are panta −y0

p; ecuatia sa va fi y =

−y0px − p2

2y0. Intersectand aceasta tangenta cu parabola, gasim coordonatele punc-

tului T2, anume

p3

2y20,−p2

y0

. Astfel, PT 2

2 =

p3

2y20+

p

2

2

+

−p2

y0+

p2

2y0− y0

2

2

=

1

4y40(y20 + p2)3.

Patratul ariei triunghiului PT1T2 se poate scrie acum ın functie de parametrul y0:

S(y0) =

1

2· PT1 · PT2

2

=1

8p· (y

20 + p2)3

y30. Derivata acestei functii este S′(y0) =

3

8py40(y20 + p2)2(y20 − p2) si se anuleaza doar ın y0 = p (deoarece y0 a fost considerat

pozitiv). Obtinem ca S are un minim egal cu Smin = p2, care se atinge cand y0 = p,

deci pentru punctul P−p

2, 0.

Solutia 2 (Gheorghe Costovici, Iasi). Vom rezolva problema ıntr-un caz cevamai general, cand P nu apartine numaidecat directoarei parabolei, ci unei drepteperpendiculare pe axa de simetrie a acesteia si care nu are niciun punct comun cuparabola. Fie y2 = ax, a > 0, ecuatia parabolei, iar Ti(xi, yi), cu y2i = axi, x1 = x2,

xi = 0. Tangenta ın Ti la parabola are ecuatia1

2(y+yi) = axxi, i = 1, 2. Intersectand

cele doua tangente, obtinem coordonatele punctului P , anume Px1 + x2

2, ax1x2

.

Fie y = k, k < 0, ecuatia dreptei pe care se afla P ; atunci k = ax1x2 < 0, deci

157

Page 76: Revista (format .pdf, 5.8 MB)

putem alege x1 > 0, x2 < 0. Aria triunghiului PT1T2 va fi

SPT1T2 =1

2

1x1 + x2

2k

1 x1 y11 x2 y2

= 1

2

k

ax1− x1

k − a

2

x21 +

k2

a2x21

.

Gandim aceasta arie ca o functie ın nedeterminata x1, derivata acestei functii este3

4· a

x41

·x21 +

k

a

x21 −

k

a

2

, iar punctele critice ale functiei sunt x′1 =

É−k

asi

x′′1 = −

É−k

a. Studiind semnul derivatei, se observa ca x′

1 este punct de minim

pentru arie si obtinem ca xp = 0. In concluzie, punctul P cautat este intersectiadreptei date cu Ox.

L150. Fie tetraedrul A1A2A3A4, iar P un punct ın interiorul sau. Notamcu Aij ∈ (AiAj) proiectiile ortogonale ale lui P pe muchiile AiAj ale tetraedrului.Demonstrati ca

VPA12A13A23 + VPA12A14A24 + VPA13A14A34 + VPA23A24A34 ≤ 1

4VA1A2A3A4 .

Cand se atinge egalitatea?Marius Olteanu, Rm. Valcea

Solutie. Fie P1 = Pr(A2A3A4)A1; din reciproca teoremei celor trei perpendiculare,obtinem ca P1A23 ⊥ A2A3, P1A24 ⊥ A2A4, P1A34 ⊥ A3A4, prin urmare A23A24A34

este triunghiul podar al punctului P1 ın raport cu A2A3A4. Aria s1 a acestuitriunghi este cel mult un sfert din aria S1 a A2A3A4. Daca mai notam h1 = A1P1,

x1 = PP1, atunciVPA23A24A34

VA1A2A3A4

=x1s1h1S1

≤ 1

4· x1

h1. Introducem analog x2, x3, x4

si h2, h3, h4; putem scrie ınca trei inegalitati analoage celei precedente. Concluziaproblemei se obtine adunand cele patru inegalitati si tinand seama de relatia lui

Gergonne

x1

h1+

x2

h2+

x3

h3+

x4

h4= 1

.

Deoarece 4si = Si doar atunci cand Pi coincide cu centrul cercului circumscrisfetei care se opune varfului Ai, rezulta ca egalitatea se atinge cand P este centrulsferei circumscrise tetraedrului.

L151. Sa se demonstreze ca nu exista numere naturale n si k astfel ıncath2 +

√32n+1

i=h4 +

√15ki

.

Cosmin Manea si Dragos Petrica, PitestiSolutie. Avem ca (2 +

√3)2n+1 = 22n+1 + C1

2n+122n√3 + C2

2n+122n−1 · 3 +

. . . + C2n+12n+1 (

√3)2n+1 = an + bn

√3, cu an, bn ∈ N, iar (2 −

√3)2n+1 = an − bn

√3.

Astfel, [(2 +√3)2n+1] = [(2 +

√3)2n+1 + (2 −

√3)2n+1 − 1 + 1 − (2 −

√3)2n+1] =

[2an − 1 + 1 − (2 −√3)2n+1] = 2an − 1, deoarece 1 − (2 −

√3)2n+1 ∈ (0, 1). La fel

stabilim ca (4 +√15)k = ck + dk

√15, (4 +

√15)k = ck − dk

√15, ck, dk ∈ N, iar

[(4 +√15)k] = 2ck − 1. Daca presupunem ca exista n si k cu proprietatile cerute,

158

Page 77: Revista (format .pdf, 5.8 MB)

avem an = ck, deci 22n+1 +M3 = 4k +M15; rezulta ca 3|2 · 4n − 4k. Dar 2 · 4n − 4k =

2(M3 + 1)− (M3 + 1) = M3 + 1 si obtinem contradictia 3|1, prin urmare nu exista nsi k cu proprietatea ceruta.

L152. Pentru a, b, c ∈ R si x ∈ R+, demonstrati inegalitatea

9

a2+ b2+ c2≤ 3 (x+ 1)

2(a+ b+ c)

43(x2+ 1)(a2+ b2+ c2) + 2x(a+ b+ c)

2(ab+ bc+ ca)

2≤ 1

a2+

1

b2+

1

c2.

I. V. Maftei si Dorel Baitan, BucurestiSolutie. Utilizand sumarea ciclica, inegalitatea din stanga se scrie 3[3(x2 +

1)(X

a2) + 2x(X

a)2] · (X

ab)2 ≤ (x2 + 1)(X

a)4(X

a2) si aceasta se obtine

adunand inegalitatile

(1) 9(P

ab)2 ≤ (P

a)4; 3(P

ab)2 ≤ (P

a)2(P

a2),

prima multiplicata prin (x2 + 1)(X

a2), iar a doua prin 2x(X

a)2.

Inegalitatea din dreapta se scrie sub forma

(x+ 1)2a2b2c2(X

a)4 ≤ (X

a2b2)(X

ab)2[(x2 + 1)(X

a2) +2x

3(X

a)2]

si se poate obtine adunand inegalitatile

(2) a2b2c2(P

a)4 ≤ (P

a2b2)(P

ab)2(P

a2); a2b2c2(P

a)2 ≤ 1

3(P

a2b2)(P

ab)2,

prima multiplicata prin (x2 + 1), iar a doua prin 2x(X

a)2.

Justificarea inegalitatilor (1) si (2) revine la demonstrarea inegalitatilor 9(X

ab)2 ≤(X

a)4; (X

a)2 ≤ 3(X

a2); 3abc(X

a) ≤ (X

ab)2; abc(X

a) ≤X

a2b2, care sunt

relativ uzuale.

L153. Gasiti toate functiile f : R → R cu proprietatea ca

fx2 + xy + yf (y)

= xf (x+ y) + f2 (y) , ∀x, y ∈ R.

Adrian Zahariuc, student, PrincetonSolutie. Cautam un z = x astfel ıncat x2+xy = z2+zy; gasim z = −x−y. Atunci

xf(x+ y) + f2(y) = f(x2 + xy + yf(y)) = f(z2 + zy + yf(y)) = zf(z + y) + f2(y) =(−x−y)f(−x)+f2(y). Rezulta ca xf(x+y) = −(x+y)f(−x), pentru orice x, y ∈ R.

Deducem caf(x)

x(x = 0) este constanta, deci f(x) = cx, c ∈ R. Cum pentru x = 0

si y = 0 obtinem f(0) = 0, rezulta ca f(x) = cx, c ∈ R, pentru orice x ∈ R.Se verifica usor ca toate aceste functii au proprietatea dorita.

Nota. O solutie corecta, dar ceva mai laborioasa, a fost primita din partea d-luiDaniel Vacaru, Pitesti.

L154. Fie P ∈ R [X] un polinom de gradul n si p : R → R functia polino-miala asociata. Stiind ca multimea x ∈ R | p (x) = 0 are k elemente (distincte), iar

159

Page 78: Revista (format .pdf, 5.8 MB)

functia f : R → R, f (x) = |p (x)| este derivabila pe R, aratati ca numarul maxim de

radacini complexe nereale ale lui P este egal cu 2hn2

i− 2k.

Vlad Emanuel, student, BucurestiSolutie. Studiem derivabilitatea functiei f . Daca x0 ∈ R si p(x0) = 0, atunci p

are semn constant ıntr-o vecinatate a lui x0. Pe aceasta vecinatate f(x) = p(x) (sauf(x) = −p(x). Rezulta ca f este derivabila ın x0. Daca x0 ∈ R si P (x0) = 0, atunci

f ′s(x0) = lim

x→x0x<x0

|P (x)| − |P (x0)|x− x0

= − limx→x0x<x0

P (x)− P (x0)

x− x0

= −|P ′s(x0)| = −|P ′(x0)| si

f ′d(x0) = lim

x→x0x>x0

|P (x)| − |P (x0)|x− x0

= limx→x0x>x0

P (x)− P (x0)

x− x0

= |P ′d(x0)| = |P ′(x0)|.

In concluzie, f este derivabila ın x0 daca si numai daca −|P ′(x0)| = |P ′(x0)| ⇔P ′(x0) = 0.

Prin urmare, f este derivabila pe R daca si numai daca orice radacina a lui P estecel putin dubla. Cum P are k radacini reale distincte, rezulta ca P are cel putin 2kradacini reale.

Pentru n = 2p, deducem ca P are cel mult 2p− 2k radacini complexe nereale, iarpentru n = 2p + 1, P are cel mult 2p + 1 − (2k + 1) radacini complexe nereale (am

folosit faptul ca radacinile complexe sunt perechi). In final avem cel mult 2hn2

i− 2k

radacini complexe nereale.Cum pentru n = 2p, P = (X − 1)2 . . . (X − k)2(X2 + 1)p−k verifica ipotezele

problemei si are 2p − 2k radacini complexe nereale, iar pentru n = 2p + 1, P =(X − 1)3(X − 2)2 . . . (X − k)2(X2 + 1)p−k verifica ipotezele si are 2p − 2k radacini

nereale, conclucionam ca numarul cautat este egal cu 2hn2

i− 2k.

L155. Fie A,B ∈ M2 (C) doua matrice astfel ıncat matricea AB − BA sa fie

inversabila. Sa se arate ca urma matricei (I2 +AB) (AB −BA)−1

este egala cu 1.Florina Carlan si Marian Tetiva, Barlad

Solutie. Consideram plinomul f(x) = det[I2+AB+x(BA−AB)], care are graduldoi (coeficientul lui x2 este egal cu det(BA−AB) = det(AB −BA) = 0). Observam

ca f(0) = f(1), ceea ce arata ca punctul de extrem al functiei f este x =1

2.

Pe de alta parte, f(x) = det(BA − AB)det[(I2 + AB)(BA − AB)−1 + xI2] =det(BA − AB)(x2 + xtr(P ) + detP ), unde P = (I2 + AB)(BA − AB)−1, ceea ce

ınseamna ca punctul de extrem este −1

2tr(P ) = −1

2tr[(I2+AB)(BA−AB)−1]. Prin

urmare, −12 tr[(I2+AB)(BA−AB)−1] =

1

2, de unde tr[(I2+AB)(AB−BA)−1] = 1.

Nota. Solutie corecta a dat dl. Daniel Vacaru, Pitesti.

160

Page 79: Revista (format .pdf, 5.8 MB)

Probleme propuse1

Clasele primare

P.174. Mirela are un mar, o para si o portocala. Mama ıi spune sa aseze fructelepe doua farfurii astfel ıncat pe fiecare farfurie sa fie cel mult doua fructe. In catemoduri poate aseza Mirela cele trei fructe?(Clasa I ) Inst. Maria Racu, Iasi

a a+1 a+2 a+3

P.175. Scrieti toate numerele mai mici ca 27 care se pot des-compune sub forma indicata alaturat.(Clasa I ) Diana Tanasoaie, eleva, Iasi

P.176. Intr-o bomboniera sunt cinci bomboane cu fructe si sapte bomboane cuciocolata. Care este cel mai mic numar de bomboane pe care trebuie sa-l luam dinbomboniera, fara sa ne uitam, pentru a avea cel putin doua bomboane cu ciocolata?(Clasa a II-a) Alexandru Dumitru Chiriac, elev, Iasi

P.177. Cum masuram 1litru de apa folosind doua vase negradate, unul de 5litru,iar celalalt de 8litri?(Clasa a II-a) Mariana Nastasia, eleva, Iasi

P.178. Aratati ca, daca restul este o cincime din scazator, atunci descazutul seımparte exact la 6. Care este cel mai apropiat descazut de numarul 100 cu aceastaproprietate?(Clasa a III-a) Mirela Cucoranu, eleva, Iasi

P.179. Se dau produsele: a× b = 60, a× c = 70, a× d = 95. Stiind ca b+ c+ deste de 9 ori mai mare decat a, sa se afle valoarea lui a.(Clasa a III-a) Andreea Amarandei, eleva, Iasi

P.180. Aratati ca din sirul 7, 28, 31, 46, 61, 100 nu putem extrage patru numere acaror suma sa se ımparta exact la trei.(Clasa a III-a) Dragos Iacob, elev, Iasi

P.181. Un triunghi si un patrat au acelasi perimetru, exprimat printr-un numarnatural. Care este cea mai mica valoare a perimetrului? Cate valori posibile aleperimetrului sunt cuprinse ıntre 100 si 200?(Clasa a IV-a) Andreea Alexa, eleva, Iasi

P.182. Aflati cea mai mica valoare a lui k astfel ıncat1

21+

2

21+

3

21+ · · · + k

21sa fie un numar natural.(Clasa a IV-a) Ionela Baragan, eleva, Iasi

P.183. Se considera noua numere naturale a, b, c, . . . , i. Media aritmetica a nu-merelor a si b este 1, media numerelor c, d si e este 5, iar media numerelor f, g, h si ieste 11. Aflati media aritmetica a numerelor a, b, c, . . . , i si 9.(Clasa a IV-a) Ionel Nechifor, Iasi

1Se primesc solutii pana la data de 31 iunie 2010.

161

Page 80: Revista (format .pdf, 5.8 MB)

Clasa a V-aV.109. Aflati catul si restul ımpartirii numarului 3 · 22009 la 5 · 22007.

Damian Marinescu, Targoviste

V.110. Determinati patru numere naturale x, y, z, t cu proprietatea ca 2x−1

+ 3 · 22y+1 + 5 · 23z+2 + 11 · 25t+1 = 2009.Catalina Dragan, Galati

V.111. Demonstrati ca numarul 20200 are 261 de cifre la scrierea ın baza 10.Geanina Havarneanu, Iasi

V.112. Demonstrati ca multimea A =

§x =

3n+ 4

4n+ 3

n ∈ N, 1000 ≤ n ≤ 2009

ªare

1010 elemente.Daniela Munteanu, Iasi

V.113. Daca S = 1 +1

2+

1

3+ . . .+

1

2009, demonstrati ca S >

13

2.

Al. Gabriel Mırsanu, Iasi

V.114. Se considera ın plan cinci drepte distincte, care ımpart planul ın maimulte regiuni. Aratati ca oricum am alege 2009 puncte din plan, vor exista cel putin126 de puncte dintr-o aceeasi regiune.

Nicolae Ivaschescu, Craiova

V.115. O multime de numere naturale A = a1, a2, . . . , a9 are elementelearanjate strict crescator; media aritmetica a primelor doua elemente este 1, mediaurmatoarelor trei este 5, iar media ultimelor patru este 11. Cate astfel de multimiexista?

Ionel Nechifor si Gabriel Popa, Iasi

Clasa a VI-aVI.109. Determinati a, b, c, d ∈ N pentru care 2a+ 3b+ 5c+ 7d = 87, daca:a) a, b, c, d sunt numere prime;b) a, b, c, d sunt patrate perfecte.

Nicolae Ivaschescu, Craiova

VI.110. Determinati perechile de numere naturale care au suma 2009 si produsulmultiplu al numarului 2009.

Dan Popescu, Suceava

VI.111. Demonstrati ca numarul A = 40! ·1 +

1

2+ . . .+

1

40

este natural,

divizibil cu 2009 · 72 (unde 40! = 1 · 2 · . . . · 40).Mihai Haivas, Iasi

VI.112. Fie a, b, c, d ∈ N∗ astfel ıncat ad+ bc = bd. Demonstrati ca

a2009

b2009+

a2008c

b2008d+ . . .+

a2c

b2d+

ac

bd+

c

d∈ N.

Catalin Budeanu, Iasi

162

Page 81: Revista (format .pdf, 5.8 MB)

VI.113. Dupa doua reduceri succesive, pretul unui frigider scade de la 2000 lei la1620 lei. Stiind ca cele doua reduceri sunt proportionale cu preturile ramase ın urmalor, aflati pretul frigiderului dupa prima reducere.

Ciprian Baghiu, Iasi

VI.114. Pe laturile [BC], [AC], [AB] ale triunghiului isoscel ABC (AB = AC)

consideram puncteleD,E, respectiv F , astfel ıncatm(ÕBAD) = 2m(ÕEDC) sim(ÕDAC)

= 2m(ÕFDB). Demonstrati ca AEF este isoscel.Doru Buzac, Iasi

VI.115. Dreptele a si b sunt perpendiculare pe segmentul [AB] ın A, respectivın B. Consideram punctele C ∈ (AB), M ∈ a, N,P ∈ b astfel ıncat ıntre oricaredoua dintre triunghiurile ACM,BCN si BCP exista cate o congruenta. Stiind ca

m(ÕBPC) = 25, determinati masurile unghiurilor triunghiului MNP.Andrei Nedelcu, Iasi

Clasa a VII-aVII.109. Fie ABCD dreptunghi, O mijlocul lui [AC], M ∈ (AO), N ∈ (OC),

P = BM ∩ AD si Q = BN ∩ CD. Demonstrati ca O este centrul de greutate al

triunghiului BPQ daca si numai daca OM = ON =1

6AC.

Petru Asaftei, Iasi

VII.110. Masurile unghiurilor A,B si C ale triunghiului ABC sunt direct propor-tionale cu 5, 4 si 3, iar BC = (2+ 2

√2+ 2

√3) cm. Demonstrati ca perimetrul si aria

triunghiului sunt numeric egale.Constantin Apostol, Rm. Sarat

VII.111. Fie ABC un triunghi si punctele D ∈ (AC), E ∈ (AB), P = BD ∩

CE. DacaDA

DC= k, demonstrati ca k

PC

PE− (k + 1)

PD

PB= 1.

Neculai Roman, Mircesti (Iasi)

VII.112. Fie ABCD trapez cu baza mare [AB], E = AD ∩ BC, O =AC ∩ BD, iar OP∥AB, cu P ∈ (AD). Demonstrati ca CP si CE sunt bisectoarele

(interioara, respectiv exterioara) unghiului ÕACD, daca si numai daca AB = AC.Claudiu Stefan Popa, Iasi

VII.113. a) Demonstrati ca√b2 − bc+ c2 ≥

√3 · bc√

b2 + bc+ c2, ∀b, c ∈ R∗

+.

b) Considerand un triunghi ABC cu m( bA) = 120, AB = c, AC = b, interpretatigeometric inegalitatea de la a).

Dan Mocanu, elev, Iasi

VII.114. Demonstrati ca produsul a doua numere naturale nenule consecutivenu poate fi egal cu produsul altor patru numere naturale consecutive.

Mihai Craciun, Pascani

VII.115. Demonstrati ca1

n+ 1+

1

n+ 2+ · · ·+ 1

4k · n> k, ∀n, k ∈ N∗.

Cosmin Manea si Dragos Petrica, Pitesti

163

Page 82: Revista (format .pdf, 5.8 MB)

Clasa a VIII-aVIII.109. Fie ABCDA′B′C ′D′ un cub de muchie a. Notam cu E,F,G,H,K,L

mijloacele muchiilor AB,BC,CC ′, C ′D′, D′A′, respectiv A′A. Calculati volumul poli-edrului B′EFGHKL.

Adrian Corduneanu, Iasi

VIII.110. Fie V ABCD piramida patrulatera regulata. Notam cu

u = m( Û(V BC), (ABC)), v = m( Û(V BC), (V CD)) si t = m( Û(V BC), (V AD)). Stabilitidaca printre numerele u, v, t pot exista perechi de numere egale. (In legatura cuVIII.98 din RecMat 2/2008.)

Claudiu Stefan Popa, Iasi

VIII.111. Fie ABC un triunghi de laturi a, b, c, astfel ıncat b + c = a√2.

Demonstrati ca triunghiul este ascutitunghic daca si numai daca b si c sunt distincte

si se afla ın intervalul

a√2

4,3a

√2

4

.

Romanta Ghita si Ioan Ghita, Blaj

VIII.112. Fie x, y ∈ R∗ astfel ıncat xy,x

ysi yÈx2 + (x+ 1)2 + x2(x+ 1)2 sunt

toate numere rationale. Demonstrati ca x si y sunt tot numere rationale.Dan Nedeianu, Drobeta Tr. Severin

VIII.113. Daca a, b, c ∈ R∗+, demonstrati ca

1

a+1

b+1

c= 2

1

a+ b+

1

b+ c+

1

c+ a

daca si numai daca a = b = c.

D.M. Batinetu-Giurgiu, Bucuresti

VIII.114. Demonstrati ca oricare ar fi numerele naturale impare m,n cu m >n+ 2, exista numere naturale x, a, b astfel ıncat x = a(a+m) = b(b+ n).

Titu Zvoranu, Comanesti

VIII.115. Demonstrati ca 5(a2 + b2)2 ≤ 4a4b4 + (a+ b)4,∀a, b ∈ [1,+∞).Lucian Tutescu si Ion Visan, Craiova

Clasa a IX-aIX.101. Prin inductie matematica se arata ca are loc inegalitate a lui Bernoulli

(1) (1 + x)n ≥ 1 + nx,∀n ∈ N, n ≥ 2 si ∀x ∈ [−1,∞), egalitatea fiind atinsa pentru x = 0. Aratati ca:

a) daca n = 2k, k ∈ N∗, atunci (1) are loc ∀x ∈ R;b) daca n = 2k + 1, k ∈ N∗, atunci (1) are loc ∀x ∈ [−2,∞);c) daca n = 3, atunci (1) are loc ∀x ∈ [−3,+∞), cu egalitate cand x ∈ −3, 0,

iar pentru x ∈ (−∞,−3), (1) are loc cu sens contrar.Dorin Dutkay, Orlando (U.S.A.) si Florin Popovici, Brasov

IX.102. Rezolvati ın R3 sistemul:

x+y+z = 2− 1

x− 1

y− 1

z;x2+y2+z2 = 6− 1

x2− 1

y2− 1

z2;x3+y3+z3 = 2− 1

x3− 1

y3− 1

z3.

Vasile Chiriac, Bacau

164

Page 83: Revista (format .pdf, 5.8 MB)

IX.103. Fie x, y, z ∈ R cu 0 ≤ x ≤ y ≤ z. Daca α ∈ R este astfel ıncatαx+ (1− α)z ≥ 0, demonstrati ca αx+ (1− α)y ≥ 0 si αy + (1− α)z ≥ 0.

Ovidiu Pop, Satu Mare

IX.104. Fie A,B,C,D patru puncte ale cercului C(O, r), M = AB ∩ CD,N si P mijloacele coardelor [AB], respectiv [CD], iar Ω cel de-al patrulea varf alparalelogramului NMPΩ.

a) Aratati ca−−→MA+

−−→MB +

−−→MC +

−−→MD = 2

−−→MΩ.

b) Paralelele prin C si D la AB si paralelele prin A si B la CD se taie doua catedoua ın patru puncte ce determina un paralelogram de centru Ω.

c) Ω = O daca si numai daca AB ⊥ CD.Diana Vranceanu, eleva si Dumitru Mihalache, Barlad

IX.105. Intr-un triunghi, cu notatiile uzuale, demonstrati echivalenta conditiilor:(i) R = ra; (ii) cosA = cosB + cosC.

Temistocle Bırsan, Iasi

Clasa a X-a

X.101. Calculati suma S =nX

k=1

arctg 1√2k+1

arcsin√2k+1k+1

.

Bencze Mihaly, Brasov

X.102. Rezolvati ecuatia1

2x+ log2

x− 7

4

+

7

4= 0.

Eugen Jecan, Dej

X.103. Fie S,U,A trei puncte distincte. Rotind vectorul−→SA ın jurul lui S, cu

un arc α ∈ (−π, π), obtinem punctul S′; rotind apoi−→UA ın jurul lui U , cu un arc

β ∈ (−π, π), obtinem U ′, U ′ = S′. Fie M ∈ S′U ′ astfel ıncat−−→S′M = k ·

−−−→MU ′, unde

k ∈ R\0,−1. Demonstrati ca pozitia punctului M nu depinde de A atunci si numaiatunci cand k = 1, β = α± π*.

Diana Vranceanu, eleva si Dumitru Mihalache, Barlad

X.104. Fie p, la, lb, lc semiperimetrul, respectiv lungimile bisectoarelor unui tri-unghi. Determinati numerele reale α si β, ın functie de p, stiind ca solutiile ecuatieix3 − p

√3 · x2 + αx− β = 0 sunt la, lb si lc.

Catalin Calistru, Iasi

X.105. Determinati cel mai mare numar real α astfel ıncat inegalitatea

sinx+ sin y

2≤ α ·

sin x+3y4 + sin 3x+y

4

2+ (1− α) sin

x+ y

2sa fie adevarata pentru orice x, y ∈ [0, π].

Marian Tetiva, Barlad

Clasa a XI-a

XI.101. Pentru a ∈ R∗+, calculati lim

x→∞x

1 +a

x

x− ea

.

D.M. Batinetu-Giurgiu, Bucuresti

*Generalizare a problemei comorii din insula a lui G. Gamow, din One, Two, Theree . . . Infinity.

165

Page 84: Revista (format .pdf, 5.8 MB)

XI.102. Determinati functiile continue f : R → R cu proprietatea ca f(x2) −f(y2) = (x+ y)f(x− y),∀x, y ∈ R.

Gheorghe Iurea, Iasi

XI.103. Fie (xn)n≥1 ⊂ R∗+ astfel ıncat lim

n→∞n(xn+1−xn) = a ∈ (1,+∞); definim

sirul (yn)n≥1 prin yn =n lnn

x1 + x2 + . . .+ xn. Calculati lim

n→∞(yn)

n.

Cosmin Manea si Dragos Petrica, Pitesti

XI.104. Calculati limn→∞

15

51+

25

52+

35

53+ . . .+

n5

5n

.

Neculai Roman, Mircesti (Iasi)

XI.105. Consideram matricele Ak, Bk ∈ Mk(C), k ∈ N\0, 1, astfel ıncatdetAk = α ∈ C∗, ∀k ∈ N\0, 1. Studiati convergenta sirului (an)n≥2 definit prin

an = limx→∞

nXk=2

det(Akx+Bk)

k! · xk.

Catalin Calistru, IasiClasa a XII-a

XII.101. Rezolvati ecuatia x2 + x+ 1 = 0 ın Z13 si ın Z19, apoi deduceti ca 247divide (37

n−1 − 1)(74n−1 − 1), oricare ar fi n ∈ N.

Mihai Haivas, Iasi si I.V. Maftei, Bucuresti

XII.102. Determinati primitivele functiei f :0,

π

4

→ R, f(x) =

cos 2x

(tg x+ ctgx)2009.

Nicoleta Bran, Craiova

XII.103. Demonstrati ca exista c ∈0,

π

4

pentru careZ π

4

0(etg x − 1)dx ≤ π2

32· etg c

cos2 c.

Bogdan Victor Grigoriu, Falticeni

XII.104. Determinati functiile derivabile f : [0, 1] → R pentru care f(0) = 0, iarf ′(x) = f(ax),∀x ∈ [0, 1], cu a ∈ [0, 1] fixat.

Gheorghe Iurea, Iasi

XII.105. Fie f : (0,∞) → R o functie semiconvexa (fx+ y

2

≤ f(x) + f(y)

2,

∀x, y ∈ (0,∞)).a) Demonstrati ca pentru orice x ∈ (0,∞), sirul (fn(x))n≥1 definit prin fn(x) =

n

f

x+

1

n

− f(x)

, este monoton.

b) Deduceti ca pentru orice x ∈ (0,∞), sirul (en(x))n≥1 definit prin en(x) =1 +

1

nx

n

, este crescator.

Dan Stefan Marinescu si Viorel Cornea, Hunedoara

166

Page 85: Revista (format .pdf, 5.8 MB)

Probleme pentru pregatirea concursurilor

A. Nivel gimnazial

G166. Demonstrati ca urmatoarele propozitii sunt adevarate.a) ∀n ∈ N, n ≥ 2, ∃x1, x2, . . . , xn ∈ N∗ astfel ıncat x1x2 + x2x3 + . . . + xnx1 =

x1x2 . . . xn.b) ∀n ∈ N, n ≥ 5, @x1, x2, . . . , xn ∈ 2N∗ astfel ıncat x1x2 + x2x3 + . . . + xnx1 =

x1x2 . . . xn.c) ∃x1, x2, . . . , xn ∈ 2N+ 1 astfel ıncat x1x2 + x2x3 + . . .+ xnx1 = x1x2 . . . xn ⇔

n ∈ 2N∗ + 1.Dan Popescu, Suceava

G167. Fie 1 = d1 < d2 < . . . < dk = n toti divizorii pozitivi ai numarului naturaln. Daca exista i, j cu j > i > 13 si d27 + d2i = d2j , aratati ca n este multiplu de 8.

Titu Zvonaru, Comanesti

G168. Pentru x, y, z ∈ R∗+, demonstrati ca are loc inegalitatea

x(y + z)

x+ yz+

y(x+ z)

y + xz+

z(x+ y)

z + xy≤ 2

x2

x+ yz+

y2

y + xz+

z2

z + xy

.

Stefan Gavril, Piatra Neamt

G169. Demonstrati ca exista o infinitate de numere irationale α cu proprietateaca α3 si α2 + α sunt, de asemenea, irationale.

Gabi Ghidoveanu si Dumitru Mihalache, Barlad

G170. O multime A ⊂ R, de cardinal 2009, are proprietatea ca fiecare elemental ei este mai mare decat o zecime din suma celor 2008 numere ramase. Aratati ca Acontine cel putin 12 numere negative.

Andrei Nedelcu, Iasiy

x0

G171. Punctele planului care au, ın raport cu un repercartezian, ambele coordonate numere naturale, le parcurgemın sensul indicat de sageti ın figura, pornind din origine.Notam cu an,k pozitia punctului de coordonate (n, k) ın sirulobtinut (de exemplu, a0,0 = 1, a0,2 = 4, a2,2 = 13 etc.).Exprimati numarul an,k ın functie de n si de k.

Lucian Georges Ladunca, Iasi

G172. O tabla dreptunghiulara m × n, m, n ≥ 2, are patratelele unitate de laintersectiile liniilor de ordin impar cu coloanele de ordin impar colorate ın negru,restul patratelelor fiind albe. A recolora o linie (coloana) ınseamna a schimba culoriletuturor patratelelor acelei linii (coloane). Aratati ca tabla nu poate fi transformataıntr-una complet alba prin recolorarea catorva linii si coloane.

Razvan Ceuca, elev, Iasi

G173. Notam cu T (a, b, c) triunghiul care are laturile de lungimi a, b si c. Demon-strati ca triunghiurile T (b, 2c, 2mb) si T (c, 2b, 2mc) pot fi confectionate (pe rand)dintr-o aceeasi bucata de carton, fara pierdere de material.

Petru Asaftei, Iasi

167

Page 86: Revista (format .pdf, 5.8 MB)

G174. Se considera triunghiul ABC isoscel cu m( bA) = 40. Sa se arate ca

nu exista puncte P ∈ IntABC pentru care m(ÕPAB) = 30, m(ÕPBC) = 10 si

m(ÕPCA) = 35.Gabriel Popa si Paul Georgescu, Iasi

G175. Fie ABCD un patrulater ınscris ın cercul de raza R. Demonstrati caAB ·AD + CB · CD ≤ 4R2.

Gheorghe Costovici, Iasi

B. Nivel liceal

L166. Fie ABCD un dreptunghi, iar C un cerc prin A, care intersecteaza (AB),(AC) si (AD) ın M,N , respectiv P . Aratati ca AM ·AB +AP ·AD = AN ·AC.

Gheorghe Iurea, Iasi

L167. Fie ABC un triunghi cu AB > AC. Cercul ınscris ın triunghi este tangentlaturilor BC si AC ınD, respectiv E. Consideram T un punct pe latura [BC] si notamcu J centrul cercului ınscris ınABT. DacaDE trece prin mijlocul segmentului [CJ ],demonstrati ca triunghiul ATC este isoscel.

Titu Zvonaru, Comanesti

L168. Demonstrati ca ın orice triunghi, cu notatiile uzuale, are loc inegalitateaa

b+ c+

b

c+ a+

c

a+ b≥ 11p2 − 15r2 − 60Rr

6p2 − 6r2 − 24Rr≥ 3

2.

Marius Olteanu, Rm. Valcea

L169. Care este probabilitatea ca razele cercurilor exınscrise unui triunghi alesaleator, sa fie laturile unui nou triunghi?

Petru Minut, Iasi

L170. Fie n ∈ N, n ≥ 2 si a1, a2, . . . , an ∈ R+ cu a1+a2+. . .+an = S. Consideramk ∈ N, 1 ≤ k ≤ n− 1 si α1, α2 ∈ R+ cu α1 + α2 = 1. Demonstrati inegalitateaX

1≤i1<...<ik≤n

(ai1 + ai2 + . . .+ aik)α1(S − ai1 − . . .− aik)

α2 ≤ kα1(n− k)α2

nCk

nS.

(In legatura cu 6117 din R.M.T. 1/1987 )Paul Georgescu si Gabriel Popa, Iasi

L171. Pentru x, y ∈ R∗+, demonstrati ca are loc inegalitatea

√x+ 3

È2(x+ y) +

√y ≤ 2(

p3x+ y +

px+ 3y).

Marian Tetiva, Barlad

L172. Fie P ∈ Q[X], cu grP = n ≥ 1. Daca P admite o radacina complexa a,avand ordinul de multiplicitate m, cu n < 2m, demonstrati ca a ∈ Q.

Adrian Reisner, Paris

L173. a) Exista functii f : (a, b) → R cu proprietatea ca |f(x) − f(y)| ≥ c,∀x, y ∈ (a, b), unde c este o constanta pozitiva?

168

Page 87: Revista (format .pdf, 5.8 MB)

b) Exista functii f : (a, b) ∩ Q → R cu proprietatea ca |f(x) − f(y)| ≥ c, ∀x, y ∈(a, b) ∩Q, unde c este o constanta pozitiva?

Geanina Havarneanu, Iasi

L174. Fie a1, a2, . . . , an, b1, b2, . . . , bn numere reale pozitive si a =

nX

i=1

√ai

!2

,

b =

nX

i=1

√bi

!2

. Aratati ca exista x0 > 0 astfel ıncat

"nX

i=1

paix+ bi

#− [

√ax+ b] ∈

0, 1, ∀x > x0.Marian Tetiva, Barlad

L175. Aratati ca[n2 ]Xk=0

C2kn Ωk = 2nΩn, n ∈ N,

unde Ωk =(2k − 1)!!

(2k)!!, k ∈ N∗ (se convine ca Ω0 = 1).

Gheorghe Costovici, Iasi

Training problems for mathematical contests

A. Junior highschool level

G166. Prove that the following assertions are true:a) ∀n ∈ N, n ≥ 2, ∃ x1, x2, . . . , xn ∈ N∗ such that x1x2+x2x3+xnx1 = x1x2 . . . xn.b) ∀n ∈ N, n ≥ 5, @ x1, x2, . . . , xn ∈ 2N∗ such that x1x2 + x2x3 + xnx1 =

x1x2 . . . xn.c) ∃ x1, x2, . . . , xn ∈ N∗ such that x1x2+x2x3+xnx1 = x1x2 . . . xn ⇔ n ∈ 2N∗+1.

Dan Popescu, Suceava

G167. Let d1 < d2 < . . . dk = n be all the positive divisors of the natural numbern. Assuming that the subscripts i, j with j > i > 13 exist such that d27 + d2i = d2j ,show that n is a multiple of 8.

Titu Zvonaru, Comanesti

G168. For any x, y, z ∈ R∗+, prove that the following inequality holds :

x(y + z)

x+ y z+

y(x+ z)

y + x z+

z(x+ y)

z + x y≤ 2

x2

x+ y z+

y2

y + x z+

z2

z + x y

.

Stefan Gavril, Piatra Neamt

G169. Prove that an infinity of irrational numbers α exist with the property thatα3 and α2 + α are irrational numbers as well.

Gabi Ghidoveanu and Dumitru Mihalache, Barlad

169

Page 88: Revista (format .pdf, 5.8 MB)

G170. A subset A ⊂ R, of cardinal number 2009, has the property that eachof its elements is greater than one tenth of the sum of the remaining 2008 numbers.Show that A contains at least 12 negative numbers.

Andrei Nedelcu, Iasi

y

x0

G171. The points in the plane whose both coordinates,in a Cartesian system of coordinates, are natural numbers,visited along the path indicated by the arrows in the figure atright, starting from the origin. We denote by an,k the positionof the point of coordinates (n, k) in the sequence thus obtained(for instance, a0,0 = 1, a0,2 = 4, a2,2 = 13, etc.). Express thenumber an,k in terms of n and k.

Lucian Georges Ladunca, Iasi

G172. A rectangular board of size m × n, m, n ≥ 2, has unit squares at theintersections of odd-order rows with odd-order columns, colored in black while theother squares remain white. To re-color a row (column) means to change the colorof all the squares on that row (column). Show that the board cannot be turned to acompletely white board by recoloring a couple of rows and columns.

Razvan Ceuca, school student, Iasi

G173. We denote by T (a, b, c) the triangle whose side lengths are a, b, c. Provethat the triangles T (b, 2c, 2mb) and T (c, 2b, 2mc) can be successively manufacturedfrom a single (the same) cardboard sheet without losses of material.

Petru Asaftei, Iasi

G174. The isosceles triangle ABC is considered with m( bA) = 400. Show that

no points P ∈Int(ABC) exist such that m(ÕPAB) = 300, m(ÕPBC) = 100 and

m(ÕPCA) = 350.

Gabriel Popa and Paul Georgescu, Iasi

G175. Let ABCD be a quadrilateral inscribed in the circle of radius R. Provethat AB ·AD + CB · CD ≤ 4R2.

Gheorghe Costovici, Iasi

B. Highschool level

L166. Let ABCD be a rectangle, and C a circle passing through A whichintersects the lines (AB), (AC) and (AD) at the points M,N and P , respectively.Show that AM ·AB +AP ·AD = AN ·AC.

Gheorghe Iurea, Iasi

L167. Let ABC be a triangle with AB > AC. The circle inscribed in the triangleis tangent to the sides BC and AC at D and respectively E. We consider a point Ton the side [BC] and denote by J the center of the circle inscribed in ABT . IfDE passes through the midpoint of the segment [CJ ], prove that the triangle ATCis isosceles.

Titu Zvonaru, Comanesti

170

Page 89: Revista (format .pdf, 5.8 MB)

L168. Prove that in any triangle, with the usual notations, the following inequa-lity holds :

a

b+ c+

b

c+ a+

c

a+ b≥ 11 p2 − 15 r2 − 60Rr

6 p2 − 6 r2 − 24Rr≥ 3

2.

Marius Olteanu, Rm. Valcea

L169. What is the probability that the radii of the excircles to a randomly chosentriangle be the sides of a new triangle?

Petru Minut, Iasi

L170. Let n ∈ N, n ≥ 2 and a1, a2, . . . , an ∈ R+ with a1 + a2 + · · · + an = S.We consider k ∈ N, 1 ≤ k ≤ n − 1 and α1, α2 ∈ R with α1 + α2 = 1. Prove theinequality P

1≤ i1< i2<···< ik≤n

(ai1 + ai2 + · · ·+ aik)α1 (S − ai1 − · · · − aik)

α2 ≤

≤ k α1 (n− k)α2

n

nk

S.

(Connected with 6117 of R.M.T. 1/1987 ).Paul Georgescu and Gabriel Popa, Iasi

L171. For any x, y ∈ R∗+, prove that the following inequality holds :

√x+ 3

È2(x+ y) +

√y ≤ 2

p3x+ y +

px+ 3 y

.

Marian Tetiva, Barlad

L172. Let P ∈ Q [X] with degP = n ≥ 1. If P admits a complex root a withits multiplicity m such that n < 2m, prove that a ∈ Q.

Adrian Reisner, Paris

L173. a) Do exist functions f : (a, b) → R with the property that |f (x)− f (y)| ≥c for ∀x, y ∈ (a, b), where c is a positive constant?

b) Do exist functions f : (a, b)∩Q → R with the property that |f (x)− f (y)| ≥ cfor ∀x, y ∈ (a, b) ∩Q, where c is a positive constant ?

Geanina Havarneanu, Iasi

L174. Let a1, a2, . . . , an, b1, b2, . . . , bn be positive real numbers and let a = nX

i=n

√ai

!2

, b =

nX

i=n

pbi

!2

. Show that x0 > 0 exists such that"nX

i=n

paix+ bi

#−√

ax+ b∈ 0, 1 , for ∀x > x0.

Marian Tetiva, Barlad

L175. Show that[n2 ]Xk=0

C2kn Ωk = 2n Ωn

where Ωk =(2k − 1)!!

(2k)!!, k ∈ N∗; by conventionΩ0 = 1.

Gheorghe Costovici, Iasi

171

Page 90: Revista (format .pdf, 5.8 MB)

Pagina rezolvitorilor

COVASNAScoala cu clasele I-VIII ”Avram Iancu”. Clasa a V-a (prof. LAZAR Emese).

BARLA Oana: P(161,163), V(95,96), VI.96; PATRANJEL Andrada-Maria: P(161,163), V(95,96), VI.96; TIMARU Carmen-Ioana: P(161,163), V(95,96), VI.96; UTAIoana: P(161, 163), V(95,96), VI.96; VLADEA Diana: P(161,163), V(95,96), VI.96.

CRAIOVALiceul Teoretic ”Tudor Arghezi”. Clasa a V-a (prof. DRACEA Dorina). VIRLAN

Leonard: P.161, V(95-97).

IASIScoala nr. 3 ”Al. Vlahuta”. Clasa I (inst. MAXIM Gabriela). CUCURUZ

Raluca: P(154,155, 157, 164-167); DASCALU Lorena: P(154,155,157,164-167); PO-PESCU Alexandru: P(154,157,164-167); ROBU Carmen: P(154,157,164-167); SER-BANOIU Alexandru: P(154,157,164-167); TORAC George: P(154,157,164-167);NICA Daniel: P(154,155,157,164-167). Clasa a II-a (inst. CRACIUN Marilena).CREANGA Adrian: P(154,155,157,164-167); FILIP Vlad: P(154,155,157,164-167);POPESCU Claudia: P(154,155,157,164-167). Clasa a II-a (inv. MARIUTA Valen-tina). ENEA Codrut Alexandru: P(154, 155, 157, 164-167); GHEORGHIU Beatrice-Elena: P(154,155,157,164-167); HERGHELEGIU Madalina: P(154,155,157,164-167);HREAPCA Alin: P(154,155,157,164-167); HUHU Paula: P(154,155,157,164-167);POPOVICIU Teodor-Andrei: P(154,155,157,164-167); ROMILA Andreea-Maria:P(154,155,157,164-167). Clasa a VI-a (prof. MARIN Mirela). MARCU Ana:V(105,106), VI(102,104,105); RUSUMadalina-Andreea: V(105,106), VI(102,104,105);TIBA Stefana-Alexandra: V(105,106), VI(102,104,105).

Scoala nr. 13 ”Alexandru cel Bun”. Clasa I (inst. COJOCARIU Ana). ACA-TRINEI Andra: P(154,164-167); PERDUN Patricia-Maria: P(154,164-167); PRISE-CARU Alexandru-Julian: P(154,164-167), SAMSON Constantin-Catalin: P(154,164-167); STEFAN Tudor: P(154,164-167); ZAHARIA Stefan-Eusebiu: P(164,164-167).

Scoala nr. 14 ”Gh. Marzescu”. Clasa a IV-a (inst. NUTA Elena). BACIU Tu-dor: P(164-167,170); CHIRILUTA George-Stefan: P(164-167,169-171); NEDELEUIulia: P(164-167,170); POSTUDOR Georgiana-Madalina: P(164-167,170); TAMBA-LARIU Ioana-Vasilica: P(164-167,170).

Scoala nr. 22 ”B.P. Hasedeu”. Clasa I (inv. BLAJUT Cristina). SAVA CosminaIoana: P(164-167,169). Clasa a IV-a (inv. STEFAN Liviu). BLAJUT Cristin-Marian: P(167-168).

Scoala nr. 26 ”George Cosbuc”. Clasa a II-a (inst. VARLAN Elena). AMARIEIRomeo: P(154-157,164-167); GHEBAN Andreea: P(154-157,164-167); PAVALUCAna-Maria: P(154-157,164-167); PICHIU Cosmin: P(154-157,164-167); TATARUAlice: P(154,157,164-167); TIPLEA Iulian: P(154-157,164-167). Clasa a III-a(inv. BUCATARIU Rica). BARHAN Stefana-Adina: P(164-166,168,169,171,172);CHIRIAC Alexandra: P(164-166,168,169,171,172) CUPET Valeria: P(164-166,168,169,171,172); FRUNZA Andrei-David: P(164-166,168,169,171,172); FRUNZA Diana-Mihaela: P(164-169,171,172); IVANOV Alexandra: P(164-166,168,169,171,172);

172

Page 91: Revista (format .pdf, 5.8 MB)

MINDRU Liana: P(164-169,171,172). Clasa a IV-a (inst. RACU Maria). APA-CHITEI Aura-Georgiana: P(164-171); BURA Emma-Andreea: P(164-170); CRA-CIUN Ioana-Daniela: P(164-171); LESOVSCHI Alexandra-Ioana: P(164-169); LUPURoxana-Elena: P(164-169).

Scoala SAM ”M. Kogalniceanu”, Tiganasi. Clasa a II-a (ınv. GALIA Paraschiva).CAZADOI Ioana-Cristina: P(154,155,157,164-167); DUCA Cristina Mihaela: P(154-155,157,164-167); SANDU Rebeca: P(154-155,157,164-167).

SFANTU GHEORGHE (Tulcea)Scoala generaka cu clasele I-VIII.Clasa a II-a (ınv. GAVRILA Elena). HALCHIN

Ioana: P(154-158); SIDORENCUAdrian: P(154-158). Clasa a IV-a (ınv. GAVRILAElena). BALAN Silviu: P(154-161); CLADIADE Bogdan-Robert: P(154-161); CUCUDelia: P(154-161); EFIMOV Cosmin Alexandru: P(154-161). Clasa a VI-a (prof.SAILEANU Sorin). SIDORENCU Andrei: V(95,96), VI.96, VII(96,98), VIII.96.

Elevi rezolvitori premiati

Scoala nr. 26 ”G. Cosbuc”, Iasi

1. LESOVSCHI Alexandra-Ioana (cl. a IV-a): 2/2008(6pb), 1/2009(6pb), 2/2009(6pb).

2. LUPU Roxana-Elena (cl. a IV-a): 2/2008(6pb), 1/2009(6pb), 2/2009(6pb).

Vizitati noua pagina web a revistei:

http://www.recreatiimatematice.ro

173

Page 92: Revista (format .pdf, 5.8 MB)

IMPORTANT

• In scopul unei legaturi rapide cu redactia revistei, pot fi utilizate urmatoareleadrese e-mail: t [email protected] si [email protected] . Peaceasta cale colaboratorii pot purta cu redactia un dialog privitor la ma-terialele trimise acesteia, procurarea numerelor revistei etc. Sugeram cola-boratorilor care trimit probleme originale pentru publicare sa le numerotezesi sa-si retina o copie xerox a lor pentru a putea purta cu usurinta o discutieprin e-mail asupra acceptarii/neacceptarii acestora de catre redactia revistei.

• La problemele de tip L se primesc solutii de la orice iubitor de matematicielementare (indiferent de preocupare profesionala sau varsta). Fiecare dintresolutiile acestor probleme - ce sunt publicate ın revista dupa un an - va fiurmata de numele tuturor celor care au rezolvat-o.

• Adresam cu insistenta rugamintea ca materialele trimise revisteisa nu fie (sa nu fi fost) trimise si altor publicatii.

• Rugam ca materialele tehnoredactate sa fie trimise pe adresa redactieiınsotite de fisierele lor (de preferinta ın LATEX).

• Pentru a facilita comunicarea redactiei cu colaboratorii ei, autorii materi-alelor sunt rugati sa indice adresa e-mail.

174

Page 93: Revista (format .pdf, 5.8 MB)

Revista semestrială RECREAŢII MATEMATICE este editată de ASOCIAŢIA “RECREAŢII MATEMATICE”. Apare la datele de 1 martie şi 1 septembrie şi se adresează elevilor, profesorilor, studenţilor şi tuturor celor pasionaţi de matematica elementară.

În atenţia tuturor colaboratorilor

Materialele trimise redacţiei spre publicare (note şi articole, chestiuni de metodică, probleme propuse etc.) trebuie prezentate îngrijit, clar şi concis; ele trebuie să prezinte interes pentru un cerc cât mai larg de cititori. Se recomandă ca textele să nu depăşească patru pagini. Evident, ele trebuie să fie originale şi să nu fi apărut sau să fi fost trimise spre publicare altor reviste. Rugăm ca mate-rialele tehnoredactate să fie însoţite de fişierele lor.

Problemele destinate rubricilor: Probleme propuse şi Probleme pentru pregătirea concursurilor vor fi redactate pe foi separate cu enunţ şi demonstra-ţie/rezolvare (câte una pe fiecare foaie) şi vor fi însoţite de numele autorului, şcoa-la şi localitatea unde lucrează/învaţă.

Redacţia va decide asupra oportunităţii publicării materialelor primite.

În atenţia elevilor

Numele elevilor ce vor trimite redacţiei soluţii corecte la problemele din rubricile de Probleme propuse şi Probleme pentru pregatirea concursurilor vor fi menţionate în Pagina rezolvitorilor. Se va ţine seama de regulile:

1. Pot trimite soluţii la minimum cinci probleme propuse în numărul prezent şi cel anterior al revistei; pe o foaie va fi redactată soluţia unei singure probleme.

2. Elevii din clasele VI-XII au dreptul să trimită soluţii la problemele propuse pentru clasa lor, pentru orice clasă mai mare, din două clase mai mici şi imediat anterioare. Elevii din clasa a V-a pot trimite soluţii la problemele propuse pentru clasele a IV-a, a V-a şi orice clasă mai mare, iar elevii claselor I-IV pot trimite soluţii la problemele propuse pentru oricare din clasele primare şi orice cla-să mai mare. Orice elev poate trimite soluţii la problemele de concurs (tip G şi L).

3. Vor fi menţionate următoarele date personale: numele şi prenumele, clasa, şcoala şi localitatea, precum şi de numele profesorului cu care învaţă.

4. Plicul cu probleme rezolvate se va trimite prin poştă (sau va fi adus direct) la adresa Redacţiei:

Prof. dr. Temistocle Bîrsan Str. Aurora, nr. 3, sc. D, ap. 6, 700 474, Iaşi Jud. IAŞI E-mail: [email protected]

Page 94: Revista (format .pdf, 5.8 MB)

CUPRINS Către cititori dupǎ zece ani de apariţie a revistei .......................................................................... 85 NECULAI GHEORGHIU (1930-2009) ............................................................................................ 87

ARTICOLE ŞI NOTE T. BÎRSAN, G. DOSPINESCU – Conjectura Beal pentru polinoame ..................................... 89 C.-L. BEJAN – Teorema lui Brouwer - un caz particular elementar ........................................ 94 D. MĂRGHIDANU – Exponentul de triangularitate al unui triunghi ...................................... 96 D. POPESCU – Asupra unui şir de integrale Riemann ............................................................. 98 A. VERNESCU – Dualitatea unor sume combinatoriale ........................................................ 101

NOTA ELEVULUI D. M. MOCANU – Asupra problemei C.O: 5004 din G.M. ................................................... 105

CORESPONDENŢE A. REISNER – Matrices à coefficients dans un corps fini ..................................................... 107

CHESTIUNI METODICE M. MIHEŢ – O metodǎ de rezolvare a problemelor ..............................................................111 S. BOGA – Metoda identificǎrii.............................................................................................115

CUM CONCEPEM ... CUM REZOLVĂM D. MIHALACHE, M. TETIVA– Bisectoarele exterioare nu sunt ca bisectoarele interioare..117

MATEMATICA ÎN CLASELE PRIMARE D.M. BĂTINEŢU-GIURGIU – Metoda falsei ipoteze - variante de utilizare.........................121

ŞCOLI ŞI DASCĂLI Liceul Teoretic "Garabet Ibrăileanu"........................................................................................123 LIDIA COHAL (1930-2009) ...................................................................................................126

CONCURSURI ŞI EXAMENE Concursul de matematică "Al. Myller", ed. a VII-a, 2009 .................................................. 127 Concursul de matematică "Florica T. Câmpan", 2009 ......................................................... 130

PROBLEME ŞI SOLUŢII Soluţiile problemelor propuse în nr. 2/2008 .......................................................................... 135 Soluţiile problemelor pentru pregătirea concursurilor din nr. 2/2008 ................................ 152 Probleme propuse ...................................................................................................................... 161 Probleme pentru pregătirea concursurilor .............................................................................. 167 Training problems for mathematical contests ....................................................................... 169 Pagina rezolvitorilor ............................................................................................................... 172 ISSN 1582 – 1765 7 lei